You are on page 1of 160
Bob HE 2008 Nguyén Van Mau (Chui bién) TOAN ROI RAC VA MOT SO VAN DE LIEN QUAN | (Tai ligu béi dudng he 2007) ) HA NOI, 05-14 ‘THANG 8 NAM 2007 ‘Trén bén muci nim thuc hién "Chuong trinh dao tao va bdi hoc sinh nang khiéu toén bac phé thong" 1A mot chang dung ciia mot chu trinh dic bit gan véi su khéi ddu, truéng thinh va ngay cing hodn thién xuét phét tir mét mé hinh do tao ning khiéu Téan hoc dae bigt tai Dai hoc Téng hyp Ha Noi. Huéng dio tao mii nhon niy mang tinh dot phé cao, da ddo tao ra céc thé hé hoc sinh c6 nang khiéu trong linh vc toan hoc, tin hoc va khoa hoc ty nhién: Vat If, Hod hoc, Sinh hoc va khoa hoc sy séng. ‘Trong ditu kign thiéu thén vé vat cht kéo dai qua nhiéu thap ky va trdi qua nhiéu thch thttc, ching ta da tim ra huéng di phi hgp, dé di lén ving chéc va én din, di tim tdi, tich luy kinh nghigm va c6 nhiéu sdng tao dng ghi nhan. Céc thé he Thiy va Trd da dinh hinh va tiép can v6i thé gidi van minh tién tién va khoa hoc hign dai, cap nh&t thong tin, sing tao phuong phép va tap dugt nghién citu. Gan v6i vige tich eye déi méi phutong phép day va hoc, chutong trinh dio tao chuyén Toén dang hung téi xy dug hé théng chuyén dé, dang né lye va da té chtte thinh cong Ky thi Olympic Toan quéc té lin thi 48, nam 2007 tai Viet Nam, dugc ban bé quéc té ca nggi. Sau gan nita thé ky hinh thinh va phét trién, c6 thé néi, gido duc mii nhon phd thong (gido duc nang khiéu) 48 thu duge nhing than tyu rye r6, duge Nha nude dau tur c6 higu qua, duge x4 hoi thita nhan va ban bé quéc té kham phuc. Céc doi tuyén quéc gia tham dit céc ky thi Olympic quéc té o6 bé day thanh tich mang tfnh én dinh va c6 tinh ké thita. Dac biét, nam nay, cdc Doi tuyén Ton vi Tin quéc gia tham dy thi Olympic quéc té da dat duge thanh tich néi bat. Doi tuyén Ton Viet Nam da von len ditng thit ba (theo sy sp xép khong chinh thttc) trong s6 95 doi tuyén cdc nude tham dy IMO48. ‘Tix nhidu nim nay, céc hé nang khiéu Todn hoc va cdc Trung THPT Chuyén thutng sit dung song song céc sich gido khoa dai tra két hap véi sich gido khoa chuyén bigt vA sich chuyén d@ cho cdc H@ THPT Chnyén. Hoc sinh céc lép nang khiéu da tiép thu t6t céc kién thite cd bin theo thi Ivgng hign hank do Bé GD va DT ban han. Hign nay, chuong trinh cai céch gido duc dang buéc vao giai doan hodn chinh bo SGK méi. Thi lugng kién thite eiing nhu trat ty kién thite co ban cé nhiing thay ddi déng ké. Céc kién thite ny dang duge cin nhée dé n6 vn nim trong khuén khé hien hanh cita céc kién thtte nding cao d6i véi céé lp chuyén ton. Vi lé d6, viéc tién hin viét hé théng céc séch chuyén dé cho céc l6p niing khiéu cn duge tién hanh khdn truong va duge xem xét toin dién tir phfa cée chuyén gia giéo duc va céc cd gido, thay giéo dang truc tiép ging day c4c lép chuyén. Duge sit cho phép cita BO GD va DT, Trudng Dai Hoc Khoa Hoe Ty Nhién, DHQGHN phéi hgp cling véi céc chuyén gia, cée nhA khoa hoc, cdc 06 gido, thy gido thudc DHSPHN, DHQG TpHCM, Vien Tosn Hoc, Hi Toan Hoc Ha Noi, Tap Chi Toan Hoc vi Tudi Tré, cée Trung THPT Chuyén, Cae $6 GD va DT... t6 chttc bai dudng cée chuyén d& nghiép vu sau dai hoc nhim béi dudng hoc sinh gidi cdc mén Toén hoc va khéi kién thitc khoa hoc tu nhién nhu 1A mot ta sich dc biet phuc vu di dudng hoc sinh gidi. Ching t6i xin gidi thigu cuén séch cia nhém céc chuyén gia, céc thay gido véi su tham gia dong déo cia céc ding nghiép tham du Trutmg h® 2007 vé chuyen dé "Tan ri rac va mot s6 vin dé lién quan". Cuén séch nay nhiim cung cép mét s6 kién thite chuyén d8 bat ding thtte 6 mie 46 kh6 vé tan rdi rac, dai s6, s6 hoc, va gidi tich. Day cing la chuyén dé va bai gidng m& cée te gid da gidng day cho hoc sinh céc doi tuyén thi Olympic Toén hoc quéc gia va qubc 8. Ching ti cing xin chan thinh cdm on c&c ban doc cho nhiing ¥ kién déng gép dé cudn séch ngay cng hoin chink. ‘Thay mat Ban Té Chite GS TSKH Nguyén Van Mau Ldi néi dau ........ Dé thj to mau va mét sé bai toan khong m4u myc Ding Huy Rud .eeeeseeceeseseeees Logic hinh thie va 4p dung Nguyén Van Mau .... Cong thife tinh s6 phan tit cha Vi Dink Hoa ... hgp cdc tap hop Mang luéi 6 vudng trén mat phing Vii Dinh Hoa ... Nguyén ly Dirichlet va mt s6 bai toan 4p dung Nguyén Duy Thai Son .. Mot sé phudng phap giai c4c bai téan té hop nang cao Dang Hing Théng ... Xay dung song anh gi Hiynh Tén Chau . m6t sé bai todn té hdp Phuong phap thiét lap hé Hiynh Tin Chau ose. seceees shite truy héi trong t6 hdp Y twéng giai va su tudng minh Idi gidi qua mot sé bai téan té hdp Lé Van Quang . Gidi thigu mot sé bai toan dai sé cé6 xudt xit tix hinh hoc Nouyén Dang Phat... Bat bién, don bién va ting dung Trin Nam Ding .. seteeeeeebeee LIT Mot s6 van dé cia Todn rai rac Nguyén Van Tién..... D6 thi to mau va mét sé bai todn khong mau muc Dang Huy Ruan Ly thuyét dé thj néi chung, dac bist dd thi to mau duge van dung dé gidi céc bai todn khong méu mu rat higu qua. Dé gidi toan thong qua dé thi cén thye hign theo hai bué Du tien xay dung dé thi dé mo ta cdc quan he, diéu kign duge phat biéu trong bai tofn. Sau dé cin cit vio céc khang dinh cita ly thuyét dé thi dé suy ra dap An. Trong phan nay chi dé cap dén toén khong méu muc. ic thong qua dé thi t6 mau dé gidi mot sé dang Déi véi méi dang todn khong mdu myc déu dua ra khing dinh tuong img vé dé thi t6 mau dé c6 thé vin dung gidi quyét hang loat bai ton thudc dang dude xét. Dé thi dugc goi la day di, néu mdi c&p dink cia né déu duge néi bing mot canh. Dé thi day di gom n dinh. Dé thi day di gdm 3 dinh (4 dinh) véi céc canh duge to mau bing cing mot mau duge goi 1a tam gic (tit giée) cing mau. 1 Dangi 1.1 Bai toén Bai toan 1 (Dé thi Olympic Toén Quéc té) Mudi bay nha bac hoc viét thu cho nhau. M6i ngudi déu viét thu cho tat cd ngudi Khéc. Cc thut chi trao déi vé 8 dé tai. Timg cap nhit bac hoc chi viét thu trao déi vé ciing mot dé tai. Ching minh ring chi cd it nhét 3 ha béc hoc wiét thu cho nhau trao déi vé cing mét vén dé. Bai ton 2 Trong mot cude giip 9 quéc té cd 17 nha ngoai giao tham gia. Méi ciip nha ngoai giao chi trao déi tryc tiép vdi nhau béing mot trong ba ngon ngi: Anh, Phdp, Ditc. Ching minh ring ludn ludn tim dude ba nha ngoai giao, ma ho c6 thé trao déi truc tiép duge bing mét trong ba ngén ngit ké trén. Bai todn 3 Méi cap déi tugng cho trudc chi cb mét trong ba quan hé: ty, ta, ta Chitng minh réng luon lun tim dude ba déi tugng, ma méi cfip trong b6 ba nay ciing 6 quan he ts (1 2) vdin mau canh (céc canh duge 16 bing n mau), sao cho khong tam gide ciing mau nao, luén ludn 06 hinh 5 canh vdi ede canh ciing mau ua chic dudng chéo duge t8 bing cic mau khéc. Chang mink. Bing quy nep theo n. 1. Co 88 quy nap: Voi n = 2 dé thj tuung tng G2 day di c6 by —1 = 5 dinh va 2 mau canh (xanh, dé) khong c6 dd thi con Ky cing mau. Khi dé Gp c6 thé biéu dién 6 dang hinh 5 canh véi canh cing m&u dé va dutng chéo mau xanh. That vay, do Gp dy di nén méi dinh xuét phat ding 4 canh duge to bing 2 mau. Chinh xéc hon, tai timg dinh, méi miu duge t6 trén diing 2 canh. Gia sit ngudc lai, tai dink A mau d6 duce t0 trén 3 canh 1A AB, AC va AD. Khi d6 mot trong 3 canh BC, BD,CD mau dé, dd thi c6 tam gide 43. Nguoc lai cf 3 canh déu mau xanh, dé thi 6 tam gidc xanh, Nhu vay mu thudn v6i gid thiét, Gid sit tai A 06 cfc canh dé la AB va AC (dung lién nét), cn AD vA AB la xanh, (dudng nét ditt). Khi 46 canh BC phai xanh vi ED la dé (hinh 1) Hinh 1 Hinh 2 Hai canh BE va CE khong thé cing xanh. Gid sit BE dd, thi CE xanh, Suy ra CD dé va BD xanh. Vay ta duge hinh 5 canh v6i canh mau dé va dudng chéo xanh (binh 2). 2. Quy nap: Gid sit khing dinh ding voi n = k. Xét d6 thi Guy, dy di v6i Beegayaa 1 dinh, k+1 mau canh va khong c6 dé thi con Ky cing mau. Méi dinh ciia Gey1 xudt phdt (bi 41 — 1).(k +1) canh voi k +1 mau, nén phai c6 ft ahét bi41— 1 canh cing mau. Gid sit tai dinh A c6 by41 —1 canh cling duge t8 boi mau ™y. Khi dé trong céc dinh d6i cia A khong c6 c&p dink nao duge n6i v6i nhau bai canh mau m, (tréi lai thi c6 Kg cing mau m,). Xét dé thi con day di Gy lap nén tit bey, — 1 dinh déi cia, A c6 canh chi t6 bai & mau (trit mu m,) va khong cé Ka cing mau, nén theo gid thiét quy nap, trong Gy ¢6 hinh 5 canh véi canh cing mot mau va dwéng chéo 8 cde mau khée (tAt c& déu khong 18 mau my). Vay trong Gs 06 didu clin khing dinh. 3. Cid tosn 1°) Xay dung dé thj mo ta quan he Cée dé thi tuong ting véi 3 bai toan da cho duge xay dung nhu sau: a) Dinh: Lay 5 diém tren mat phing, khong c6 3 diém nao thang hang tuong tng v6i 5 thanh vién (5 86 nguyén duong, 5 d6i tuong da chon ra). Ding ngay tén céc thanh vin (cdc 6, tén céc déi tong) dé ghi tren céc diém tung img. b) Canh: Dang - Canh dé dé n6i gitta hai dinh tuong ting v6i hai ngudi quen nhaw (hai s6 6 uée chung, hai déi tuong c6 quan hé ty). ~ Canh xanh dé néi gitta hai dink tuong ting véi hai ngudi khong quen nhaw (hai s6 nguyén t6 cling nhau, hai déi tugng c6 quan hé ty) Dé thi G; (1 <4 < 3) mo tA ton bo quan hé diéu kign dugc cho trong bai toan i, nén trong G; khong ¢6 tam giée cing mau, 2) Suy ra dép én. Theo khang dinh 2 v6i n = 2 dé thi G; Ia da giée 5 canh véi ce canh mau dé va céc dudng chéo mau xanh ho&e nguge Iai. Khi dé dya theo dung gip khic Khép kin mau dé ma sp xép cée thinh vien (céc s6; cdc d6i tutgng) tuong tg ngudi xung quanh mt ban trén (lén mot duéng tron), thi méi thanh vien (mdi s6, méi d6i tuong) sé ngéi gitla hai ngudi ma thinh vién cé quen (dting gia hai s6 ma n6 06 wéc chung; ding giita hai d6i tugng ma né e6 quan he 2). 3 Dang 3 3.1 Bai toan Bai toén 1 Trén méit phiing léy 6 diém thy y, khong c6 3 diém ndo thdng hang va khodng céich gita cdc cip diém khéc nhau tig doi mot. Ching minh ring tén tai tt nhdt mot cp diém ma doan théng néi gitta chiing 1 canh ngdn nhat ciia mot tam gidc nado dé, déng thai la canh dai nhét ciia mét tam gide khéc 6 dinh la cde diém da cho. Bai toan 2 Ching minh ring trong n (n> 6) ngudi ty ¥ ludn chon duige n — 4 66 ba, ma trong méi B69 ba nay hot ting doi mot quen nhau hoiic titng doi mot khong quen mhau. Bai ton 3 Ching minh ring trong n (n > 6) 86 nguyen duong tiy ¥ luén luén chon duge n—4 b6 ba, ma trong méi b6 ba nay ting cap s6 cd ude chung hotic nguyén té cing nhau. Bai toan 4 Chiing minh riing trong n (n > 6) adi twong tiy y lon chon ra duge n— 4 86 ba, ma trong méi bp ba ndy hode titng cp cd quan hé ty hodc tig cap c6 quan hé tz Bai toan 5 Voi n = 5 thi cdc khdng dink phat biéu trong céc bai todn 2, 4 con dring nita khong? Néu cdc khéng dink trén khong ding hay cho phan vi du. 3.2 Khang dinh Khang dinh 3 Dé thi day di gdm n dink (n > 6) va duge t6 bing khong qué 2 mau canh, thi ludn c6 tt nhat n — 4 tam gide cing mau. Chutng minh. Trung hop 1: DB thi diy di G, 06 n dinh (n > 6) duge to bing mot mau canh. Khing dinh dé dang duoc ching minh. Trutng hop 2: Da thi day dii G, c6 n dinh (n > 6) véi 2 mau canh (ching han xanh, dd). Ta phai khing dinh Gy 06 it nhét n - 4 tam giée cing mau. Diéu nay duge ching minh biing quy nap theo s6 dinh cita d8 thi. 1. Ca sé quy nap: n = 6 thi dd thi tuong img G5 day dit v6i 2 mau canh (xanh, 48). ‘Ta phai ching minh Ge o6 it nhét (6 — 4) = 2 tam giée cing mau. Theo dinh lf 2.19, Ge ludn 6 ft nh&t mot tam giée cing mau. Do dé ta phai ching minh trong Gs c6 thém ft nhat mot tam giéc cing mau nifa. Khong mt tinh téng quét, ta goi céc dinh ciia Ge 1 A, B,C, D, E, F va tam giée cing mau la tam giée ABC véi céc canh mau dé (nét lién), (hinh 3). Hinh 3 Ta xét cée trutng hop sau c6 thé xdy ra: 1°) Ci ba canh AD, AE, AF déu duge to mau dé (hinh 4), Khi d6, néu c6 ft nh&t mot trong ba canh DE, EF, DF dé thi trong Gs o6 thém st nhét mot tam gide cing mau nia (tam giéc dé) Nguge Iai, néu c& ba canh DE, EF, DF xanh (nét ditt) thi trong Ge c6 tam giée cing mau thit hai (tam giée xanh). 2%) CA ba canh AD, AB, AF du dige t6 mau xanh (hinh 5) Ching minh tuang ty trudng hop 1°). 10 Minh 5 Hinh 6 Néu c6 ft nhdt mot trong ba canh DE, EF, DF xanh thi trong Ge c6 them ft nhit mot tam giéc nia cing mau (tam giée xanh). Neuige lai, c& ba canh DE, EF, DF dé thi trong Go c6 tam giéc cing mau thi hai (tam giée 43 3°) Trong ba canh AD, AE, AF c6 hai canh dd, ching hen AD, AE dugc to mau dé (hinb 6) Khi dé; néu 6 St nh&t mot trong ba canh CD, DB, CE dé thi trong Gs c6 them it nh&t mt tam giéc nifa cing mau (tam gide dd) Neue lei, cf ba canh CD, DE, CE xanh thi trong Ge 06 tam gide cling mau thit hai (tam giée xanh). 4°) Trong ba canh AD, AE, AF c6 ding mot canh dé, ching han AD duge t6 mau 6 (binh 7). Hinh 7 Hinh 8 a) Mot trong hai canh BD,CD dé. Khi dé trong Ge c6 tam giée ching mau tht hai (tam gide da). b) Ca hai canh BD, CD xanh (hinh 8). by) BF xanh: Khi dé trong Gy 06 tam gie cing mau thit hai (tam giée AEF xanh). ba) EF dé: bai) BE dé (hinh 9) bait) Ho&e BF hoje CE dé, ta cé tam giée cling mau thit hai (tam giéc 8). bei2) C& hai canh BF vi CE xanh (hinh 10) bei21) Hote DE hote DF xanh, ta c6 tam giée cling mau thtt hai (tam giée xanh) uw Hinh 9 bai22) Ca hai canh DE va DF déu dé, ta cé tam gidc cing mau thit hai (tam giée DEF 4) ba2) BE xanh (hinh 11) Hinh 11 Hinh 12 bz21) DE xanh, ta c6 tam gidc cling mau thit hai (tam giée BDE xanh). bo22) DE dé (hinh 12). be221) DF dd, khi dé ta c6 tam giée cing mau thit hai (tam giée DEF dé) bo222) DF xanh (hinh 13). bz2221) Hoke BF hoje CF xanh, ta cé tam gidc cng mau thtt hai (tam gide xanh). b22222) Cé BF va CF déu dé, ta c6 tam gic cing mau thtt hai (tam gidc BOF 8). ‘Vay trong moi trudng hop, Ge déu c6 it nbAt hai tam gide cing mau. 2. Quy nap: Gid sit dink ly ding voi n = k. Xét dd thi Guys dy dii voi (4+ 1) din, dugc t6 bing 2 mau xanh, dé. Ta phai ching minh Gx4. 06 it nat (k-+1—4) = (k-3) tam giée cling mau. That vay, vi Gis day dit gdm (k +1) dinh (k+1> 7) voi hai mau canh nén o6 ft nhét mot tam gide cing mau. Gia sit céc dink cha Guy 1A Ay, Aa,...,Aeyi va tam giée cing mau IA tam giée AjA2As (ching han mau dd) (hinh 14). Loai Ai va cé canh xudt phét tir Ay ra khdi dd thi Gey1, ta c6 dd thi Gy voi hai mau canh (xanh, 46). Nen theo gid thiét quy nap, trong Gy luon o6 it nhét (k— 4) tam side cing mau. 12 Hinh 13 Hinh 14 Khoi phue dinh Ai cling céc canh thude A, ta duge 48 thi Gii2 véi hai mau canh (xanh, 48) va Gey 06 St nbat (k-4+ 1) hay (k 3) canh cing mau. Khing dinh duge chiing minh. 3.3 Giai toan Giai bai todn 1. Xét t&t cd céc tam gide c6 dinh 18 cée diém da cho. Vi khoaing céch giiia cc cap diém da cho khée nhau ting doi mot, nén méi tam gidc c6 dinh IA cée diém a cho déu 6 canh ngfn nbat va canh dai nhét. Déi véi méi tam giéc nay ta ding mau xanh dé to canh ngfn nhét. Sau khi tat cA cic doan thang n6i giita cdc c&p diém da cho duge phép t6 mau xanh da to xong, phan doan thing cin lai to mau dé. Dé thi G nhan duge 18 dé thi day di gdm 6 dinh v6i céc canh duge tO bing 2 mau (xanh, dé), nén theo khang dinh 3 trong G cé it nh&t hai tam giéc cing mau. Vi tam gide mao efing c6 canh ngin nhAt duge to mau xanh truée, nén cée tam giée cing mau déu li tam giée xanh. Khi d6 canh dai nht trong méi tam gide nay 1A doan thing cin tim. Béi vi trong tam gidc ta xét n6é déng vai trd canh dai nh&t, nhung vi doan thing nay c6 mau xanh, nén n6 da 1A canh ngén nh&t cia mot tam gid ndo dé trong céc tam. giéc c6 dinh 1A céc diém da cho. Gidi bai toan 2-3. 1°) Xay dyng dé thi mé té quan hé. Céc dé thi tuong ting v6i céc bai ton 2-4 duge xéy dung nhu sau: a) Dinh: Lay n diém (n > 6) tuong tng véi n ngudi (n sé nguyén, n d6i tugng) da. chon ra. Diing ngay tén ngudi (cdc 86, ky hiéu cdc déi tugng) dé ghi trén cdc diém tung ing. b) Canh: Ding - Canh dé dé néi gitta hai diém tuong tng véi hai ngudi quen (hai s6 6 wéc chung, hai d6i tugng c6 quan hé t1). ~ Canh xanh dé n6i gia hai diém tuong (mg véi hai ngudi khong quen nhau (hai s6 nguyén t6 ching nhau, hai d6i tugng c6 quan hé ta) Dé thi G (2 1) doi mot nguyén t6 cing nhau. Héi trong céc sé thudc {1,2,...,n} (n > 1) 06 bao nhiéu sé khong chia hét cho bat crt sé nao thudc {a1,a2,..., Am! Gi Nhén xét ring s6 céc sé nguyén duong khong vugt qué n chia hét cho sé (1 1) khong chia hét cho b&t ott sé ndo thuge {a1,02,-.-,4m} tinh bling cong thiic ~ (+++) + (2a) G++ Ete) ee 4102" Ory. Nhan xét ring, khi céc tinh chat duge ligt ké nhut trong Dinh ly Silvester khong c6 tinh doc lap thi céng thttc tinh todn phai thay ddi. Ta xét vi du sau day. Bai toan 4 Héi trong tap A = {1,2,...,280} c6 bao nhiéu sé khong chia hét cho 2, 3, 4,5, 6,7. Giai, Nhan xét ring, s6 céc s6 trong A khong chia hét cho mt trong céc s6 2, 3, 4,5, 6,7 cing bing mét s6 céc sé trong A kh6ng chia hét cho mét trong céc s6 2, 3, 5, 7. Ta xét xem trong tap A c6 bao nhiéu s6 chia hét cho mot trong cdc 6 2, 3, 5, 7. Ki higu A, 18 56 céc 86 thudc A chia hét cho 2, Ay 18 86 cée s6 thudc A chia hét cho 3, Ag la 86 cdc s6 thude A chia hét cho 5 va Ay 1 86 céc s6 thud A chia hét cho 7. Khi d6 A, U ApU Ag U Ag Ia tp céc 86 chia hét cho mot trong céc s6 2, 3, 5, 7. Ta c6 = [=| = 280 _ 40, va 22 [Arn Add [2 0; 1420 al = [22] = 6 | [42 Ad = [>| = 15; [As Ad = [FI | va 280 1A. Agn As = ES = 9141.42 Ay] 280 ‘0 ain Asnad = [ ] = Vapy == F Vosby..wd Ching minh. Phuong phép ching minh hoin toan nhu d6i véi Dinh ly Sylvester khi ta gin méi déi tuong cing mét gié tri bang 1, 23 Cong thic tinh sé phan tw cia mét hgp cac tap hop Vi Dinh Hoa ‘Trong cudc s6ng nhiéu khi xudt hign nhiing bai toén phai tinh s6 lugng phan tit cha mot tép hop thong qua nhitng tap hgp con cita ching. Ching han: ‘Vi du 1 Trong mét bai kiém tra todn 06 hai bai todn. Trong ca lép c6 30 em lam duge bai thet nhét va 20 em lam duoc bai thit hai. Chi cé 10 em lam duac c& hai bai todn kiém tra. Hay tinh 86 hoc sinh trong lép. Goi Aa tap hop hoe sinh gidi duge bai ton thi nhét, va B la tap hap hco sinh gidi duge bai toén thit hai, thi ANB IA tap hop hoc sinh gidi dugc o& hai bai ton. Bai tofn dugo dat ra 1a phai tinh s6 phan ti cia AU B. Néu A va B la hai tap hop rdi nhau, thi thy dé dang ring: |ANBl=|Al +B]. ‘Trong trutng hop A va B c6 giao khéc réng thi ding thite trén khong cdn ding nita, ma ta s8 06 cong thie [AUB =|Al +|B]-|An Bl, bai vi trong téng |A| + |B] thi cée phin tit chung trong |AN BI (cia cd A va B) duge tinh lap déng hai lan. Sit dung cong thitc nay, ta thy s6 hoc sinh cita lép trong vi dy trén cba ta 1a 30 +20 — 10 = 40 em. Nhiéu khi, bai ton ta g&p trd nén phitc tap hon khi phi tinh s6 phan ti cha mot tap hgp c6 nhiéu hon hai tap hgp. Vi du 2 Lop 12A phdi lam mét bai kiém tra Todn gém cé ba bai todn. Biét ring méi em trong lép déu lam duoc it mhét mot bai, trong lép cé 20 em lam dusge bai todn thit nhét, 14 em gidi dude bai todn thit hai, 10 em gidi duge bai todn thit ba, 6 em gidi dugc cd hai bai todn thit nhét va thit ba, 5 em gidi dude cé hai bai thit hai va thit ba, 2 em gidi duoc cé hai bai thit nhdt va thi hai, vad cd mot em dude 10 diém vi da gidi duoc cd ba bai todn. Héi ring lép hoc 06 bao nhiéu em tat ci? Goi A la tap hop cée em hoc sinh gidi dugc bai toan thit nh4t, B 1A tap hgp cée em hoc sinh gidi duge bai todn thit hai va C la tap hyp cae em hoe sinh gidi duge bai ton 24 thit ba, ta phdi tinh s6 phn tit cia tap hyp AU BUC. Trong nhigu trugng hop khée, chiing ta phai tinh s6 phan tit cia mot hgp gém nhiéu tép hop con, va phan nhiéu cée bai todn nay 1a céc bai toan khé véi hoc sinh khong hé duge hoc cong thie tinh té hop. 1 Xé€p cé lap véi tan sé cho truéc Mot phn tit a; ctia mot day k phin tit cho truéc aia2...a, dutge n6i la c6 tin s6 lip ky, néu nhu n6 xudt hign trong day ding &; lin. Trong myc nay ching ta xét bai todn sau: Bai toan. Cho trudc mét tap hop A = {ay;a9;...; an} vd k la mot s6 duong nguyén cing wai cde 36 ty nhién ky, kay... hn théa man D7 ky = k. Hay tinh s6 P(k, ko,...,hn) cc day k phan ti ciia tap hop A sao cho phan tit a; cia A cé tan sé Idp la ki cho moii 1 cho moi gid trid 1 ta 06 (ety)? = Docherty! io Ching mink. Dé khai trién (x + y)", ta thuc hign phép khai trién lay thita mot céch hinh thc ma khéng rit gon ching. Ching han (ety) =crt sy tyct yy. Nhu vay ta 06 thé biéu dién hinh thie (e+uy (z+y)(z+y) (ey) = Dreier. cas 6 d6 c6 t&t cA 2" s6 hang cicp...cy voi c: € {2;y}. Trong khi thyc hién phép rat gon, ta phai dem nhém t&t cA céc s6 hang c6 cing s6 mi cia z va y lai voi nhau. Véi méi O 2 tap hop tay §. IALUA2U++-UAnl = SOIAd— SO A Aste ra Lign met (HPAL ARN An > An; Angi. Luu § ring Bay gid xét n+ 1 tap hgp tay y Ai, Aa. (A1UA,U “UAR) 9 Anas = (4s Ana) (™) i= Cho nén ta 06 |(AyU A2U---UAn) Angi! = Uf; (4s Ansi)]- Sit dung (*) cho vé phi cia (*), ta thu duge ALU---U An) A Ang] YAN veal — D IANA Anat Liga + SANA AN Ana 1 iGiek on mk (1) JALN Ag AR A Ansa Dit A= A,UAdU:--UA, va B = Anyi va 6p dung ding thie |AUB| = |A]+{B|—|AnB| (trudng hgp n = 2), ta thu duge diéu cn chting minh att IALUUAnU Ansa] = 1A SO A Ase ro 1s mot (HTP ALA Aa An Anal ‘Vay dinh ly duge’chting minh cho trutng hop n+ 1. Dinh ly duge chiing minh. Céch 2: Ta xét mot phan tit a € AyUA2U---U A, bit ky. Gid sit ring a thudc vio r (1 3 diém thi khi néi c&c diém voi nhau béi cdc doan thing cé dinh Ja diém trong n diém da cho thi ta thu duge mot da giée 18i hoe mot doan thing chita céc diém cdn lai 6 bén trong, va ta goi n6 (da gide 18i ho&c doan thing nay) 1a bao Iéi cia tap hgp n diém nay. R&t nhiéu bai todn cia If thuyét té hgp c6 thé gidi bing c&ch van dung bao Idi cha céc hinh hodc cia tap hgp diém cho truéc. Vi dy 1 Trén mat phang cho truéc mot 6 diém khong cing ndm trén mét dudng thang. Chiing minh ring ton tai ba diém sao cho dudng tron di qua ba diém nay khong chia diém nao cho trude d bén trong. ‘Ta xét bao léi cia tap hyp cac diém cho truéc nay. Do c6 ba trong s6 céc diém nay Khong thing hang, cho nén bao ldi cite chting IA mot da. giée di. Xét mot canh AB cha da giéc Ii nay. Trong nhiing dinh cdn Iai, gid sit C 1 dinh nhin canh AB véi mot goc Jon nh&t. Khi dé dudng tron ngogi tiép tam giée ABC khOng chita diém nao da cho bén trong. Ngo&i tam giéc Ia da giéc o6 3 dinh ludn 1a hinh 18i, mOt n-gide bat ld voi n > 4 c6 thé khong phai la da giéc Idi. Mot da gidc duge goi 18 da gide cd canh khong ty cit néu anh cdc canh cha n6 déi mét khong c&t nhau triy nhing canh lién tiép c6 thé c6 dau mit chung. Moi da gidc Idi cing la da gidc c6 canh khong tut c&t. Cho truéc mot da giéc cé canh khéng ty cét, ching ta luén c6 thé chia n6 thanh cfc hinh Idi, cy thé JA chia né thAnh cdc tam gidc, béi cdc dutng chéo khong c&t nhau trong da giéc. Dé ching minh didu nay, truée hét ta chimg minh két qua sau: inh If 1 Trong mot de gidc 6 canh khéng ty cit lun cé mét dudng chéo nim hoan toan trong né. ‘That vay, néu da gide da cho IA da gidc Idi thi khiing dinh cia bai todn 1a hién nbién. Neuge Iai, néu da gide da cho khong phai lA da giéc 16i thi n6 c6 mot dinh, goi la A, 30 6 gc 16n hon 180°. Tir dinh A nay ta ké mot tia Ax ndo dé, cit bién cia da gide tai mot diém M. Do A > 180°, cho nén c6 mt trong hai canh bén cia A, ching han AB, sao cho tia AB (di tir A tdi B) khong c&t canh cia da gidc c6 chita diém M. Khi ta cho tia Az quay quanh A theo chiéu tir vi tri ban dau t6i vj tri AB, tht diém M chay tren bién ciia da giée va lic nao dé né phi di chuyén tit mOt canh nay sang canh khéc cita da gidc d& cho. Tai thdi diém d6, tia Az phai di qua mot dinh C no d6 khée A ca da giéc. Lite dé dutng chéo AC chinh IA dung chéo nim hoan toAn trong da giéc. Dinh If sau day hién nhién 18 hé qua cba dinh If trén: Dinh li 2 Mét n-gide cé canh khéng ty cét ludn cé thé chia thank n — 2 tam gide béi cde dudng chéo khéng ct nhau ndm hoan toan trong da gidc. Ta ching minh khdng dinh bing quy nap theo s6 dinh cia da giée. Voi n = 3 khang dinh cia dinh If hién nhién ding. Gia sit két lun cla dinh If ding cho moi k-giée (k 3 vak-+é=n-+3. Theo gid thiét quy nap thi céc k-giée va é-gidc nay o6 thé chia thanh k — 2 va €~2 tam giée bai céc dudng chéo khong cét nhau va hon ton nim trong chiing. Bang céch d6, ta da chia (n+ 1)-giée da cho thanh k+€—4=n—1 tam giée bai céc dung chéo khong c&t nhau va hoan toan n&im trong (n+ 1)-giée da cho. Dinh If duge chitg minh cho trugng hgp (n+ 1)-gide. ‘Ta goi mot tam gidc c6 céc dinh 1a dinh luéi cia mang hi6i 6 vuong 1A tam giée don néu nhut n6 khong chifa m6t dinh hi6i ndo khéc trén canh hofe bén trong tam giéc. Dinh If 3 Céc dinh cia mot k-gide cb canh khong ty cit F (khong nhét thiét phdi 16i) nam é cae diém nguyén. Bén trong né cén diém nguyén, con trén bién m diém nguyén. Ta néi che diém nguyén lai vdi nhau bing cdc dogn thdng khdng cit nhau sao cho tét cd céc tam gide thu duge la tam gide nguyén, khi dé s6 tam gide don thu duce bing nhau va bing ding 2n +m —2. Goi s la 86 cdc tam gid don thu duge. Téng cdc géc cia s tam gide don nay 1a 1 x s. ‘Téng nay bang diing téng cc géc quanh céc diém 6 bén trong F’ va céc diém trén chu vi cia F (gdm céc dinh cia F va céc diém nguyén 4 trén bién cilia n6). Téng cdc goc quanh céc dinh cia F bing 7 x (k — 2) (do F c6 thé chia thanh & — 2 tam giée bai céc dudng chéo khong ty cét 4 hoan to&n bén trong né theo dinh If trén). Teng cdc g6c quanh méi diém nguyén 6 trén canh cia F 1a 7, va téng cdc géc quanh méi dinh trong F 1a 2r. Do dé ta c6 ding thite: wx (k—2) 40x (m—k) +2nxn=ax s, ‘Va ta thu duge dang thite 2n +m — Bai tap 31 1. Ching minh ring mot da gide IA da giée Idi khi va chi khi b6n dinh bit 1d cha ching tao thinh mot tit gide Idi. 2. Trén m&t phiing cho truéc nam diém sao cho khong cé 3 diém nao trong ching ~ thing hing. Ching minh ring c6 thé chon ra trong ching 4 diém la dinh cia mot tit gide Idi. 3, ‘Tren m&t phing cho truée n > 5 diém trong d6 khong c6 3 diém ndo thang hing. Ching minh ring tén tai it nhdt C?_, tit gide 18i ma céc dinh cia chting 18 4 diém trong s6 n diém da cho. (Ki thi Todn quéc té lan thi 11, n&m 1969) 4, Trén mit phng cho mot s6 hitu han diém. Ching minh rang luén tim duge mot diém sao cho gan né nhat c6é khong qué 3 da cho. 5. Tren mat ban dat n hinh vudng bing cét-tong van hinh vudng bing nhya sao cho céc hinh vudng cing loai biing cét-tong (cing nhu cfc hinh vudng cing bing nhya) doi mot khong c6 diém chung. Sau khi xép ching lén mat ban, ngudi ta nhan xét thay ring tap dinh hinh vudng bang nhua hon toan tring véi tap dinh hinh vudng bing cét-tong. Hay ching minh ring cdc hinh vuong bing cét-tong dutge dat tring hoan toin v6i céc hinh vudng bing nhva. 2 Mang lwéi dinh 6 vuong M6t hé théng vo han 6 vuéng tao trén mat phing giéng nhit ta lat gach mot cdi san bdi nhitng vien gach lét 0 vudng dutge goi A mang Iudi dinh 6 vudng. Céc 6 vuong nay duge g0i 18 cée 6 vudng co s8. Cac dinh 6 vudng chinh la céc diém nguyén (diém c6 cA tung do lan hoanh do 1a cdc s6 nguyén) cita mot hé truc toa dd song song v6i cdc canh cia céc hinh vudng co sé va c6 don vi géc 1a do dai canh hinh vudng co 86. Mot da gidc c6 dinh 1a céc dinh Iuéi cia mang 6 vudng dude goi la da gide nguyen. ‘Véi mang lu6i dinh 6 vudng cé nhiéu bai todn khé thi vi. Sau day 1a mot tinh chét co ban ciia mang lu6i dinh 6 vung. Dinh If 4 Da gide déu duy nhét cb dinh tai céc didn ludi 6 vudng [a hink vwdng Dé thdy rang ta cé thé dung dugc nhitg hinh vudng c6 dinh tai céc diém luéi 6 vudng, ching han hinh vu6ng co sd cia mang luéi 6 vudng 1a mot hinh vudng nhu vay. Bay gid ta chimg minh ring ngoai hinh vudng, ta khong thé dyng mot da giée déu nao khée ¢6 dinh tai céc diém ludi cia mang ludi 0 vudng nao ca. Ta chting minh bing phwong phdp phan ching. ‘Trude hét ta ching minh ring khong thé dung duge tam giée déu c6 dinh tai cdc diém ]u6i 0 vudng. That vay, gid stt dung duge tam giée déu canh a c6 dinh tai céc diém lu6i @ xis a mot 56 v6 ti, do a? 1 mot s6 nguyén. Mat khéc, din tich ota tam giée c6 ba inh I diém lui cba mang luéi 6 vudng 18 mot s6 hitu ti, v6 Ii. Vay, khong thé dumg duge tam gide déu c6 cha mang lu6i 0 vudng. Khi dé dign tich cia tam gide déu ny 18 32 dink tai céc diém ludi cia mang Iuéi 6 vuéng. Bay gid ta chting minh ring khéng thé dung duge nga gidc dau c6 dinh tai céc diém luéi cia hinh vudng, That vay, gid sit khiing dinh nay khong ding, va tén tai ngi gide déu c6 dinh tai diém luéi cia mang 6 vudng, dé gon ta goi dé 1& nga gide dau tét. Xét tap hop: A= {a?:a 18 canh cia nga gide ddu tét}. Do A la mét tap con khéc réng cia tép hop céc s6 nguyén, cho nén tdn tai mot phan tinhé nbét a cia A. Xét nim dinh cia ngii gide du canh a nay, ‘Ta, dem néi céc dung chéo cita ngit giéc lai. D8 théy ring méi dutng chéo cia mot ngii giéc déu song song véi canh déi dign ciia n6. Do d6 hai dudng chéo thy ¢ cia nga sie cling v6i hai canh d6i dién cia chiing lap thinh mét hinh binh hanh, cho nén giao diém cita hai dudng chéo cing 1A mot diém cia mang luGi 6 vudng. Dé chting minh diéu nay ta lap mot hé toa d6 6 truc song song véi canh ciia 6 vudng co sé vi c6 do dai bing 6 dai ciia canh hinh vudng co sd. Khi d6 céc dinh cita mang It6i 6 vudng cd 8 chinh la céc diém 6 toa dé nguyén. Ban doc dé dang chting minh ring giao diém nay c6 toa do bing téng hai toa dé cia hai dinh cuéi cia hai canh di dign trit di toa d6 cia dinh chung cia hai canh nay. Nhu vay nim giao diém cita cée dutng chéo lap nén mot hinh ngii giée déu cé nim dinh 1a nam diém nguyén. Hinh 1: Ngii gide déu DE théy ring ngi gide déu nay cé canh nhd hon a, mau thudn v6i gid thiét ring trong céc ngii gidc déu c6 nam dinh 1a cdc diém nguyén, nga giée déu cé canh nhé nhét Tia. Mau thudn nay chting té ring khong tén tai ngii gidc déu nao c6 cd nim dinh 1 nm diém nguyén. Mat khéc, ta c6 thé théy ring khong thé dung duge Iye giéc déu c6 dinh tai mang luéi dinh mang Iuéi 6 vudng, vi néu cé mot luc giée déu nhu vay, thi cing 6 mot tam gide du (duge tao bdi ba dinh doi mot khong ké nhau cia lye giée déu nay) c6 dink lA dinh luéi ciia mang 6 vudng da cho. Bay gid ta ching minh ring véi n > 7 thi khong thé dung dugc n-giée déu cé dinh tai dinh ludi mang 6 vudng. That vay, gid sit dumg duge mot n-gide déu AyAy--+ A, 06 dinh tai céc dinh cia mang lu6i 6 vudng. Khi dé ban kinh R cia dudng tron ngoai tiép n-gide J6n hon can a ciba n-giée déu A)Ay--- Ay. Ta chon mOt dink O cia mang lu6i 0 vudng va chon céc dinh By, B2--- By sao cho OB;=AiAis1 (Aner = Aj). Vi cde dinh A; va O 1 dinh lugi 6 vudng, cho nén céc diém B, cling 1a dinh IuGi 6 vudng, Ngo&i ra, ta thay 33. dé dang lA B,Ba--+ B, 18 mot n-gidc déu, déng dang véi n-giéc déu A; Ag-+- A, theo hé s6 £ <1. Cit tién hanh nhu vay, sau mot s6 k buée hitu han dit Ién, ta thu duge mot negidc déu c6 canh nhé hon canh cia 6 vudng co sd 1a diéu vo li. Do d6 khéng thé ding duge n-giée déu (v6i n > 7) c6 dinh tai dinh lui cla mang 6 vudng. Bai tap 1. Tim bén kinh 16n nhét cia dutng tron chi di qua céc dinh cia mang luéi 6 vudng ma khong ct canh 6 vudng nao é diém trong cila nd. 2. Cho truéc mét s6 dinh cia mang luéi 6 vudng, ching minh ring ta c6 thé to mau céc dinh ndy béi hai mau xanh va dé sao cho trén méi duéng nim ngang va trén méi duttng thing diing s6 diém duge to dé va s6 diém duge td xanh xAp 24 nhau. (D8 thi Tosn quéc té nim 1986) 3. Chiimg minh rang néu di doc theo céc canh ciia 6 vudng co sé tit mot dinh bat ki i ta tré vé dinh ban dau sau hitu han bude (c6 do dai bing canh hinh 6 vung co 8d), thi sé bude di cilia ta sé la mot s6 chin. vao mdi 6 vudng cd sé mét sé nguyén sao cho méi sé nay bang ding trung binh cong cia bén s6 8 bén 6 vung cd s8 c6 canh ké véi n6. Hay chting minh rin, a) Tit 8 céc s6 duge dién bang nhau, néu nhu né bi chan. b) C6 mot cach dign sao cho cde s6 duge dién khong nhdt thiét phai bing nhau tht cB. 5, Trén ban cd vo han ta thyc hign mOt trd choi nhu sau: ‘Dau tién xép n? quan vao mot hinh vudng gm n x n 6 vudng canh lién canh sao cho trong méi 6 vudng ciia hinh vung chita mgt quan c’. Céch di trong trd chai 1a quan cd chi dutge nhdy theo mot chidu ngang hoc chiéu doc qua mot 0 c6 chtta quan cd d ngay sét bén canh sang mot bén mot O tréng tiép ngay sau a6. Khi d6 quan c¥ 6 6 bj nhdy qua sé bi loai b3. Tim céc gid tri cla n dé c6 thé két thic trd chai sao cho trén ban cd chi cdn lai ding mot quan cd. (Ky thi Toén quéc té néim 1993) 3 Dinh li Picard Mot img dung khé thiét thye cia mang ludi 6 vuong Ia img dung vao viée tinh dign tich ofa céc hinh phang. Ta 6 thé thy hign vigc tinh dign tich cia mot hinh phing bing céch dem phi né béi mot ludi 6 vubng di nhé ri tinh dign tich hinh da cho. Co 86 cho vige tinh dign tich da giée nguyén (da gide o6 dinh 1A céc diém toa d@ nguyén) 18 vige xéc dinh dign tich cia cfc hink tam gidc cé dinh la cfc dinh luéi (con goi la dink nguyén) ma khong chia dinh nguyén ndo khéc bén trong hode trén canh cia né (tam giée don), 34 Dinh Ii 5 Dien tich ciia tam gide don trén mang ludi 6 vudng don vi diing bang }. ‘Truéc tién ta théy dign tich cla mot tam giée nguyén khong nhd hon } Vita cé thé ngi tiép n6 trong mot hinh chit nhat béi céc canh di qua dink né va song song véi céc dudng thing cia mang luéi 6 vuong. Khi d6 dién tich cite tam gide nguyén bing téng hokc higu cia dign tich cia céc hinh chit nhét con va cée tam giée vudng ¢6 canh la 56 nguyén duge tao thanh, nén c6 dang £, véi k la mot s6 nguyén duong, tic IA it nhét cing phai a 3. Mat khdc gid sit hinh chit nhat phi tam giée don ABC cho truéc theo céch trén c6 dé dai hai canh 18 m van sé c6 dign tich bling m xn va chita (m—1)(n—1) diém nguyen bén trong va 2(n +m) diém nguyén trén bién. Ta chia hinh chif nhat nay ra thnh cdc tam gic don béi cic doan thing néi cdc diém nguyén, bit dau 1A eée canh cha tam gidc ABC, va thu duge nu dinh If 3 ca myc truéc cho ta 1 2(m —1)(n~1) +2(m-+n) —2 = 2mn tam giée don, Do dign tich ciia méi tam giée don (ciing I tam giée nguyén) khong nhé hon }, cho nén dign tich ola hinh chif nhat phi n6é khong nhé hon mn. Do é day xy ra ding thiic, nén dign t{ch cia mdi tam giée don thinh phan, dac biét IA dién tich cia tam giée don ABC, cing bing ding } Dinh ls Picard sau day cho ta céch tinh dign tich cia eée da giée nguyén 6 canh Khong ty c Dinh li 6 Cac dink cia mot da gide F ob cank khong ty cit (khong nhdt thiét phi 101) ndm 4 céc diém nguyén. Bén trong n6 cé n diém nguyen, con trén bién m diém nguyén. Khi dé dign tich ctia né bing Span4% ‘Theo dinh If 3 da chting minh 6 muc trudc, ta c6 thé chia F thinh 2n +m —2 tam gide don béi céc doan thing n6i céc diém nguyén nim trong hote trén bien ciia n6. ‘Theo dinh If da chimg minh 6 trén, dign tich otia méi tam gide don chinh bing }, nén dign tich cia F ding bing n+ Sr 2 -1 Bai tap 1. Céc dinh cia mot tam giée nguyén ABC khong chita mot diém nguyén nao trén canh ngoai céc dinh cia n6 va chita ding mot diém nguyén 6 trong tam gidc. Ching minh ring diém nguyén 6 trong tam giéc nay IA trong tam cia tam ide, 2. Tren mat. phang ly nim diém nguyén phan biét. Chimg minh ring tdn tai trong ching hai diém nguyén sao cho doan thing néi hai diém nguyén ndy di qua mot diém nguyén nao d6. 3. Chiing minh ring trong mét hinh ngit gic 1éi nguyén luon cé it mht mot diém nguyén 1a diém trong, 35 4, Hay tinh dign tich nhé nhét ma mot ngii gidc 1éi c6 dinh 1a céc diém nguyén cé thé 06. 5. Trén mét t8 gidy cé ké mang ludi 6 vudng c6 n 6 bj tO den. Chiig minh ring cé thé cit tir to gify nay ra mét s6 hitu han céc hinh vuéng théa mn déng thdi hai digu kign: a) Tat c& céc 6 den déu nim trong cdc hinh vuéng nay, b) Tile dign tich bi to den va dign tfch toin phan trong méi hinh vudng nay & 4 gitta : waz Nguyén ly Dirichlet vA mét sé bai toan 4p dung Nguyén Duy Thai Son Nguyén ly Dirichlet (thuat ngit tiéng Anh: the pigeonhole principle, ciing ¢6 noi goi 23 the drawer principle) - 6 dang don giin nhit - duoc phét biéu dau tién béi G.Lejeune Dirichlet (1805-1859), mot nha ton hoc Ditc gb¢ Php, nhu sau: "Néu nh6t n+ 1 con thé vao n cAi chudng (n € N*) thi ludn 6 (ft nhét 18) hai con thé bi nhét trong cing mot chudng" Mot céch téng quét, ta cé nguyén lf Dirichlet mé rong: "Néu nhét m con thé vao n céi chudng (m,n € N*) thi ludn tn tai mot chudng chita ‘m1 fe nhét 1a1+ [=] con tho". n 6 day, ky higu [a] dutge ding d@ chi phin nguyén ciia s6 thuc a tite 1 s6 nguyén 1én nhét khOng vust qué a. Dang phung phép phan chting, ta cé thé dua ra m6t céch ching minh khé ngéin gon cho nguyen ly Dirichlet (ngay c& duéi dang mé rong); hoc sinh THPT cing c6 thé lam duge vige nay; va didu d6 khong hé lam gidm di gié tri cia ban than nguyen ly. Nguyen ly Dirichlet c6 rét nhiu img dung (higu qué dén bat ngd): sit dung n6, ta c6 thé ching minh duge nhiéu két qua sdu sxe ciia toan hoe. Chinh vi vay, tai céc cude thi hoc sinh gidi toan (quéc gia va quéc 8), nguyén ly Dirichlet thutng xuyén duge khai théc. Dé minh hoa, duéi day, ta xét mot s6 bai toén cu thé. Bai 1 (Putnam 1993). Cho mot day s6 gdm 19 s6 nguyén duong khong vuot qué 93 ua mét day s6 gdm 93 56 nguyén dutong khong vugt qué 19. Ching minh ring tw hai day sb 6 ta 06 thé lan lugt trich ra hai day con cé téng cic s6 hang la bing nhau. Gi . Ta xét bai todn téng quét: Cho hai day (hit han) cée s6 nguyén dung: MS S-StmSn, miyws-Smsm Khi d6, tan tai cfc "chi s6"1 7 x 256 nén theo nguyén ly Dirichlet o6 tam hoc sinh (khéc nhau) Aj, Aa).--,Ag, mA bai Lim ciia ho thi tmg v6i cée bd thude cing’mét tap con Ando 6 (trong s6 256 tap con n6i trén). Nhung 2000 — 8 = 1992 > 7 x 256, nén lai theo nguyén If Dirichlet - tdn tai tém hge sinh (khée nhau, trong s6 1992 hoc sinh cdn lai), 18 Bh, By,..., Bs, 06 bai lam ting voi cée bo thudc cling mot tap con B ndo dé (trong s6 256 tap con n6i trén). Cudi cing, vin c6 1992 — 8 = 1984 > 7 x 256 nén ding nguyén If Dirichlet mot lin nia, ti 1984 hoc sinh cdn lai, ta tiép tuc tim ra tam hoc sinh (khéc nhau), 1& C,,C2,...,Cs, ma bai lam cia ho img véi céc b6 trong cing mot tap con C ndo d6. Tit bén hoe sinh bat ky trong s6 24 hoc sinh Ai, Aa, +) As; Br, Bay. Bay Cry Cay. +s Cay ta Iudn chon ra dude hai hoc sinh 6 bai lam ting véi cdc bé thudc cing mot top con (mét trong ba tap con A,B ho&c C); gid sit, ching han, 46 1a c&c hoc sinh Aj v& A; (i,j €Z; 1 $i 8 x 3 theo nguyén If Dirichlet, cit 25 hoc sinh thi tim duge bén hoe sinh (trong s6 25 hoc sinh nay) c6 bai lam ting véi bén bO z,y,z,t € D C S doi mot khée nhau; vA theo (3), hai hoc sinh ndo trong s6 bén hoc sinh dé cfing c6 bai lm khéc nhau 6 ft nhét 18 hai cau héi, ding nhu yeu cdu cla bai toan. Tu 1/ va 2/, ta thay: 86 tut nhién bé nhét phai tim 1A n = 25. 41 Nguyén ly Dirichlet va m6t sé bai toan Aap dung Nguyén Duy Thai Son Nguyén lf Dirichlet (thuat ngit tiéng Anb: the pigeonhole principle, cling c6 noi goi 1a the drawer principle) - 6 dang don gin nhAt - duge phét biéu dau tien bai G.Lejeune Dirichlet (1805-1859), mot nha ton hoc Die g6c Phép, nhu sau: "Néu nhét n+ 1 con thé vao n cAi chudng (n € N*) thi ludn o6 (ft nhét 14) hai con thé bi nhét trong cing mot chudng". Mot c&ch téng quét, ta c6 nguyén ly Dirichlet mé rong: "Néu nhét m con thd vao n céi chuéng (m,n € N*) thi luén tén tai mot chudng chita ft nbd a1 + [ ‘m—1 con thé", 6 day, ky higu [a] duoc ding dé chi phan nguyén cia s6 thuc a tite 18 s6 nguyén lon nh&t khong vugt qué a. Ding phung phép phan chting, ta c6 thé dita ra mot céch ching minh khé ngéin gon cho nguyén ly Dirichlet (ngay c& dudi dang mé rong); hoc sinh THPT cing c6 thé lam duge vige ndy; va diéu dé khng hé lam gidm di gié tri cia ban than nguyén ly. Nguyén ly Dirichlet ¢6 rét nhiéu img dung (higu qua dén bit ngd): si dung n6, ta c6 thé ching minh duge nhiéu két qua séu sic cia toin hoc. Chinh vi vay, tai cdc cude thi hoc sinh gidi ton (quéc gia va quéc té), nguyén ly Dirichlet thutng xuyén dude khai théc. Dé minh hoa, duéi day, ta xét mét s6 bai toén cu thé. Bai 1 (Putnam 1993). Cho mét day s6 gdm 19 86 nguyén duong khéng vust qué 93 vd mét diy s6 gm 98 86 nguyén duong khong vugt qué 19. Ching minh ring tit hai day sé dé ta cé thé lan lugt trich ra hai day con cé téng cdc sé hang la bing nhaw. Giai. Ta xét bai toan téng quat: Cho hai day (hitu han) cdc s6 nguyén dung: BM StQS--StmSn, WS ys Sym Sm. Khi dé, tén tai céc "chi s6" 1 < iy < ip Sm, 1S i < jo 1 va) MS bytw—1 + Uy) — Op = OS M— Use S byipy-1 — ap Fy < bya, méu thudn v6i dinh nghia cia f(p). Mau thudn dé chiing té ring: qua thuc, mdi higu (1) 18 bé hon m, Bay gid, néu mot trong cdc higu do trigt tiéu: byip) — ap = 0, thi r6 rang ta c6 ngay dpom véi céch chon i := 1 =: fi, ia = p, jn := f(p). Trong trutng hop cdn lai, theo nhan xét trén, toan b6 m higu 6 (1) déu thude ZN [1,m — 1}, mot tap hgp chi 6 m —1 phan tit, nén theo nguyén ly Dirichlet, c6 hai higu bling nhau; tite 1A, tén tai r,8€ZN{l,m], r>s, dé Byer) — ar = by(s) = Oy => byte — bya) = Oy — O53 Va ta cling 06 dpem, v6i i = 8 +1, i= 1, f= f(s) +1, j= f(r). Bai 2 (Vo dich Cong hoa Czech 1998). Cho X la mét tap hop gém 14 s6 nguyén dwong phan bigt. Chting minh ring cé mot 56 nguyén duong k <7 va c6 hai tap con k-phin tt: {ar} a25...; a4}, {b159;..-3 be} ri nhau ctia X sao cho (aade _ a 1) a ay” ap al Gm +S iar Gidi. Xét Cf, = 3432 tap con 7-phan tit cia X. Téng (céc) nghich do cita céc phan tit 11 1 trong méi tép con nay 16 rang I8 khong vugt qué ++ 5++---+ 7 < 2,6 nén phai thude vio mot trong sé 2600 nita khong: 0 1 1 2 (saan' sana): (eon! soa 38 Theo nguyén lf Dirichlet, tdn tai hai tap con khée nhau c6 tng nghich do cée phin tir thude vao cling mot nita kho&ng. Logi bé khéi hai tap con dé oéc phan tit chung (hai tap con 7-phan tit khéc nhat thi c6 t6i da séu phan tit chung), ta sé thu duge hai tap . con k-phin tit (véi k nguyén duong, k < 7), thod yéu edu cita bai toan: higu cia hai téng nghich dio céc phan tit trong hai tap con nay sé sai khéc nhau ft hon 1/1000, Bai 3 (Iberoamerica 1998). Cac dai dign ctia n quéc gia (n € N*\{1}) ngdi quanh mét ban trén theo cich sao cho hai ngudi ngdi sdt bén phdi hai dai dién bat kj véi cing quédc tich thi phdi c6 quéc tich khdc nhau. Hay xdc dink (theo n) sé lin nhAt cé thé 06 cdc dai dign ngéi quanh ban tron dé. Gidi. Ta sé ding X,, X2,...,Xq dé ky higu n quéc gia c6 dei dign ngdi tai ban tron; céc dai dién 06 quéc tich X, (N* 3 i 2, j < 1999. ‘Tw céch xay dung trén, dé dang ching mink A(m; j) < A(n; J), A(m;j)/A(n; 3) vi moi bé ba s6 nguyén ducng m,n, j ma j < 1999 vam < n. Tir céch xay dung trén, cing d& thay (vdi mdi i € Nt AGi1), AG 2)... A(G 1999) l& 1999 s6 nguyen duong lién tiép (khong bé hon a1) do 6, theo gid thiét ciia bai tosn vé tap hyp A, thi tin tai j, € ZM[1; 1999] d8 A(i;j,) € A. BAy gid, Vi ji, ja... 52000 € ZN(L: 1999], nén theo nguyén ly Dirichlet, c6 hai s6 nguyen dung m 7 x 256 nén theo nguyén If Dirichlet ¢6 tam hoc sinh (khéc nhau) Aj, Aa,--+Ag, ma bai lim ciia ho thi tmg véi cée bd thudc ching:mét tap con A ndo 6 (trong s6 256 tap con néi trén). Nhung 2000 ~ 8 = 1992 > 7 x 256, nén lai theo nguyén lf Dirichlet - tdn tai tam hoc sinh (khéc hau, trong s6 1992 hoc sinh cdn lei), 1a Bi, Ba, ..., Bs, 06 bai lim ting véi céc bd thude cing mot tap con B nado dé (trong s6 256 tap con néi trén). Cudi cing, vin c6 1992 ~ 8 = 1984 > 7 x 256 nén ding nguyen ly Dirichlet mot lén nifa, tit 1984 hoe sinh cdn lai, ta tiép tuc tim ra tam hoc sinh (khde nhau), I C,C2,...,C, ma bai lam cia ho img véi cée bo trong cing mot tap con C ndo d6. Tit bén hoc sinh bat ky trong s6 24 hoc sinh Aj, Aa, +, As; Bi, Ba, ..-, Bs, C1, Cay... Ca, ta Iuén chon ra dutge hai hoc sinh 6 bai lam tig véi céc bd thudc cing mot tap con (mét trong ba tap con A,B ho§c C); gid sit, ching han, dé JA céc hoc sinh A; vA Ay (i,j € Z; 1 8 x 3 theo nguyén ly Dirichlet, ett 25 hoc sinh thi tim duge bén hoc sinh (trong sé 25 hoc sinh nay) cé bai lam tig véi bén bd Z,y,z,t € DC S doi mot khée nhau; va theo (3), hai hoc sinh ndo trong s6 bén hoc sinh dé cing c6 bai Tam khéc nhau 6 ft nhdt 18 hai cu héi, ding nhu yeu cdu cia bai tosn. ‘Tit 1/ va 2/, ta théy: s6 ty nhién bé nhAt phai tim 1k n = 25. 41 Bai 6 (Dé nghs, Todn 11, kj thi Olympic 30/4 nam 2006 chiing t6i dé dua theo ¥ cia mét dé thi Vo dich Romania 1997). Goi A la tap hop tt cd céc 66 bax = (a1, 22,3) ma 22,3 € (0, 7]NZ. BO x = (21;29\23) € A duge goi ld troi han 66 y = (is yaiys) € A, néu x Ay vd 2; > y; vdi moi i € {1;2;3}; khi d6, ta viet x > y. Tim sé ty nhien n bé nhit sao cho moi tap con n-phén tit cia A déu chia ét nhét la hai b6 jy maar y, Giai, 1/ Truéc hét, xét tap hgp B= {x € Alzy + 22+ 23 = 11}. Cé thé kiém tra truc tiép ring B 1A mdt tap con 48-phan tit cia A (B gdm ding: 4 phan ttt cé dang 2 = (0; 29; 11—22) void < 22 <7; 5 phan tit cé dang 2 = (1} 22; 10-29) voi3 < 22 <7; 6 phn tik c6 dang x = (2; 22; 9-2) vi 2 < x <7; 7 phan tit c6 dang x = (3, 22,8—z2) v6i 1 < 22 < 7; 8 phan tit c6 dang (4,%2,7 ~ a2) véi 0 < 22 < 7; 7 phan tit 6 dang 2) V6i 0 S 2 < 6; 6 phan tit c6 dang x = (6;2;4 — 2) voi 0 y. 2/ Tiép theo, cho N 3 n > 49, va B la mot tap con n-phan tit bat ky clia A. Ta sé ching minh ring B chtta it nhAt hai bo z,y max > y. Muén vay, xét céc tap con sau day cita A := {ee Alny 2 € Al(ey Vite = T},Co i= {2 € Al(a) =1V 22 < 6) V (21 > 1 Az, =6)}, 2Va2S5)V (a, >2Am=5)}, Coss {2 © Al(z = 3022 <4) V (01 2322 =4)},C Ae. UC = {a € Alm > 4V 22 <3} {2 € Clas = J} (i,j EZ 1 17 phan td; nhung chi c6 16 tap con Dp, (p,q € Zi4 << ps7; 0 y (dpem). Tit 1/ va 2/, ta thay: s6 ty nhién bé nhét can tim IA n = 49. 42 Mot sé phuwong phap giai cdc bai toan té hop nang cao Dang Hing Thing ‘Trong khoa hoc cing nhu trong di séng ching ta thutng gap bai ton xdc dinh s6 luong cée d6i tuong cé mot tinh cht ndo d6. Ta goi dé Ia bai toén dém.Té hop 1 mot nganh ton hoc nghién cttu céc bai ton mang cfu tric rdi rec trong dé cé bai toan dém. Kj nang va kién thitc ciia toén té hop IA rat can thiét cho nhiéu khoa hoc tit kinh té t6i sinh vat, tin hoc , h6a hoc va quan trj kinh doanh. Nh@ céc phuong phép ma té hgp cung cp, ching ta c6 thé xdc dinh duge s6 long céc phan tit cia mot tap hgp mét céch nhanh chéng va chinh xée ma khong cdn ( vi nhiéu khi cling khong thé) liét ké duge vi 86 d6 rt lén. Chuuong trinh phd thong (Iép 11) da trang bi cho hoc sinh hai quy t&c d&m ea ban(quy tée cOng va quy tie nhén) , céc khéi niém hoén vj , chinh hop, td hop. Nhd dé hoc sinh c6 thé gidi dude cée bai toén té hop co ban, tutong d6i don gidén. Tuy nhién d6i vdi nhiing bai todn té hgp phic tap, cn cé nhiing céc phyiong php "cao cip" hon,skc bén hon, Bai gidng nay nhim cung c&p mot 6 phuong phép dé 06 thé cong phé duge cde dang ton khé trong té hap. 1 Quy t&c cng téng quat Ban chét todn hoc cit quy t&e cOng( phat biéu cho cong viée voi nhiéu phitong én) 1& cong thiic tinh s6 phan tir cia hgp n tap hgp hitu han doi mot khong giao nhau. Cy thé ta 06 Cho n tap hop Ar,...,An ddi mbt khong giao nhau. Khi dé JA1UA2U...An| = > 14s Trong mhiéu bai toan té hyp, ching ta phai tinh s6 phan tit cia hop n tap hop bat ky ( khong nhét thiét rdi nhau). Khi d6 ta c6 quy tic cong cho s6 phin tit cia hop clan tap hop bat kj , thutng duge goi IA cong thtéc bao hm va loai tri. Dinh ly (Cong thtte bao ham va loai trit) 43 Cho n > 2 tap hgp hitu han Aj,..., A, . Khi d6 ta cd [Ay U Ag U..Anl = =SlaI- Slain Ad+ SO [Ain ayn al idan 1 iGjek on feet SY ya, 944. Agl 1 ackencty n ek (1A Ag... An Dinh ly nay c6 thé chimg minh tuong déi d& bing phuong phép quy nap , xin dinh cho ban doc. ‘Vi du 1 Trong tap S = {1,2,..., 280} c6 bao nhiéu s6 khéng chia hét cho 2, 3, 5,72 Cidi ‘Ta dém xem trong tép S c6 bao nhiéu s6 chia hét cho ft nh&t mot trong cée s6 2, 3, 5,7 Kf higu Ay = {k € 3: k chia hét cho 2}, Az = {k € S: k chia bét cho 3} Ay = {ke S :k chia hét cho 5}, Ag = {k € 5: k chia hét cho 7}. Khi d6 Ay U Ag U AgU Ag IB tap cée s6 chia hét cho it nh&t mot trong cdc s6 2, 3, 5,7 Ta c6 2 2 [Ail = 282 = 140; a] = (282) = 99; 4s] = 222 = 56; 144] = 28° = 40 2 3 5 7 280 80 [Ai dal = FP = 465/44 As = 2 = 28;|A,N Al = [2M Aal = PRP = 385140 dl = (222) = 13514979 Aad 15 21 280 [Av Aa Adl = [P| = 95141042 0.Adl = 280 |ALN Ag M1 Aal = Ug) = 451420 Asn Aad [AL Ag Ag 9 Aah = [222 2 ‘Sit dung cng thite bao him va loai trir ta thm duge |Ay U Ag U Ag U Ay| = 216. Thanh thi, trong tap S c6 280 — 216 = 64 s6 khéng chia hét cho 2, 3, 5, 7. ‘Vi du 2 (Cong thitc ham Euler) Voi méi sé nguyén duong n > 1, ky higu 9(n) Ia s6 cde s6 nguyén duong bé hon n va nguyén t6 véi n. Ham ¢(n) duge goi la him Euler. Né dong mot vai trd quan trong trong nhiéu bai toan s6 hoc. Ching minh ring trong d6 71, P2, ...,Pm 1a tht cd céc ude nguyén té phan biét cia n, 44 pape cee Gadi Kj higu $ = {1,2,....n}. Ta dém xem trong tp S'c6 bao nbiéu s6 chia hét cho it ah&t mot trong céc 86 p1,....Pm Goi Ai = {k € S: k chia hét cho pi} (i = 1,2,...m). Khi dé Ai U Ap... Am la tap céc 86 chia hét cho it nh&t mot trong céc $6 pi, ..sPm- Ta cé n Pi:Pig--Piy 145, 14g. Aal Do dé theo dinh ly 1 JA, U.A2 U... Amel = n n Pi i & n PsP n fee YS pt Lists PuPa~Pis geek (® PLP Pm Vi @(n) chinh bing s6 cfc s6 khong chia hét cho tht c& cfc $6 pi,...Pm nén n— [AU A2U... Am = 2 Thiét ké cdc céng doan thich hgp Dé &p dung duge quy tii han didu cét yéu IA phai thiét ké mot mo hinh gdm viée thuc hign lign tiép cdc cong doan. Quy tée nhan phét biéu: V6i mi céch thyc hién 3 cong doan trudc thi cong doan thit k oé thé lam theo n, céch. Nhu vay: s6 céch thue hién 6 méi cOng doan phai khGng phy thudc vao cach ndo da duge thuc hién 4 cong doan truée d6.Thanh thit, muén sit dung duge quy téc nhn , trong mé hinh cia ta gém viée thyc hién lign tiép c&c cOng doan, 86 céch thyc hién & méi cOng doan I& phat nhu nhau véi moi céch da dge thyc hign 6 cOng doan tribe 46 Vi du 3 C6 4 ngudi A,B, C, D can chon vao chite giém déc, ké toan trudng va chi tich HDQT.Gié sit vige chon nhén sy phai théa man yéu cdu : Ong A khong thé duge chon lam giém déc, chite chit tich HDQT phai la ng C hoe Ong D. Héi cé bao nhieu ech chon, 45 Giai: C6 mét Idi gidi nhu sau: Viée chon ba vi trf giém déc, ké todn trudng va chi tich HDQT tién han theo 3 cong doan: Cong doan 1: Chon giém d6c: C6 3 cach chon chite gidm déc ( chon B,C,D). Cong doan 2: Chon K6 Tofn truéng : Cé ba céich chon ké toén trudng tit ba ngudi edn lai . Cong doan 3: Chon Chi tich HDQT C6 hai céch chon ( 6ng C hoe ong D) Theo quy téic nhan thi s6 céch A 3.3.2=18. Och nay khong déng: Vi s6 céch thyc hign cong doan 3 phy thudc vao két qué cia céc cong doan 1,2 trude dé : Néu 8 céc cong doan trudc , 6ng C va ong D khong dite chon thi cong doan 3 méi cé hai céch. Can néu C ho&e D da dugc chon thi 8 céng doan 3 chi c6 mét céch hose tham chi khong c6, ‘Tuy nhién néu ta thiét ké vie chon ba vi trf gidm déc, ké todn trudng va chi tich HDQT tién hanh theo 3 cong doan khéc thi vin c6 thé ép dung quy tée nhan, Cy thé Cong doan 1: Chon Chi tich HDQT: Luon c6 hai céch chon: C hog D. Gong doan 2: Chon Gidm déc: Ta Iudn o6 06 hai céch chon da é két qua cia cong doan 1 thé no Sau cong doan 1 cdn ba ngudi trong dé o6 ong A. Bé ong A ra ta cdn hai ngudi c6 thé chon vao chite Giém déc. Cong doan 3: Chon Ké Toén truéng : luon 6 hei céch ( tit hai ngudi cdn Iai ) Vay két qua 18 06 2.2.2=8 céch chon. Day Ia dép s6 ding, ‘Vi du 4 a) Gid six c6 8 van dong vien béng ban tham du mét gidi du. Trong vong dau cia gidi, ban t6 chic edn phan ra 4 cap du. Héi o6 bao mhiéu céch ghép thinh 4 cp diu? b) Gia sit c6 2n van dong vien béng ban tham dy mot gidi déu. Trong vong diu cia gidi, ban té chite cin phan ra n cp déu. Hdi ¢6 bao nhiéu céch ghép thinh n cép dau? ©) Tit b) chiing t8 ring v6i mdi n € N* ta c6 (n+1)(n-+2)(2n — 1)(2n) chia hét cho m. Giéi a) Ta thiét ké vige thyc hin chon theo cée céng doan sau: Cong doan 1: Chon 2 ngudi trong 8 ngudi lam thin c&p du thit nhit. CO CG} céch chon. Cong doan 2: Chon 2 ngudi trong 6 ngudi cdn lai dé lam thanh c&p du thit hai. C6 C8 c&ch chon. Cong doan 3: Chon 2 ngutai trong 4 ngudi cdn lai dé lam thanh c&p du thit ba. C6 C? céch chon. Céng dogn 4: C6 3 = 1 céch chon: Hai ngudi cdn lai sé lam thanh c§p déu thit tu. Theo quy tc nhan c6 C3.C3.C2C} céch chon. Vi thit ty 4 c&p du khong duge xét dén nén s6 céch ghép thinh 4 cp dau lA C3.03.03.03 = 105. 46 b) Ly luan tumg ty nhu trén, s6 céch ghép thanh n cap déu la _ hy Cyn C203 _ nl c) D8 bién adi (2n)! _ (n+1)(n +2)(2n — 1)(2n) nla * ViT I mot s6 nguyén dung nén cong thite nay chting t6 (n +1)(n+2)(2n — 1)(2n) chia hét cho 2” 3 Sit dung phép song 4nh C6 n ngwdi dén dy mét budi néi chuyén trong mot hoi tru’ng gém 200 ghé. Gid sit moi ngudi chi chiém mot ghé ngdi va méi ghé chi cé nhiéu nhét mot ngudi ngdi. Néu ta duge thong béo ring moi ngudi déu cé ché ngdi thi ta két luan duge n < 200 Néu ta biét them ring khong c6 ghé ndo tréng thi ta biét ngay JA n = 200. Néu c6 mot s6 ngudi phai dmg vi khong c6 ghé thi ta suy ram > 200. Nhuu vay cé thé xéc dinh hay uéc luong s6 phan tit cia mét tap hgp A nado dé thong qua mot tap hop B ma ta da biét 86 phan tit cita B nh’ mot phép tung ting (nh xa) gidia A voi B.. Day 14 mot phitong phap dude sit dung rat hiéu qua dé gidi nhiéu bai ton dém nang cao. Cho nh xa f: A> B, « Anh xa f duge goi 1a mét don 4nh néu véi hai phn tit bit ky a1,a2 € A, néu a; # ay thi f(a1) # f(aa). Tite Af(a1) = f(a2) + a1 = a2. * Anh xa f duge goi 1A mot ton énh néu vdi moi b € B déu tén tai a € A dé F(a) * Anh xa f duge goi lA mét song anh néu moi b € B, tén tai va duy nht a € A dé f(a) = b. Noi céch khéc f 14 song Anh khi va chi khi n6 déng thdi 18 don énh va toan énh . Dinh ly Cho A va B a hai tap hgp hitu hen. + Néu od mot don énh f : A B thi JA] < |B © Néu c6 mét toan anh f: A — B thi |A| > |B| * Néu c6 mot song énh f : A B thi [A] = [BI Vi dy 5 Cho tap A = {1,2,...,2n}. Mot tap con B cha A goi la mot tap can néu trong tap dé sé céc 86 chin va s6 cdc s6 lé bing nhau.( Tap réng 1A mdt tap cn vi sé. céc 86 chdn va s6 céc s6 1é trong tap rong déu bing 0). Héi cé A 6 chita bao nhiéu tap can? Ching han v6i n = 2,A = {1,2,3,4} . A c6 6 tap con cn Ia céc tap sau 0, (1, 2}, (1, 4}, {2, 3}, {3, 4}, (1,2, 3,4}. a7 Gidi: KY higu X = {2,4,...,2n} la tap hop tht od céc s6 chin cia A va Y = {1,3,...,2n—1} IB tap hop tht cd céc 36 18 cia A. Goi C 1a ho t&t cd ede tap can cha Ava D 18 ho céc tap con cite A cé ding n phan ti. Ta lap mot énh xa f titC vao D nhw sau: Gid sit B 1A mot tap cin . Ky higu By, By tuong ting la tap cée s6 chin vA tap céc $6 18 cia B. Khi d6 dat F(B) = BOY \ By). Do B la tap can nén|B;| = |Ba|.Thanh thit | f(B)| = |By|+|¥ \ Bal = [Bil +[¥|— 1Ba| = I¥| =n. Vay f(B) €D, Tiép theo ta ching minh f 14 mot song énh. + f la don énh: Gia sit f(B) = f(C). Suy ra By U(Y \ Ba) = Gn (¥\ G). Vi Bi, Cy la tap céc 86 chin ; (Y \ Ba), (Y \ C2) 1A tap eée s6 18 nén tit 46 suy ra By = C1; (¥ \ Ba) = (Y \ C2) . Do d6 B, = Cy; By =O + B=C.. + fT mot toan anh: Gid sit M € D la mot tap con cia A c6 n phan tit. . Ky hieu M1, Ma tuong ting la tap céc s6 chin va tap cfc s6 18 ctia M. Dat By = My; By = ¥\ Mp va B= B\U By. Ta cé 1Bal = [Mas |Bal = ¥| — [Ma] =n ~ [2M] = [MI ~ [Mal = [Ma Vay |Bil = |Bal do d6 B 1a mot tap can, Ré rang f(B) = ByA(¥\ Bs) = MyUM; = M. Vi c6 mot song anh gitta ho cfc tap cén va ho céc tap con c6 n phan tit cia A nén theo dinh If tren s6 cfc tap can cia A bing s6 cée tap con o6n phan tit cla A. Vaya 6 tht of 18 CR, tap can, Vi du 6. Cho trudc s6 nguyén duong n va s6 nguyén duong r tho& min r 1 ( do aig1—a; > 2). Dodd by < dy << Snr $l Ta dinh nghia f(A) = B Ta o6 B € B, do vay f 14 mot énh xa tit A vao B. Ta s& ching minh f 1a song anh. + f Ia don énh: Ty cong thite by = a; ~(i- 1) = a; i +1 suy rag; =b +i Do d6 néu f(A) = f(A’) thi A= A’ + £18 todn anh: Gid st B= {by < be < ++ 11010111 (1,4,1) — 10111101 (0,2,4) > o1101111 (3,0,3) — 11100111 Ro rang phép tung ting d6 18 mot don Snh. Nguge lai, voi mdi day 8 ky ty v6i 6 ky tu 1 va2 ky tu 0 khi ta dém tir tréi sang phai m& c6 :a s6 1 lien tiép ,s6 0, 0 86 1 lien tiép , s6 0 vac chit s6 1 lien tiép thi day dé sé ting voi bo (a, b,c) thod min a+b-+c= 6. Chang han day 10110111 sé ting véi bd (1,2,3) tite 18 ting voi sy Iya chon : 1 kem xodi,2 kem 50 cdla va 3 kem sita, Day 01011111 ting véi bo (0,1,5) tite 18 tng voi sy Iya chon 1 kem 56 cdla va. 5 kem sifa, Nhu vay ta da thiét lap mot song anh gitfa tap hop céc su Ifa chon véi tap hop cée day nhi phan d6 dai 8 trong d6 06 6 ky tu 1 vA2 ky tut 0. Do d6 86 eée suf Iya chon bing 86 céc day nhi phan d6 dai 8 trong d6 06 6 ky ty 1 va 2 ky ty 0. Mat khée mot day nhi phan d6 dai 8 voi 6 ky tu 1 va 2 ky tu 0 tuong ting véi céch chon 2 vi tri trong 8 vi tri dé ghi s6 0 ( 6 vi trf cdn lai ghi s6 1). Thanh thit c6 C? = 28 day nbj phan do dai 8 véi 6 ky tu 1 va 2 ky tut 0. Do dé s6 cde su lta chon 18 28. b)Mot sv Iya chon : " a kem xoai, b kem s6 c6 la va ckem sita " dugc ky higu béi mot b@ ba (a,b, c) trong dé a,b, c IA cde s6 nguyén duong thod man diéu kigna+b+c=6. 49 Voi m8i b9 (a,b, ) thod man diéu kign trén ta cho tuong ting véi b9 (2, y, 2) véi Premed tea Garena Ca ee} nguyén khong am thoi man ditu kien +y+2=a46+c-3=3, D8 kiém tre Hing day 1A mot phép song énh Billa tap céc b6 ba (a,b,c) trong dé a,b,c 1A céc s6 nguyén duong thod man diéu kin a+b+e=6. véi tap céc bo ba (2, y, 2) 1d cfc 86 nguyén khong am thod mn digu kién T+y+z=at+bt+c-3=3. B&ng suy luin tutong ty nhu cau a) ta tim duge s6 cdc su Twa chon 18 G2 = 10. Chit yj: Ta c6 bai todn téng quét sau: Mot cia hing kem 6 bén m loai kem . Mot nhém C6 nm ngudi vio an kem va goin c6c kem. Héi 8) Ho ¢6 tét cA bao nhieu su Iya chon? 5) Ho 6 tht 8 beo nhieu sy Ita chon trong &6 cdi m loni kem déu 6 mat? Dép sé: a)Cr1 BORG 4° Dang cong cu sé phic Vi du 8 Cho p lA mot sé nguyen t6 va m € N*. Gia sit po B= {(h1y bp) 0S m1: Dik =0 (mod Dp} a Xe dinh s6 phan tir cia tap [| Giéi V6i méi r = 0,1,...,p— 1 ta dink ngbia tap E, E, = {(h, P(e) = ED a*) = = ae ti ok mt Gia sit F(z) = D® ane” . Dat pa An = {(hy tps) iki =n} an = [Anl 50 Khi d6 E, = |E, Unssr (mod pn, Y l= Yo a. ner "(mod p) ner (mod p) Goi w 1A mot nghiém phitc tuy § cha o(2) = 2 +... +241=0.Tacdwi #1 WIS j wi" = wu. Viho {jm}, j = 1,2,..,p—1 a he thing du thu gon (modulo p) nen{w",i = 1,2,..,p— 1} = {w,j 1,2, ..p~ 1}. Suy ra F(w) = 1. Dat b = |B]. Vin =r (mod p) + w" =w" nén tit (1 ) suy ra = owe = Per @) 1= Fi = = Dat Q(z) = TPT} bea + bo ~ 1. Theo (2) voi mi nghiém w of g(z) ta c6 Q(w) = 0. Do 6 tdn tai hing s6 @ Q(x) = ag(x) + bya = +++ =b; = by — 1. Tadd Hence Trudng hop 2 plm. Khi dé wim 0> FW) =0+0= Fl) = Mg ane = DPT bud” 9 Opa C6 «Do vay v6i mi r = 0,1,2,...,p—1 ta IE) = ‘Tom lai Vi du 9 Cho p > 2 1a 6 nguyén t6 1é va 86 nguyén duong n > p. Xéo dinh s6 cic tap con A cia $= {1,2,...,.n} thod man hai digu kign sau i) |Al=p ii) S(A) = 0 (mod p) trong 46 S(A) 1a téng céc s6 nim trong A. Gidi V6i mdi r = 0,1, 2,....p— 1 goi my la 86 céc tp con A cia S thda man r (mod p) = Diet. Cé dinh mot nghigm phite bat ky w cia f(z). Ta 06 {u', 1,2,...,p~ 1} la p— 1 nghiém phan bigt ota f(x). (That vay nhyt trong vf du 8 da chi rau! £1, (WP = (uP)! = 1. Néuut ud ud = 1 pli — 7 mau thulin ) Do do p ~No(2) = [[(e- Xét da thite Q(2) = [T.y(x - w*) = D2, aia*. Theo dinh ly (3) > www? = (1) an p 1 p thi ng cia vi du trén voi n = 2p. QR=k (modp), 646k = [2] 5 Thiét lap quan hé truy héi ‘Vi dy 9 (Bai todn thép Ha noi) Tuyong truyén ring tai mot ngoi thap & HA noi cé mot tam dé bing ding trén d6 c6 dat ba chiée coc bing kim cithg.Litc khai thién lap dia, trén coc s6 1, Phat t8 Nhu Lai da xép 64 chiée dia bing ving 6 dwéng kinh khéc nhau sa0 cho cc dia 06 dutng kinh Ién hon xép & dui, céc dia 6 phia trén cing 6 trén cao cang nhé dan. Cac nha sur dude yéu cdu chuyén tt cd céc chiée dia d coc 56 1 sang coc 86 2 v6i quy t&c eau: - Méi lan chi duge chuyén di mot chiéc dia. ~ Trong qué trink di chuyén khong duge dit dia Ién lén trén dia nhé ( do dé cdn thiét phi c6 thém chiéc coc trung gian thit ba). Gid sit méi lan chuyén mot chiéc dia mt mot gidy. Héi céc nha su cn ft nhAt 1a bao nhiéu nim dé chuyén tit cA céc chiée dia & coc sé 1 sang coc 86 2? Giti Gia sit lie dau trén coc 86 1 06 m chiée did. Goi up 18 s6 In it nh&t dé chuyén tht cf cée chiée dia & coc 86 1 sang coc s6 2. Ta thit tinh mot vai gid tri cia uy Vin = 2. Ta cn thyc hién ba phép chuyén sau -Chuyén dia bé sang coc s6 3 - Chuyén dia lén sang coc s6 2. ~ Chuyén dia bé va coc 6 2 Vay = 3 Voi n = 3. Ta can thuc hign theo ba giai doan sau - Chuyén hai dia 6 phfa trén sang coc s6 3. Nhu da théy 6 trutmg hyp n= 2, ta cn 3 phép chuyén. 53 - Chuyén dia 1én nhAt sang coc s6 2 - Chuyén hai dia 4 coc sé 3 vé coc s6 2. Nhu da théy 6 trutmg hop n = 2 ta cin 3 phép chuyén. Vay ta cin 3+1+3=7 phép chuyén. Do d6 ug = 7. ‘Trudng hop n = 3 ggi ¥ cho ta thiét lap he thite truy héi ma day (up) phai thod man. Dé chuyén duge n chiéc dia theo quy tc trén ta phai thyc hién theo ba céng doan sau: - Cong doan 1: Chuyén (n — 1) dia 6 phia trén chiéc dia lon nhAt sang coc sé 3 theo quy téc trén. Ta can un; phép chuyén. Chiéc dia lén nhAt vin gitt nguyén 6 coc s6 1 Khi di chuyén (n — 1) chiée dia 6 tren né. ~ Cong dogn 2:Chuyén dia lén nhAt sang coc s6 2. - Cong doan 3: Chuyén (n — 1) dia coc 86 3 vé coc 56 2 va dat len trén chiée dia 16n nh&t Ta céin un; phép chuyén. Do vay dé chuyén n chiéc dia tix coc s6 1 sang coc 86 2 ta cin un + 1+ Unt = 2tm-1 +1. Vay ta c6 hé thite truy hdi sau Un = Qn + 1, Tut he thitc truy hdi nay ta cé thé lap duge cong thite cia s6 hang téng quét cia day. Bling quy nap dé chimg minh duoc Un = 2 = ‘V6i n = 64 thi ug, = 18.446.744.073.709.531615, D6 lt s6 lan chuyén dia mA céc nha sut phai thuc hign dé hoan thanh cong vige. Néu méi lan chuyén mot dia mét 1 giay thi cdn dén hon 500 t} nim céc nha sit méi chuyén duge tit c& 64 chiéc dia sang coc 36 2. ‘Vi du 10 (IMO 1979, bai s6 6) . Gia ott A va £ la hai dinh déi dign cia mot bét gide déu. Mot con éch bat dau nhdy titdinh A. Tai mdi dinh cita bat giée (trit dinh B) , m6i ct nhiy con éch chi c6 thé nhay t6i hai dinh ké véi dinh 6. Khi con éh nhay vdo din E né sé bi ket vinh vién 4 dé. Cho trude s6 nguyen duong n . Héi véi n oft nhdy, 06 bao nhiéu cach dé con éch nhéy vao dinh B. Gidi . Goi ay 1886 cach dé con éch nhay vao dinh B. Dé thfy a; = a2 = a3 = 0; a4 = 2 Gia st tir A theo chigu kim déng hé cdc dinh lan lugt ls A> BC D+ EB — FG — H — A. Tit A con éch dén B phai qua mot s6 Ié bu6c;Ttr B con éch dén C phai qua m@t s6 18 bude.Tit C con éch dén D phai qua mot 6 18 buée.Tit D con éch dén E phai qua mét 86 lé buéc. Vay s6 buée dén E ditt khost phai 1A mot s6 chin, Néi cach khée néu n 18 thi kh6ng ¢6 céch ndo nhay vao B. Vay azx-1 = 0. Ta cn tinh any, k > 1 Xuét phat tir A,véi hai buée nhdy dau tién con éch 06 thé 06 cée céch sau NASBA NQAGHoA 2QAWBAC )ASHAG Néu theo céch 1) thi s6 céch tdi E 1a az4-2.Nén theo céch 2) thi s6 céch téi E 1a 024-2. Goi Cn, gn ln lut 18 s6 cach dé con ch, xudt phat tuttong ting tir C,G, nhay vao dinh E voi n ci nhay. Vi ly do déi xing ta c6 cy = ga. Vay néu theo céch 3) thi 6 céch. tdi E 1A c2p-2; néu theo cach 4) thi s6 céch tdi E 1A goe-2 . Theo quy tc cong ta c6 Ang = Og4—2 + Oae—2 + Cok-2 + Gok-2 = 22-2 + 2eyp-2 (5) Xuét phét tir C,véi hai buée nhdy dau tién con éch 06 thé c6 céc cAch sau : 1)O4+BOA %wW)C4+B4C 3) C9 DC 4c) C4 D> EB Néu theo céch 1c) thi s6 céch téi E Ia az4-2,Néu theo céch 2c) thi s6 céch t6i 1a cax-2.Néu theo céch 3c) thi s6 céch ti E 18 c7,~2.Néu theo céch 4c) thi s6 céch t6i E 1a 0. Theo quy téc cong ta c6 Cae = 24-2 + 2erk-2 6) ‘Tix (5) va (6) riit ra cm, = age — O24-2 > Crk-2 = 24-2 — 24-4. Thay vio (5) ta duge nx = 4a2k-2 ~ 2are—4 Dt uy = are ta 06 Up = Attn ~ 2u_-2) tH = Oe = 05 Ue = Oy = trinh dic trimg ta di dén cong thiic sau (2+ v8) = (2 vay . Bling céch gidi phuong a te k= 1,2,.0 Vi du 11 Cho sé nguyén duong n va $ = {1,2,...,n}. Tim s6 céc tap con (ké cd tap r6ng) cia S m& khong chtia hai s6 nguyen duong lién tiép. Gidi Goi aq 1& 86 phai tim. D@ thy a; = 2,a = 3,a3 = 5. Ching han véin = 3.06 5 tap con thod 1a 0; {1}; {2}; {3}; {153}. Goi A, 1a ho cde tap con cé tinh chat da néu. M6i tap A © Ang2 gdm hai loai : Loai 1 gém céc tap chita'n + 2.Loai 2 gdm céc tap khong chifa n +2. Néu A la tap loai 1 thi A khong chifa n + 1. Do d6 néu bd di khdi A phin tirn +2 ta duge mot tap con cia A,. Ngugc lai véi mi tap con B cia Ay thi tap A= BU {n+ 2} 1a tap loai 1 cla Anse. Thanh thit s6 tap loai 1 18 a,. Moi tap loai 2 76 rang 14 mét tap con cila Any: vA ngugc lai. Thanh thit s6 tap loai 2 18 an41. Do dé ta cé quan hé sau Ong? = Ong +n Mat khéc véi day Fibonacy ta c6 Fas2 = Fai + Fn. Via = Fy = 2:09 = Fy = 3503 = Fy =5 tasuy ra a, = Fuso. Vay am? — pt? vB Oy = Fuga bd6a=48,a= eo ‘Vi dy 12 Cho s6 nguyen duong n va S = {1,2,....n}. Goi cy 1a 86 cdc tap con cia S ma chia ding hai sé nguyén dutong lign tiép . Chimg minh rng 2nFnoi — (n+ 1)Fa fa 55 Gidi Goi C, 18 ho ce tap con 6 tinh ch&t di néu. Méi tap C © Casa gdm ba loai : Losi 1 gém céc tap chita n+ 2,n +1. Loai 2 gm cée tap khong chitan +2. Loai 31a. céc tap chita n+ 2 nhutng khong chtta n+ 1. +) Néu C la tap loai 1 thi C cling khong chita n ( vi néu C chifan thi C chita 2 cp sé aguyén lien tiép 1A.n,n+1 vAn-+1,n+2) . Bé di khi C phan titn,n+1,n-+2 ta duge mot tap con cita {1,2,....m—1} khng chia hai s6 nguyen dung lién tiép, do dé n6 IA phan tit cia An1.Nguge lai véi méi tap con A cia A, thi tip C= AU{n+2,n+1} 18 tap loai 1 cita Cu42. Phép tuong tng ndy 1A song anh. Thanh thit s6 t€p loai 1 1a Qn = Fri +) M@i tap loai 2 15 rang I mét t&p con cia Caya VA ngudc lai. Thanh thit s6 tap Toai 2 18 cay. +) Néu C Ia tap loai 3 thi C khong chtta n+1. Do 46 néu bé di khéi C phan tin +2 ta duge mot tap con cita.Cy.Nguge Iai voi mbi tap con B cia C, thi tap C = BU(n+2} 1 tap loai 3 cia C,42. Thanh thi ta c6 he thite sau uta = ata + on + Fret Ti d6 bing quy nap, sit dung he.thitc truy hdi cia day Fibonaci ta c6 céng thie néu tren, £ te Xay dung song anh gidi mot sé bai todn td hop Huynh Tén Chau (THPT Chuyen Luong Vin Chénh, Phi Yen) ‘Trd ngai lén nh&t khi gidi mot bai toén té hgp 1A xée dinh huéng di. Ré rang dé c6 kha ning dinh huéng t6t thi vige rén luyén e&e phutong phép tiép cn Ia rét can thidt. Bai viét ndy gidi thigu véi cdc ben doc mét phutong phép hiéu qua trong nhiéu bai toan té hop ma ta tam goi lA phwong phdp song énh. Phuong phép nay dya trén két qué hién nhién sau: "Néu cé mot song dnh di tir mot tap hitu han X t6i mot tap hitu han Y tha luc luong (tic s6 phén tt) cia X va Y bing nhau" 1, Mot céch tu nhién, két qua trén hudng ching ta dén ting sit dung song énh dé so sinh luc lugag hai tp hop. Bai toan 1 (Vo dich Lién X0) C6 mot nhém ngudi ma trong dé, méi cap khong quen nhau c6 ding hai ngudi quen chung, cdn méi cip khong quen nhau thi khong cb ngwdi quen chung. Ching minh ring 36 ngudt quen ctia méi ngudi la nhw nhaw Ldi gidi. Gid sit a quen b va tap cée ngudi quen ciia a va b (khong ké a,b) 1A A va B Méi ngu’i a! thuge A sé quen véi duy nhét mot ngudi thude B (do a’ va b khong quen nhau, hon nia ho da cé mot ngudi quen chung JA a). Tuong ty, méi ngudi thude B cing quen véi duy nht mOt ngudi thuge A. Vay tdn tai mot song nh di tir A t6i B, tite a va b c6 86 ngwoi quen bing nhau. Néu a khong quen 6 thi tén tai c quen cA a va b, do dé s6 ngudi quen cia a va b bing nhau do cing bing sé ngudi quen ciia c. Bai toan 2 (Trung Quéc - 1997) Trong céc sau nhi phan cé do dain, goi am la sb cdc au khong chita qud 2 36 lien tiép 0,1,0 va by la s6 cdc cau khong chiia 4 sé lién tip 0,0,1,1 hodic 1,1,0,0. Chiing minh ring: buss = 2am. Léi gidi. Ta goi mot xfia thudc loai A néu n6 khong chita 3 sé lien tiép 0,1,0 va got mét xdu thudc loai B néu né khéng chita 4 s6 hang lién tiép 0,0, 1,1 hole 1,1, 0,0. V6i mdi xéu X = (21, 22)...,n), ta xy dug f(X) = (ys, yo,--.,Ynt1) abit sau: = 0, ve = 21 +22 +--+ + a4 (mod 2). R6 rang X chita 3 sé lien tiép 0, 1,0 khi va chi khi f(X) chita 4 sé hang lien tiép 0,0,1,1 hoi 1,1,0,0, titc IA X thuéc loai A khi va chi khi f(X) thuge B. 37 Vay f I mot song nh di tit tap céc xau loai A do dai n dén tap cdc xau loai B do dain +1 ma bit diu bing 0. Nhung tit mdi xau X thude B ta nhan duge mot xau X cling thuge B bang céch déi céc phan tit cia X theo quy té&e 1 — 0,0 + 1 nén s6 cée xau loai B c6 d6 dain +1 gp déi s6 céc xdu loai B dé dai n +1 ma bat dau bing sé 0. Ti 6 ta c6 dpem. 2. Tit vige so sénh Iyc lugng céc tép hgp, phuong phép song énh c6 thé gitip ching ta dém s6 phan tit ctia mot tap thong qua sit so sinh Ive lugng ciia tep dé véi mot tap khéc ma ta da biét s6 phan tit cia n6. Bai ton 3 (V6 dich Ucraina - 1996) Goi M la ctc s6 nguyén duong viét trong hé thap phan cé 2n chit sd, trong dé cé n chit sd 1 van chit sé 2. Goi N la sé tat cd cdc 86 viét trong hé thap phan cé n chit s6, trong dé chi cé chit s6 1,2,3,4 va sé chit s6 1 bang 56 chit s6 2. Chiing minh ring M = N. Lai gidi 1. Voi méi s6 06 n chit s6 gdm céc chit s6 1,2,3,4 va s6 chit s6 1 bing 86 chit s6 2, ta "nhan doi" thanh s6 c6 2n chit s6 theo quy t&c sau: dau tién, hai phién ban cia s6 ny duge viét ké nhau thanh s6 6 hai chit 56, sau d6 cfc chit s6 3 6 n chit s6 dau vA céc chit 56 4 4 n chit s6 sau duoc déi thanh chit s6 1, ce chit 86 3 6 n chit sé sau va cdc chit sé 4 6 n chit s6 du duge déi thanh chit s6 2. Vi du: 1234142 > 12341421234142 — 1212122121112, Nhut thé, ta thu duge mt s6 c6 ding n chit s6 1 van chit 96 2. Ré rang day 14 mot don énh. Dé chiing minh day 1A mot song énh, ta xy dig énh xa nguge nh sau: voi mbi 6 c6 n chit a6 1 vA n chit 96 2, ta o&t n chit s6 du van chit s6 cuéi rdi cong ching theo cot véi quy te: 1+1= 1, 2+2=2,14+2=3, 2+1=4, vata thu duoc mot 86 c6 n chit s6 gdm céc chit s6 1,2,3,4 vdi s6 chit 56 1 bing 86 céc 86 2. Vi du 1212122129112 — 1234142 nu sau: 1212122 1221112 234142 Nhu thé song énh gitta hai tap hop da duge thiét lap va ta c6 M = N. Céch xay dung song Anh nhv trén khé dep song hoi ciu ky. Ching toi da tim ra mot loi gidi ngén gon hon nhwt sau: Li gidi 2. Vk € S = {0,1,...,n}, ta thuc hign 2 lin (d9c lap véi nhau) vige dénh déu k trong n vi trf trén mot hang, Sau 46, 3 méi vi tri ta ghi: chit s6 3 néu duge dénh déu 3 lan, chit s6 4 néu khong duge dénh déu lan nao, chit s6 1 néu chi duge dénh du lin dau va chit s6 2 néu chi dude dénh déu lén sau. R6 rang Khi cho & chay trén S thi s6 t&t c& céc s6 thu duge chfnh 18 NV. Hon nita, Vk € S thi c6 thé xem s6 céch dénb du lan du 1a s6 céch chon k trong 86 n vi tri, cm 86 céch dénh déu lén sau 1a s6 céch chon n — k vi tri trong n vi tri. Do d6 tong s6 céch dénh du khi k chay trén S ding bing s6 céch chon n phan tit tit 2n phan tit (tic 1a C},), va s6 nay chinh la M. Vay ta c6 diéu phai ching minh. 58 Bai toan 4 Cho céc 36 nguyén duong n,k udin > k. Xét phép todn f déi véi bo sip thit tu X = (a1,...,aq) nhw sau: mdi lin chon k s6 lién tiép tiy 9 trong X va déi déu ctia chting. Tim 86 céc b6 thit ty X = (x,,...,tn) thda man cdc diéu kién: i) Moi phan tit ciia X déu thude tap {0,1}. ii) Co thé thuc hign hitu han lan phép todn f dé tir X mhan dugc b6 (1,2,-..,1) Ldi giai. Xét bo thit ty X = (x1,...,a,) tity Ta c6 2 nhan xét sau: 1) 06 ding n ~ k +1 nh6m k sé lien tiép. 2) Sau mét s6 chin lan thyc hign phép ton f cho mot nh6m k s6 lign tiép trong X thi gid tri & s6 dé khong déi. Nhu vay, m6i phuong én thyc hign hiu han lan phép ton f cho X tuong tg véi mét b6 nhj phan A = (a;,az,...,a,), trong 46 @, tinh theo modun 2 ciia s6 ln thyc hign f cho nh6m & 86 lién tiép (2,,2441,...,Tize-1), va X tr thanh (apace aaa CIEE MEG (AH). 3. Ciing tir vige so sénh luc lugng céc tap hop, phuong phép song anh 14 mot cong cu die luc dé thiét lap va ching minh céc cong thite t8 hop. Thong thuong, ngudi ta xay ding mét song 4th di tit mot tap vao chinh n6, vi nguyén tic 6 day c6 thé phat bigu nhu sau: "Khi dém s6 phan tit mot tap hgp bling nhiéu céch thi cée két qua thu duge bing nhau". Chang han tif bai ton 3, néu ta tinh N theo céch "truyén théng": c6 CC. céch chon vi tri cho i chit s6 1 vA i chit s6 2, con Iai 2"-* céich chon vi tri cho cée chit 56 3 va 4, véi i chay tir 0 téi il thi ta da ching minh duge mot ding thite khé. that vi boa LAC 0 Cp. Bai toan 5 Ching minh ring udin € N thi Oh, = (C8)*+(CR)? +--+ (C8)?. Léi gidi. Ta dém s6 céch chon n phan tit tt mot tap gdm 2n phan tit theo hai céch. Céch thit nhét: méi lan chon ran phan tit, khi d6 86 oéch hién nhién 1A C,. Céch thit hai: truéc hét ta chia tap 2n phan tit thinh hai tap con, m6i tap gdm n phan ti; sau d6 chon tir tap con thi nhat & phan ti (c6 C* céch chon) vi chon tit tap con thit hain—k phan tit (c6 Op-* = Cx cach chon), ta s8 c6 (C#)? cach chon; cuéi cing cho & chay tit 0 tdi n ta duge téng s6 céch chon can tim 1 (C2)?+(Cl)?+----+ (C8)? Tir d6 ta 06 dpem. Bai ton 6 Chting minh ring vdim,n € Z+, m> k thi Cran = Cn On + Or Ons 2 + C8, Chay 59 Lai gidi. Ta dém 36 céc bo s6 nguyen T = (a1,02,...,amsngi-t) VOi1 Crane is is Cée lug thita cia 2 goi ta lien tudng dén s6 dén sb tap con mét tap hgp. ‘Trong céc tap con cia tap $ = {1,2,...,.m+n+1}, théy 06 C#,,2"-* tap dang {01,02,.-. anti}, (1 <é< m4 1) trong d6 a < az < ++ < any Worn =n tha d V6i 0 < k 1) chit s6 théa man céc diéu ign: i) N gm ode chit 86 {1,2,4,5} va hiéu 2 chit 36 lién tiép luon lon hon 1 ii) N chia hét cho 11. Bai tap 6 Dat Tinh 62 my ' i prasanioeriinitonerns ites rw wis Sith Phuong phap thiét lap hé thitc truy héi trong té hop Huynh Tén Chau Mot trong nhtng phuong phap o6 higu qua dé gidi bai todn td hop la thiét lap he thie tray héi. Noi dung co bin cia phuong phép nay nhu sau: Thay vi ta dém trye tiép J(n) theo yeu cu bai toén, ta sé thiét lap he thie quan he gita f(n), f(n —1),... d tir dé tinh duge f(r). Bai todn 1 (Bungari - 1995) Cho sé nguyénn > 2. Hay tim s6 cdc hodn vj (a1, 02... ,an) ctia 1,2,...,m sao cho tdn tai duy nhét mot chi 56 i € {1,2,...,.n— 1} théa a; > ais. Bai gidi. Goi S,, 1A s6 céc hodn vj théa diéu kien bai todn. Dé ¥ ring s6 céc hoén vj mA nla Sq1. Con 86 céc hodn vj (a1,a2,...,dq) Vi aj =n (1 1). Ré rang a; An—1 (i= 1,2,...,h) nén s6 céc tap con nhu vay la: |Sy_o| + 1. Do vay: [Sa] = [Smal + |Sn-2| +1 Voi chit ¥ |Sp| = 2, [53] = 4 ta cé: tS Mat khée 86 tap con khong réng ofa tap {1,2,...,n} 18 2" — 1. Vay 86 tap con ma trong méi t@p con khong ¢6 2 phan ti nio la hai s6 nguyen lign tiép 1a: PA 4) 2 Bai ton 4 06 n qué bong biyba,...,by vd In hép hy,ha,...hon. Biét ring qué bing by (i = 1,2,...,n) chi bd vdo duge ofc hop hi,ha,...,ha:. Héi cb bao nhieu cach bd K(1 1 thi sinh ngdi trén mot bai trom, Héi cd bao nhiéu ctich phat dé sao cho hai thé sinh ngéi canh nhau luén cé dé khac nhau, biét ring trong ngan hang dé 6 ding m (m > 1) dé va hién nhién méi dé cé nhiéu ban. Bai giai. Nhan zét: Do thi sinh ng@i theo vong tron, nén mot cich tu nhién ching ta nghi t6i vige tim cdch "cit" vong tron thinh hang thing. Céch 1. Ky higu Py 18 s6 céch phat dé hgp Ig cho n hoc sinh a1, a2, ...,aq ngdi theo vong tron (mot céich phat dé duge coi 1a hgp 1é néu méi thf sinh duge nhén chi mot dé va hai thf sinh bat ky ngdi gin nhau thi nhan duge hai loai 48 khéc nhau) Ta viét a; = a; (i j) néu a; va ay cing logi dé va a; # a; trong trutng hop ngudc lai ta ching mink: Pasi = (m~ 2)Py + (m~1)Poa @ Xét mét céch phét dé hop le cho n + 1 thf sinh ay,a9,..., 41. Néu ay # ap thi bd an41 di ta c6 mot cach phat dé hop lé cho n thi sinh (a1, @2,..., a), va 06 (m — 2) céch phét d8 cho an41 Néu a1 = ay thi ta b6 Gn4s Va dq di ta c6 mét céch ph&t dé hgp 1é cho (n — 1) thi sinh (41, a2,...,@n-1). Ta ¢6 (m— 1) céch phét d8 cho (aq, @n41) dé hop 18 voi an = a). Vay ta 06 duge (1). Mat Kkhée dé thay: Pp = m(m~ 1); Py = m(m—1)(m 2). B&ng quy nap ta ching minh duge: P, = (m — 1)" + (m—1)(-1)", C&ch 2. Bai toan tuong duong véi viée dém s6 cée day (a) (i = 1,2,...,n) thda: a; € M = {1,2,...,m}Wi=1,2,....n va ay f 02,02 £ G3,--. 50a #2 ‘Trong tap hgp céc day (a;) théaa; € M, Vi =1,2,...,nva.a, # a2, a2 ¢ a3,...50, # a1, B01 An, By lin lugt la tap hop céc day (a;) mA a, # Gn VA a; = an, Do v6i méi day thude By, néu bé di s6 a, thi ta duge mot day thuge A, nén [Bal = [An]. Mat khée, dé thay |An| + [Bal = _m(m —1)"-! (do a, ¢6 m céch chon, aj41 ¢6 (m ~ 1) céch chon khée a; v6i moi i= 1,2,...,n— 1) Vay |An| = m(m — 1)" ~ [Anal ‘V6i chit ¥ |Ay = m(m — 1) ta duge: |An| = (m— 1)" + (m—1)(-1)". D6 chinh Ia dép s6 can tim, han zét: Thue chAt c& hai loi gidi JA nhu nhau. Nhung voi 18i gidi 2, "dudng nhu" te di hinh thanh nén mot huéng giai méi. Dé th4y rd higu luc cia hudng méi ndy, ta dén v6i bai tofin sau: Bai todn 6 (IMO - 1979) Cho A vd E la hai dinh déi tam ctia mot hinh tam canh déu. C6 mét con éch bét déu nhdy tit A. Tai bat cit dink nao trit E, éch c6 thé tdi mt trong hai dinh ké. Néu éch nhdy tdi E thi né ditng lai 6 46. Goi ay la sd dudng di phan 65 biét ciia ding n bude nhdy dé éch nhdy tx A dén B. Ching minh ring: Bart =0, Oma al(24+ v9)" -(2- vA] Bai giai. Ki hiéu céc dinh nhu hinh ve. ans = 0 1A hién nhién. Goi b, 1a s6 dung di tit C t6i E qua n buéc nhay. Qua 2 bude diu tién, éch c6 thé v8 A ho&e dén C hoke G. Do do: Gan = 20-2 + bana Vn > 0 (1) Tit C (hode G), sau hai buée nhay éch c6 thé vé lai C hose téi A, néu n > 2, do dé: an = Qan2 + Om-2 Yn >1 (2) Tit (1) va (2) suy ra: Gan = 402n—2 — 2dan—4 Cang vi a2 = 0, a4 = 2 ta dé dang c6 diéu phai chttng minh. Bai toan 7 (Dy tuyén IMO - 1996) Cho bing 6 vudng n x n (n > 1). Hoi 0b bao nhiéu céich dénh ddu cdc 6 vudng trong bing sao cho trong méi hinh wudng 2x 2 cb ding 26 vudng duoc dank déu. (Hai cdch dank déu duge coi la hdc nhau néu c6 mét 6 vuéng no dé ma trong céich nay thi duge dénk déu cén trong céch kia thi khong) Bai gidi. Goi S,, 1a s6 céch dénh du trong bing n xn. Xét tap T gdm che 6 vudng nim trong ct cuéi cing (tinh tir phai sang) va hing cuéi cing (tinh tit trén xudng), ta Boi A, 1A céc céch dénh déu ma o6 hai 6 vudng ké nhau trong T cing dutge dénh déu hole ching khéng duge dénh dau va B, IA cée céch dénh déu ma céc 6 vudng trong T duge dénh diu xen ké. Dé théy mai céich dénh déu thude By sé img véi mot céch dénh du thude By, con méi céch dénh déu thuge A, sé tmg véi mot céch dénh dau thudc Ani vA mot céch dénh dau thudc By. (Digu nay suy ra khi xét bang 6 vudng (n — 1) x (n— 1) 06 duge tix bing n x n sau khi bé 7) es f ‘ f Tir dé ta c6: [Bul =|Bn-al, [Ant = [nal +1Ba-il. (n> 2) Ma Sp =|An| +|Bql Vn > 1, nén S, = 254-1 — Spa Wn > 3. Dé thay ring: Sp = 6, Sj = 14. Tw d6 bang quy nap ta cé: S, = 8n —10, Vn > 2. Bai toan 8 Tim cic sé nguyén duong n théa man: i) n 6 1000 chit sé. ti) Tat cé cde chit s6 cian la ie, tii) Hiéu ctia 2 36 lién tiép bat ky cia n ludn bang 2. Bai gidi. Trong tap hop Si céc s6 nguyén duong n c6 k chit s6 théa (ii) va (ii), goi At, Br, Ck, Di, Ex lan hot 1a tap hop cée s6 tan cing bai 1,3,5,7,9. Tix méi s6 thude Ay néu ta bd di chit s6 tan cling thi nhan duge mot s6 thude Bea, mat khdc tit méi s6 thudc By-1 néu ta bé sung thém s6 1 lam chit s6 tan cing thi nhan duge mot s6 thuge Ag, do dé |Ae| = |By-r|. TH mbi s6 thuge By néu ta bé di chit s6 tan cing thi nhan duge mét s6 thude Ay; hoc C1, néu ta bé sung them 36 3 lam chit s6 tan cing thi nhan duge mot s6 thude By, do a6: |Be| = |Ax—al + |Ck2| Twong tut |Cx = |Be-a| + |De-al, [Dil = Ceal + [Baal va [Bx Sit dung 5 ding thie trén, bing céch thé lien tuc, ta 6: [Daal (Vk > 1) [Si] = |e] + [Bel + [Cel + [Dal + [Zed = |Aeal + 2]Be1] + 21Ceal + 2)Dy-al + [Bea] = Aga] + 3|Bx-2| + 4|Cx-2| + 3[Di-2] + 2|Ex-2| = 3]Aj-a| + 6|Ba-s| + 6|Ci—s| + 6|De-s| + 3121-3] Suy ra: [Se] = 3]S,-2 (Wk > 3). Do [52] = 8 nén |Siooo] = 8.3! ‘Tuong ty nh trén ta c6: Bai toan 9 Tim sé cic bd sé nguyén (a1, a2,...,dn) (n> 1) théa man: ail <1, Vi=1,2,...,0 lax—aiyal <1, Wi=1,2,....2-1 Bai gidi. Trong tap S, gdm céc bn s6 nguyén (a), a2,...., an) tha d& bai, goi An, Bas Cn 1a tap hop ee bd ¢6 a, bing —1,0,1 tuong ting, Ta c6 ngay: |S,] = |An| + |Bn| + [Cal M&t khéc, dé thay tix mdi bé thudc A, hoe B, ta c6 thé bé sung Qny1 = —1 dé duuge mot b6 thude Ans nén |Ansil = [Anl + [Bal Titong tu ta c6: |Ca4i| = |Cn| + [Bal va [Basal [nl + [Bal + 1Cn| Spl. 67 Tir d6 ta c6: [Snot] = [Antal + [Basa] + Crs] = (|Anl +1Bal +[Cal) +1 Bust + | Bal = 2|Sn] + [Spal . Két hgp v6i |S2| = 3, |So| = 7 ta tinh duge: joy) =< GEVO + = vIH uo Re Tacé thé phét biéu bai todn dudi dang phite tap hon, ching han: Bai toan 10 Tim sé cde da thife P,(x) bac n chén théa man céc diéu kién sau: i) Cée he 86 cia Pa(x) thudc tap {0, +1}. ii) Ton tai da thite Q(z) ciing cb céc hé 36 thude tp {0,41} sao cho P,(z) = (#7 -1)Q(z) Cu6i cing xin néu ra mot s6 bai tap dé ban doc rén luyén. Bai tap 1 (VMO - 1997) N dudng tron chia mat phéng ra lam bao nhiéu phan néu b&t cit cap hai dudng tron nao ciing cit nhau tai hai diém phan biet va khong c6 ba dudng trén nao 6 giao diém chung. Bai tap 2 Cho n la sé nguyen dung. Dat $ = my + cay2 +--+ + tat trong dé iy: (@ = 1,2,....n) nhdn gid tri 0 hode 1. Goi A(n),B(n) lan lugt la sé 2n-b6 (2iy-+ stn thy --1Yn) €6 Sle v8 chin, Ching minh: A(n)_ (2"~1) (—) ay Bai tap 3 Ky higu M = {—1,0,1}. Tum 36 cdc b6 (a1,...,an) théa: 1. a; thude M v6i moi i= 1,2, 22a; —ai41 thude M véi moi n. s2ye.yn 1 Bai tap 4 Cho trudc 36 nguyén duong n. Xét tat cd cdc téng S=ay ttayat---+2nYn vdi x,y € {0,1}, i= Tn Goi Sp la s6 téng lé, Py 1a 86 tong chin. Ching minh: Sa 1 PPS =m Y tung giai va su tudng minh Idi gidi qua mét sé bai todn té hop Lé Van Quang ‘Trong céc ky thi IMO, céc bai toan té hgp (Combinatorics) duge dt trutde dau bai bing chit "C". Vi du: Bai 6 IMO 2005 do Rumani dé nghi duge ghi: C6(ROM). Khi doc dau bai ctia cée BTTH (Bai tosn té hp) thi hoc sinh déu cé thé hiéu céc gid thiét va két uan kha dé dang, nhung gidi duge chting IA diéu kh6 khan. Tix bang két qué diém cho thay s6 hoc sinh gii duge diém t6i da rét it, didu dé chiing t6 day 1A loai bai todn kh6, tham chi c6 trong tay 1d gidi clia téc gid ra bai ton dé thi khong phai hoc sinh nao cing hiéu day dii va cin ké ldi gidi. Ning ngudi ty gidi bai toan dé bing mot céch khée thutng hiéu dugc Idi gidi cia téc gid mot céch khé dé dang. Tai sao lai nhut vay? Mot sé BTTH thutng dé cAp mot s6 yéu t6 rang budc theo nhing quy tée nto dé. ‘Yeu cdu cita bai todn 1 dénh gié mot dai lugng nado dé lién quan dén céc yéu t6 di dé cap, hoke ching minh m9t quy t&e n&o dé ludn thye hién duge, hotic ching minh mot quy lugt mo d6 nghiem ding. Latoc dé tyr nhién dé tiép cén vige gidi loai bai tofn nay da duge hinh thanh cho hoc sinh tit cée lop dui gdm cée bude: 1. Chon an dé mo t& céc yéu t6 trong dau bai think mot phuong trinh, mot bét phuong trinh ho&%c mét hé hén hgp chita an da chon. 2. Xitly céec diéu vita mo ta theo yéu cu ciia bai todn bing céch giai ra nghiém hote bién déi thinh nhimg két qua gitip cho vige hinh thanh quy téc hay quy luat théa yeu cdu bai todn. ‘Tir ¥ tudng gidi nhu thé thi khau then chét nb&t 1a thé hien tuéng minh ra mot loi gidi cu thé. Do bai todn khé, ngudi gidi duoc bai toan chic chin phai chi ra duge méi quan hé ndi tai cla céc yéu t6 trong bai toén thong qua cdc ky nang bién déi tinh x40 hoke nhiing nhan xét tinh té, bin chét nhét tix he da mo ta dugc. Ngay 6 buéc 1, vige khéo chon dn, ho&c dit thém dn phy hoe tich hgp cde yéu t trong dau bai... IA sy séng tao rét c& biét riéng cia ngudi gidi. Hoan thanh buéc 1 da 18 mét thanh cong ma khong phai hoc sinh ndo cing lam tét, nhung diéu cét yéu la xit 1y thanh cong d bude 2. Trong bude nay thing nay sinh mét 86 vin d8 1a céc két qué thu duge thug 1A do cée phép bién déi hé qua. Vige khdo sét nguge lai 1A can thiét, hokc it ra gidi quyét duge vin dé tdn tai tinh huéng ma da chi ra, Dua ra mot vi du cu thé dé ching td tdn tai tinh huéng cing khong phai d@ dang, 69 dn tao duce mot quy trinh hgp ly, chit ché, o6 he théng dé xay dung duge tinh huéng 6i lic lai khé hon yéu cu cia diu bai. Céc BTTH nay déu do céc nha toén hoc ling danh trén thé gidi séng téc nn trong Tai gidi cia ho thudng thong béo mot khém phé méi vé tri thttc todn, mot "bit bién" no dé, hoge kién thiét mot thuat tosn nao dé... Doc cée Idi gidi cia ho, hoc sinh hoc tap duge nhiing céch dat vin dé mot céch sdng tao, nhitng ky ning bién déi dieu Tuyén béc thay, nhiing hogt dong vé tich hgp céc dit kign rigng 1é thanh nhitng két qua su sé ma tir dé 06 thé dua ra nhing két luan x4c dng, céc khéing dinh ma ho thiting dat ten 1a cde "bé da". Cing ¥ tung gidi nhung c6 thé c6 nhiéu céch dé thuc hign su tudng minh Idi gidi, Trong 6 c6 Idi gigi mA hoe sinh cho 1a kh6 hiéu. Viee goi ¥ cho hoc sinh mét tung gidi VA dong vién hoc sinh né Iyc thyc hign theo eéch cia minh, dé tuéng minh ra mot Ji gidi cu thé cho BTTH, phai chang la.céch hop ly dé gidp cho céc em hoc sinh mdi bat du lam quen véi céc BTTH héc bite nay. Sau day 1A céc bai todn minh hoa cho eée diéu vita dé cap. Ba BITH duge chon minh hoa nim trong céc ky thi: Chon hoc sinh gidi Quéc gia 2005 cita Viet Nam, USAMO lan thtt 30 va, IMO 2005. Chiing ta s8 xem xét ching & hai Khia canh: ¥ tung gidi va su tudng minh Idi gidi. Bai toan 1 (HSGQG 2005, Bai 3) Trong mat phiing, cho bat gide Idi AiArAaAAsAgArAg ma khong 06 ba dudng chéo nao cia né cét nhau tai mét diém. Ta 901 mbi giao diém ciia hai dudng chéo ciia bét gide la mot mit. Xét cde tit gidc Idi ma méi tit gide déu c6 ci bén dink la dinh cia bat giée da cho. Ta goi méi tit gic nhu vay la ttt giée con. Hay tim s6 nguyen n nhé nhét c6 tinh chat: cb thé 16 mau nit sao cho vdi moi i,k € {1,2,3,4,5,6,7,8} vai A k, néu hy higu s(i,k) la 86 tt gide con mhan Ay Ay lam dinh va déng thoi cé giao diém hai dudng chéo la mét nit da duge +6 mau thi tht cd cho gid tri (i,k) déu bing nhau. Giai. Goi n 1a s6 nguyén nhé nh&t thod bai todn. Ta cé 3(i,k) = 9(1,2) voi moi i,k € {1,2,3,4,5,6,7,8} vai Zk. Do mét mit tuong ting v6i C? c&p dinh nén: nC} = > s(i,3) = O3.s(1,2) 4 3n = 14s(1,2) 1S Suy ra n chia hét cho 14. Ti dé: n > 14. C&ch t6 mau 14 niit thod man bai todn sau: {1,2,3,4}{1,2,5, 6}{1,2,7, 8}{2,3,5,8}{2,3,7,6}{3,4,7, 8}{3, 4,5, 6}{1, 4,8, 5}{1, 4, 7,6}5,6 Nhfn xét. 70 i | } i { | | | 6 i f ~ Voi mot hinh lép phuong c6 thé ghi lai m@i dinh mot s6 chon trong tap {1, 2, 3, 4,5, 6, 7, 8}, hai dinh khée nhau ghi hai s6 khéc nhau. ~ MBi canh hinh lép phucng c6 thé tuong ‘ing v6i ding 3 canh song song véi n6. ~ Méi dutng chéo cita mat hinh lap phuong c6 thé tuong ting voi ding 3 dutng chéo cia mat cing nim trong mat chifa n6 hoc trong mat déi dién véi nd. ~ M@i dudng chéo (chinh) cia hinh lp phuong 6 thé tuong ting voi ding 3 duing chéo (chinh) cdn lai. V6i ¥ tudng trén c6 thé hiéu ly do tai sao lai chi ra ditge céch té mau nhu trén, Bai toan 2 (USAMO 2001, Bai 1) Cé 8 cdi hop, méi hop chita 6 tréi bank. Tum 36 mhé nhét sao cho mdi bank tu) y déu duge to mot trong n mau thod man déng thoi hai diéu kign sau: 1. Trong méi hép, khong 6 hai bank nao duge t6 cing mot mau. 2, Hai hop bét ky) cb chung khdg qué mot mau. Giai. +) Goi a; 1a sé mau xuft hign i lin. i= 1,2,...,4, (k 1 ta 06: ae 1a it GOP = G ape 1 L&y (1) tri z (2) réi cong véi 30) ta duge: po da + Eo aR ee Tir do: 7 +) Céich 16 sau cia 23 mu théa bai toén (goi tén mau la: 1,2,...,23) HopI 13 4 5 6 7 HopIl 1 8 9 10 11 12 Hoplll 1 13 14 15 16 17 HopIV 2 3 8 13 18 19 HopV 2 4 9 14 20 21 Hop VI 2 5 10 15 22 23 Hop VII 6 11 16 18 20 22 Hop VII 7 12 17 19 21 23 Nhan xét. +) O hinh duéi, méi duémg tuong trung cho méi hop, cée giao diém 6 trén dudng tugng trung cho céc banh. +) C6 dang 8 dudng, méi dutng chita ding 6 giao diém va cé tat cd 23 giao diém. Hai dudng bat ky c6 téi da mot diém chung, +) MBi céch dénh s6 23 giao diém, ti 1 dén 23, cho ta mot céch t6 mau trén cdc banh 6 8 hop théa céc digu kign bai toa, Bai toén 3 (IMO 2005, Bai 6) Trong mot hy thi hoc sinh gidi, cic tht sinh phai gidi » - a 2. A 6 bai tovn. Biét ring wii hai bat todn bt hj luon od niu hon = 6 tht sinh dy thi, gid duge od hai bai todn nay. Ngodi ra khong oé tht sinh ndo gidi duge od 6 bai todn. Ching minh rang od it nhét 2 thé sinh sao cho méi ngudi trong ho gidi duge diing 5 bai todn. Giai. +) Goi n la s6 thi sinh tham gia ky thi vA ts IA 86 thf sinh gidi dugo dling k bai toam (k= 0,1,2,3,4,5,6). Ta o6: t= 0; N= ao +4 +22 +49 +44 + os. (4) Ta cd chting minh: 25 > 2 72 Speech ne rr pe vai aataalcH i; +) Voi i,j vai Aj, goi 9(i,j) = s(7,2) 1a 96 thf sinh gidi duge c& bai i va bai 5,(i,5 = 1,2,3,4,5, 6) ‘Theo gia thiét lun c6: 5s(i,j) > 2n. Do d6: 5s(i,j) > 2n +1. Co tht c& CF = 15 cap (i,j) mA i < j nén: 5s, 5) > 15(2n + 1). roa Do d6: 5S => s(i,7) > 32n +1) (5) tg +) Ta cing 6: S = Ct, + Chas + Chg + Obs = xp + 323 + 6x4 + 1025 (6) Te (4), (5), (6): y+ 3x3 + 64 + 10z5 > 6(Z9 + 2 +22 +03 +24 +05) +3 hay 4s > 6 + 62; + 5x2 +3z3+3>3 a) Ti do: B21 +) Ta ching minh thém zs khong thé bing 1. Gid stk 2s = 1. Lite do tit (7) cho 29 = 21 = Tit (6) cho: = =0,vaty=n-1. S=6n+4. 8) ‘Trudng hop nay c6 duy nh4t mét thi sinh A lAm duge ding 5 bai, cdn lai tt c& déu lam duge diing 4 bai. +) Goi bai duy nhét ma thf sinh A khong lam duge la bai r va & 18 86 thi sinh gidi duge bai r. M@i thi sinh trong k thf sinh nay ngoai’vige gidi dugc bai r con gidi duoc dang 3 bai nifa trong s6 cdc bai tosn cn lai nén: 6 ak= D7 str) 0 selder +) Dat 2n +1 = 5a. Ta 6: 9(%,j) > a. Néu a khéng phai 18 s6 nguyén thi: (i,j) >@ + 58(i,j) > 2n+2. Suy ra: DY 5s(i, 3) > 15(2n +2) & S>6n4+6. aaa 73. ‘Trai voi (8) +) Néu a la s6 nguyen thi higu s(¢,7) —a I s6 nguyén khong am. Ti $ = 6n-+4 viét lai Dicg(s(t, 4) — a) = 1, suy ra trong 15 96 hang 9(i, j) voi i < j, phai c6 14 s6 hang c6 cing gié tri IA a va ding mot s6 hang 06 gis tri la. a+ 1. Goi s(p,q) = a+ 1. Do d6 gis tri cia D9. jar (74) chi 06 thé 1A 5a hole 5a +1 tity theo r Khong thudc ho&e thudc {p,q}. Két hop véi (*), ta c6 hote 5a chia hét cho 3 hoke 5a +1 chia hét cho 3 (*). +) vi thi sinh A gidi duge 5 bai, nén tdn tai mot bai t khée véi cée bai p, q,r ma tht sinh A gidi duge. Goi h 18 96 thi sinh gidi duge bai t. Trong s6 h thi sinh nay, thi thf sinh A gidi duge bai ¢ va them ding 4 bai nia, vA A—1 thi sinh cdn lai ciing gidi due bai ¢ va thém ding 3 bai nia. Vi vay: ° . 44+3h—-1)= 5 s(t,3) hay D> s(¢,9) = 3h +1 sage jana Dot ¢ {p,q} nen: 6 DY s(,3) = 5a. anit Suy ra 5a = 3h +1 va Sa-+1 = 3h +2. Diéu nay mau thuiin véi (**). Qua ldi gidi bai todn trén, t6i cho ring cée bai toén sau ciing c6 cing § tuéng gidi va cc em hoc sinh c6 thé thyc tap tudng minh léi gidi. Bai todn 4 Chon la sé nguyén lon hon hodc bing 3. Trén méi canh va méi dung chéo cia n-gidc déu AyAz....Aq ngudi ta mudn ghi mot sb nguyén duong nhé hon hod bing P, sao cho cée diéu kién sau déu duge théa man: i) Moi 36 nguyén ta 1 dén p déu duge ghi. i) Voi méi tam gidc A,AjAx tiy yf déu 05 hai canh duge ghi hai s6 gidng nhau va s6 nay lén hon sé dugc ghi trén canh con lai. Hay xée dink s6 nguyen p lin nhat dé cb thé ghi céc sé théa man céc diéu kién dat ra. Voi gid tri nay cia p, hdi 06 bao nhieu céch ghi thda man diéu kién bai todn, Bai toan 5 C6 m_+2 cdi hop, méi hop chtta m trdi banh. Tim s6n nhé nhét sao cho méi bank duge t6 mét trong n mau théa man déng thoi hai diéu kién sau: 1. Trong méi hép, khong c6 hai bank nao duge 16 cing mot mau 2. Hai hép bat ki cé chung khéng qué mét mau, Bai toan 6 Trong mét hy thi hoc sink gidi, cdc thé sinh phdi gidi 6 bai todn. Biét ring 74 t 2 2 16; hai bai todn bat by) ludn 6 nhidu hon = 36 tht sinh du thi gid duc cf hat bat todn nay. Ngodi ra khong c6 thi nao gidi duge cd 6 bai todn. @. Goi k la 36 tht sink gidi duge ding 5 bai todn. Tum gid inj nhd mhét ctia k. , ly b. Chitng minh tén tai ba bai todn ma cé nhiéu hon 5% thé sinh dy thi gidi duge. . 1 © Chong mink tn ti bin bai todn ma 05 nhiéu hon = 36 tht sinh du thi gidi duge. Bai toén 7 Cho mét n-gide Idi c6 din tich S. Ching minh ring tén tai mot cank AB ctia da gidc va mot diém M thudc mién da gide nay sao cho khodng cach tix diém M 4s én dudng thing AB khong nhé hon =. Tai ligu tham khao [1] Toan hoc va Tudi tré s6 340/2005; 344/206: Bai cia Thay Vi Dinh Hoa (DHSP Ha Noi). [2] Toan hoc va Tudi tré s6 341/2005: Bai ciia Thy Hoing Ngoc Canh (THPT chuyen Ha Tinh). (3] USA and International Mathematical Olympiads 2001 Titu Andreescu and Zuming Feng. {4] http://www.mathlinks IMO Shotlist 2005. [5] IMO 2006 Solutions. | | Gidi thiéu mot sé bai toan dai sé cé xuat xt tiv hinh hoc Nguyén Dang Phat Bai viét ndy gidi thiéu voi ban doc mat s6 bai todn dai sé dac thi, cd cudt wit ti hinh hoc, ma noi dung bao gdm hai thé loai la hé phwong trinh dai 36 va cuc tri dai 36. 1 Mé6t sé hé phuong trinh dai sé dic biét co xuat xit tit hinh hoc. Trude hét, hay xuSt phat tir mot s6 hé phuong trinh dai s6 bic hai don gidn (06 xubt xif tit hink hoc). Bai toan 1. Gii va bién luan he phuong trinh sau: (a #0, b #40, c#0); Q) Lai gidi so luge 1 A Dat 7 1A gid tri chung cia ba ty s6 trong hé phuong trinh (1) réi tinh ¢ (ma ta thutng goi la dn phu) vac, y, z. Ta duge: a=t(2?—yz), b= ty? — 22), c= (2? — zy), Ti d6 suy ra (néu t < 00): abe 2(2° + y° + 29 — 3zyz); (2) va hai hé thie nita tuong tu (46i voi ¥ - ca va c? — ab), thu duge nh’ hoén vj vong quanha+b—+esavarsysz-. Cuéi cing ta di dén két qué sau day: ‘Tra 18. 1°) Néu ba s6 a,b,c khée khong di cho va khée nhau d6i mot thi 2,y,z cing vay 16 (c #0, y #0, 2402, y, z doi mot khée nhau). Khi 6, ¢ cing khée khong va hé phuang trink (1) la v6 dink), 06 biéu thite nghiém nhw sau: z y Feet Fog HA 6) 2) Néu a = b =c thi he phuong trinh (1) cing 18 vd dinh, vi khi d6 x = y = z va ly gid tr] thy J, ké cA 0. Nhan xét: 1°) Biéu thitc nghiém ciia hé (1) 6 tink ch&t d6i xing, dutge suy ra tit (1) bing céch thay cdc hing s6 a,b,c lin lugt béi cée dn x,y, 2, va nguoc lai, thay x,y,z bai a, b,c tuong tmg. Ta néi ring he (1) c6 tinh chit déi hop. 2°) Tit 16i gidi ciia he (1) ta dé dng suy ra hai he phuong trinh sau (hé ba phuong trinh ba dn) c6 nghiém duy nhét: a oy ec! qa’) ntytz=atb+o (abc#0) e p—2x @ 6 (”) xyz = abe; (abe ¢ 0) Bai todn 2. Gidi he phuong trinh (An 18 z,y, 2): z-a_y-b_z-c¢ @ —bo d=e = (t-a) + (y—B)? + (2-0)? @) =a Déng thdi, ching minh ring: Néua =b = =0 va gidta a’, U’, ¢ 06 he thiée A(a? +8? + 6?) + (pa! + qb'+ rc) +B =0, (i) (trong 46 A 40, BAO; a fa, VAL, ¢ Ac; PAO, g £0, r #0) thi git x,y,z cing c6 hé thifc sau day: B(x? +4? + 2”) + Ape + qy+rz)+A=0, (ii) Huéng dan. Dat ¢ 1a gid trj chung cia ba ti s6 4 v6 tréi cia (*), cing c6 nghia dat t= (e— a) + (y—6)? + (z—c)? la dn phy. Latu 9 ring dat dn phy nhu thé ndy cing 1a phuong phép chung dé gidi he phuong trinh dai s6 c6 dang (*) nhu he phwong trinh trén day. Sau khi thuc hién mét s6 phép bién déi dai s6 tuong duong ta thu duge dép 86 cia bai ton, Phan nay danh cho ban doc tu kiém nghiém. ‘Tra 18%. Bidu thitc nghigm cia he c6 thé duge viét (sip dat) dudi dang sau day: ad-a_Y-b c¢-c ta y-b z-e = (a =a $Y bP + (6-0; c+) 7 i i han xét. Biéu thite (**) cia hé (*) c6 tinh chat d6i xing cing giéng nhu biéu thite nghiém (3) cia he (1) duge chi ra trong bai toan 1 é trén. Cu thé la (**) duge suy ra tit (*) bling céch gid nguyén cfc hing s6 dit kién (44 cho) a,b,c nhung lai thay céc hing s6 a’,¥,¢ lan Iugt béi céc An x,y, 2 vA ngude lai, thay +, y, z lan lugt béi a’, Wc. Xuét xf cia bai ton 2. Ti bai toan dai s6 1 trén day, nhan nhan xét vé tinh chét, "déi lap" cha biéu thitc gitta nghiém ("An") va hing s6 dit kién, tac gid bai viét nay lién tung dén tinh chét "46i hop" cia phép bién hink nghich dao trong mat phiing cing nhu trong khong gian. Trude hét, viét phyong trinh vécto ciia phép nghich déo trong khOng gian cyte O va phutdng tfch p =r? = 1) rdi da xudt thanh bai todn 2 trén day. OM.OM' =1 (OM 770M) & OM = 2 hay IA: Sau 6, viét phuong trinh vécto cia mat ciu (hay duéng tron trong m&t phing) AZ? 420.7 +B =0ri xuat bai todn chtmg minh tiép theo (Phép nghich dio bién mat cu thanh mat cdu hay mat ph&ng). Bai ton 3. Giai va bign luan hé phuong trinh (An z, y, z): (a — x) cosatan? a + (y — b) cos 8 + (z —c) cosy (x — a) cosa + (b— y) cos f tan? B + (z —c) cosy (x — a) cosa + (y — 8) cos 8 + (c — z) cosy tan?y = 0. cn) (cosa #0, cos £0, cosy £0; ee) . 1 : 2 =, tan?g= 1, ton? a Hung din. Thay tanta = > ~1, tan? = a5, tanty = a 1 vio he phuong trinh (***) thi dua duge he nay v8 dang sau day: an fe oo (x a) cosa + (y ~ 6) cos + (2 - ¢) cosy = as = cr) ‘Tra lWi. 1°) Néu o,f vay khong c6 méi lién hé gi véi nhau, cy thé Ia: cos? «+ cos? f+ cos? # 1 thi hé phuong trinh 6 nghigm duy nhét z= a, y=b, z=c. 2°) Néu cos® a + cos” 9 + cos? y = 1, hé phuong trinh c6 v6 s6 nghiém, dang: r=a+ Acoso, y=b+AcosB, (trong d6 1A mot tham sé tiy ¥, 4 € R) z=ct eos, Xuét of ctia bai todn 3. Bai toén trén day duge phét hign dong thdi véi bai toén 4 (duge trinh bay ngay sau day), ndy sinh tis vige dat bai todn 4. Bai toan nay cé nhién 8 dé han nhiéu so véi Bai todn 4. Nhan xét: Ching ta c6 thé dé dang mé rong bai todn cho trudng hgp nhiéu dn: n an 21, 2,-.+4q, dOng thoi cho n gOc a1, 02)... an. Bai todn 4 cing vay. Bai toan 4. Gidi va bien luan he phuong trink (dn 1a «, y, 2) ane _b-y_c-2__ ple? ty? +22) + g(zcosa +ycosf + 20084)? cosa cos cosy (p+ g)(acosa+ y cos B + 20087) Q (a) trong d6 p #0, ¢ #0, P+a40, 00); (i) M Yetta aye Pare (ii) théa man he thite (iii) sau day: Ely? + 2? — 23)2? = OP + 2 — a?)a?; (iii) Chi thich: Khong cn phai gidi he phuong trinh 2) Dio lei, néu 2, y, 2 1A ede nghiém duong cita he phutong trinh (2) (gdm 2 phuong trinh (ii) va mot phuong trinh (iii): Ha = {(it), (#i)} thi cing théa man he thie (i) ‘Tom Iai Fa{(@), (ii) > Ha{(éi), (ei)} 3°) Ching minh ring eéc bé 1; (j = 1,2) ¢6 nghigm (duong) kkhi va chi khi céc s6 duong a,b,c da cho théa man cée bat ding thiic sau day PKAPre);, P 0): om P+y+ 24 Pad Pe2¢e (a>0,b>0,c>0,d>0); (*) Pega ye R= 2+ PoP 4a; (+#) Xuét xf cia bai ton 8. Bai toén 8 trén day c6 xudt xt tir bai toan hinh hoc khéng gian sau day: Bai ton 8’. Cho tit dign gin déu ABCD (ma ti dign d&u Ia trutmg hop d&e biet), nghia I8 cée canh d6i dign bing nhau: BC = DA,CA = DB, AB = DC. Hay xéc dinh céc khoang céch tit mot diém P da cho dén céc dinh cia ttt dien biét ring diém Q, déi ximg véi P qua tam O cia mat cdu C(ABCD) ngoai tiép tt dign di cho c6 khoang céch dén céc dinh A,B,C,D cia ttt dign theo tht ty bang QA =4,QB =b,QC =¢,QD =d. Huéng dn. Ap dung dinh ly vé dutng trung tuyén trong tam gid vA tinh ch&t cia trong tam ctia mot hé diém. Tuy nhién cting cé thé sit dung phuong phép vecta, Dap 86. Gidi he H{(*), (*)}, ta duoc két qua: 1 1 HOFAtP a), Poet e+a—W), 2 1 FOrPP-A, Pater e yea) (***) Bai todn luén c6 nghiém (duong) duy nhét, xéc dink bdi céc hé thite (***). Muén vay, hay ching minh tinh chat sau day, d&c-trung cho mét tit dién gan déu (bao gm c& ti dién déu), tuong ty véi dinh ly Pompiu trong hinh hoc phéng, déc trumg cho mot tam giée déu. Dinh ly Pompiu (trong hinh hoc khong gian) c6 noi dung nhv sau: Binh phutong céc khoang céch tit mot diém P bat ky trong khéng gian dén cdc dinh cia mot tt dién gin déu ABCD da cho luén biéu thi dign tich céc mat cia mot tit dién (7) nao d6. (Digu 6 c6 nghia cy thé nhw sau: Binh phutong khoang céch tit P dén mot dinh ndo d6 cia tit dign gan déu ABCD di cho khong 1én hon tng binh phuong cée Khodng céch tix P dén ba dinh can Iai). 82 Tit dién (7) nay suy bién thanh mot tt diém phAng (tite 14 c6 thé tich v(T) = 0) khi va chi khi diém P tring véi diém xuyén tam déi cia mét dinh nado dé cia ttt dién gan déu da cho ABCD trén mat cu C(ABCD) ngoai tiép tut dien 6. Hu6ng dan chiing minh dinh lf Pompiu trong hinh hoc khong gian. Sit dung vecto va tinh chét cite tam ti cy cia mot hé diém, chimg minh cée bat ding thie hinh hoe sau: PA? < PB? + PC? + PD?, va ba BD.T tuong tt Q) PB? < PC? + PD? + PA?, 2) PC? < PD? + PA? + PB, (3) PD? < PA? + PB? + PC?. (4) Dfu ding thite xdy ra khi va. chi khi: P € {4’, B,C’, D’}, trong d6 A’ = (E9(A), B’ = €o(B),C’ = E(C) va D’ = o(D), O A tam ciu C(ABCD), tiy theo P tuong tng thudc vao bit dang thitc (1), (2), (3) hay (4) Cha thich bé sung. Trong muc 1.2 trén day, tac gid bai viét nay dA gidi thigu them 4 bai todn méi cing vé gidi nhing hé phuong trinh bac hai c6 3 hode 4 an, trong dé ¢6 chi 16 xuit xit hinh hoc d6i véi ba bai todn 5, 7, va 8. Chinh viée giai céc bai todn nay, céc ban da cho 101 gidi dai 36 cita céc bai tofn hinh hoc 6’, 7’ va 8’, 1a xuét xit hinh hoc clia céc bai todn dai s6 5, 7 va 8. Ngoai ra, téc gid cée dé ton nay ciing dé nghi ban doc tim tdi them 1di gidi thudn tity hinh hoc ctia céc bai todn hinh hoc xuAt xtt 5', 7” va 8, g6p phn lam phong phé va da dang cho Idi gidi céc bai ton 46. D6i véi bai toén 6, tée gid cia né cing xudt phét tir mot bai toan hinh hoc ma dé xudt, nhung lai khong gidi thieu trong bai viét. Tée gid c6 dung ¥ dé lai, danh cho ban doc phan dodn, suy xét vé ngudn g6c rudt aif n&o tit hinh hoc cia bai todn dai s6 (bai 6) may. Tuy nbién, xin luu ¥ ban doc 1& vige phén dodn vé zudt att hinh hoc cia bai toén 6 thyc ra khéng cé gi Ad Khan lm. Cc ban chi cin quan tam dén ¥ nghia hinh hoc cita céc dang thie (ii) va (ii) cing nhu ¥ nghia hinh hoc cia hai bd s6 dutong (a, 6, c) va (a,y,2). Tu 46 ban dé dang chi ra déi tugng hinh hoc khéng gian nao dn 6 ding sau 6 con sé, trong d6 a,b,c la da cho con 2, y, z 18 cdc dn s6 duong cén tim. Sau cing, xin gidi thigu hai bai todn vé hé phwong trink dai sé bac hai phitc tap han va chtta nhiéu an (5 hoc 6 an). Bai todn 9*. Gidi vd bign luan he phuong trink bac hai ( séu dn x:,ye,i = 1,2,3 a nhitng s6 duong): wy [2tusmtwantn=d (*) apt yt + kaa = 23+ yf + koays = 23 +98 + hesys = C7; (#4) trong 6 c va dla nhiing s6 duong va k € R; c,d, k cho trudc, a) Lai giai thw nt: Lai gidi gém hai bude sau day: 83 Buée 1: Néu hé phuong trink H{(*), (**)} 6 nghiem thi 2; = x2 = 23 (= 2) va w= t= Ys (= 9). Tit (*) thay yy = d— zs, ye =d— 2, va ys = d— 2p vio (**), sau khi thuc hign cde phép tfnh ri rat gon ta dugc hé ba phuong trinh sau d6i véi ba dn 7,22 va x3: (ker, ~ 2x9)d — (03 + kagx1) +X =0 (i) va hai phuong trinh tuong tu bing céch hodn vi vong quanh céc chi sé: 1 + 2+ 3 > 1, trong dé ta di dat X=? -2 +3 a3. luge d bing ba biéu thitc khéc nhau nhu sau day, trong dé biéu thitc sau duge suy tir biéu thite diing truéc n6 nh’ hodn vi néi tren ai tkegrs-X _ 23+kogm—X _ af-keye2—X a = Btsana# kag — 2g ‘hay — 23 kag — 2a, (ii) (v6i quy uéc néu méu s6 triét tiéu thi tir s6 cing triét tieu). Sau d6 ép dung tinh chét cita day céc ty s6 bing nhau dé khit dai luong X, ta thu duge ba ding thite sau: d=t=t=ts, trong dé = (3 = t2)(a9 + 22 ~ kar) (ii) 2(ea — 21) + k(a2 — 24) va t nhan duge tix t; cling nhw ts nh&n duge tit ty bai hodn vi vong quanh cée chi s6 1+ 2 3 ~ 1. (Lé duong nbién cing d8 nhan ra ring dén lugt ¢; thi nhén lai tit ty cing béi hodn vi vong quanh céc chi s6 nhu da néi 6 trén). Dén day tit cdc c&p ding thc t; = t; (i #7 € {1,2,3}) ta déu thu duge sy triét tiéu cia mot biéu thie c6 dang mot tam thite bac hai déi v6i k sau day: S(k) ) Ching han, tit ts = t, thi a, va 7 lin lugt déu c6 dang 18 nhiing da thiic bic 3 thudn nhét va d6i xing déi voi cde d6i s6 1, 22,7 theo nghia hodn vi vong quan cée chi s6 1, 2,3 nhu da néi 6 trén. Cu thé 1a: t ak? + Sk +7=0 a = ~ay0? ~ 230} — 2,23 + 3xy2025, ) B= 2x +a} + 2} — Bx2909) +7, (vi) = (eh + 2} +23 — eae? ~ zor} — 2529) + 209 + 2} + 2§ — 2y0? — x2} ~ 2402) (vii) Sau vai phép bién déi dun gién nifa, cfc hé s6 a, va trong phuong trinh (iv) cbn duge biéu thi duéi dang mot s6 biéu thite khdc nia, thuan tién hon cho viée xét diu cha chiing. That vay, ta 06 cdc biéu thie sau: (2 — t)(a3 — 21) 22 + (wa — ar9)?r4, (v’) 21 +92 +25) [(2~ 29)? + (xy — 23)? + (a1 —22)"] +7, 7) = (22~ 23) [208 2) + (23 —29)] + (2s ~ 22) (22 - af - 2) (vr) 84 thife ntfa cia cing nhu hai biéu thite nifa cite y nhd hoén vi vong quanh 1 > 2-3-1 nhu da n6i 6 trén. Ty dé suy ra, ngodi (vi’) thi 8 cling con cé hai biéu thite khée nita. D6i véi vige xét dau cia a, 8 va 7, khong mt tinh téng quét ching ta c6 thé gid dinh quy wdc ring: 0 < 21 < 2 < zy (hokc 0 < a < a3 < 1, howe 0 < 23 $m < 22). Ching han, v6i gid dinh 0 <2; < 22 < as ta thdy ngay ring a <0 va 72 0 cdn 6? > 0. Tit do suy ra biet s6 cia (iv) khong am: A = 6? — dary > 0. Ngoai ra, véi céc biéu thiic (v’) va (vii?) cita a vay, ta nhén ra ngay ring: a= 0 & =m = 03 Y=0 == 25, Vado dé: =0 & y= suy ra: , kéo theo @ = 0. Mat khac, dé thay: Tr A = 6? + (—4ay) = le = 0 va, 4-06 eau Béi vay, ta di dén két luan: A=0 4 2, = 22 = 23 (= 2) va do d6, kéo theo yi = yn = ys (= 9). Nhung A = 0 lai la diéu kign cin va dit dé k 1 nghiem duy nhét cia he phuong trinh HA («), (#*)} duge xét (néu hé nay c6 nghiém). ‘Tém Iai 1a; lap Iudn trén day chimg'té ring: Néw he 1 séu phuong trink {(+), (##)} 66 nghiém thi céc dn x, 22,23 bling nhau (va dat bing z), déng thai céc 4n 11, ye, va ciing bing nhau (va dat bing y). D6 la dpem. Va do dé, bai toan quy vé viée gidi mot +hé hai phuong trinh bac hai d6i v6i hai dn x va y sau day: Bude 2: Gidi va bién ludn hé hai phuong trinh (én x vay déu duong) rt+y=d, (Q) Pty? + kay = 2. @) ‘Tit (1) va (2) ta duige (2-Kay=P-2 (3) Ti d6 d8 dang suy ra: - Néu k= 2 vic # d thi hé phuong trinh {(1), (2)} v6 nghiém. - Néu k = 2 va c= thi hé phuong trinh c6 vo s6 nghiém, mién sao théa man (1). Cy thé 1a. I= d, { - (o 0va(2—k)(@ —c*) > 0. (©) ee <2, e2, ord. ® Cdn cdc nghiem duong x,y cia he {(1), (2)} 1a hai nghiem cia (5), xéc dinh béi: ford = (6%) K=F (ds (SON), Gay b) Nhan xét. Sau khi thyc hign hai bude gidi nhu da trinh bay & trén, vige gidi va bign luan hé phuong trinh bac hai véi 6 an dung 21,22, 73; y1, Ya, ys dén day da hoan t&t. Day I& mot he phuong tinh dai s6 d&c bigt, gdm ba phutong trinh bac nhat (*) va, ba phuong trink bac hai (**) ding thdi dai hdi tit c& cée An zi,y¢ (¢ = 1,2,3) déu la nhiing s6 thyc duong. Bay gid ching ta hay quan tam dén qué trinh bign ludn vé vige gidi hé phitong trinh {(1), (2)}. Trutéc hét hay dé ¥ dén gid tri dc bigt k = 2 lam cho hé phudng trinh ho&e vo nghi¢m, hogc c6 vo s6 nghi¢m. Néu k = 2 va c = d thi (2) duge ‘viét lai 1a (46i chiéu véi (1)): a +y? + zy = 2 = d? hay lac? + y? — 22ycosm = 2 (= d*) (20) Khi d6, hé thie (10) biéu thi dinh ly ham sé cosin d6i véi mot tam gide "suy bién" cb hai canh véi do dai ©,y cdn canh lén nbit c bing téng hai canh c6 do dai x va y (véi c= d thi tit (10) suy ra: s+ = c (= d). Sau nila, trong hai bit dng thiic (7) va (8) biéu thi digu kign c6 hai nghiém duong (cé nhién 1 duy ohAt) cia phwong trinh bac hai (8), ta hay dé § dae bigt dén bét ding thiic (7). Dé la bat ding thite diéu kin quy dinh méi lién he gitia hai s6 thuc duong c,d va s6 thuc k dé (5) c6 nghiém duong, cling tite Ja dé bai todn cé 1di gidi. Déi véi bat ding thtic kép (7) ta die biét chi ¥ dén bit ding thite k <2 (v6i e < d). Khi dé ta 06 thé dat k = —2cos-y (véi 0 < 7 < 1), 7 dutge hoan toan xéc dinh: +y = arccos =£ va do dé, (2) duge viét lai duéi dang: a +y? — 2zycosy=C; (c BE = CD, d.p.c.m) Bay gid ta tré lai ching minh diéu khang dinh tren day bing phuong phap phdén ching. That vay, gié sit trén cung AyB da cho ta tim duge ba diém Cy, C2, Cy doi mot phan biét théa man (*), ciing tite IA théa mén day dang thiic (1). Khong mat tinh tng quét, ta c6 thé gid dinh ring: a, < az < a3 vA do dé theo (1) thi dua dén: by < by < by; (2) Sau dé, ta dug mot g6c Cy = + ri ly cée diém Ap, As, Az, Ay tren Cz va Bo, Bi, Ba, By trén Cy sao cho CA = CBy = $; CA; = by vi. OB, = ay (i € {1,2,3}). Thé thi ta duge: AA;BiC = AABC, (c.g.0); do dé A\By = A,B) = AyBy = AB =. (3) Va tit cde bat ding thitc (2) suy ra AA;B,C nim gon trong c& hei tam giée AyB,C va AsBsC. Ta sé chitng té ring: AB, < AzBy ho&c A\B, < AsBy. ‘That vay, tit (1) ta duge bo — bs = a2 — a1 VA by — by = a5 — ap. Do d6 ta c6 AaAg = B,Bp; (Hinh 2) (4) AAs = BpBa (6) Trén hinh 2 ta duge hai tit gidc 181 ApByBy Ay va A3B3B2A, tuong img théa man (4) va (5), trong d6 hai c&p tia (A243, B,B,) va AgAi, ByB2) ddu ct nhau 6 C. Béi vay, theo bé dé 6 trén ta duge AaB, > AsBr, 6 AgBs > ArBo. @” Xét hai trutmg hop cé thé xay ra: 1°) Néu a; < by (Hinh 2) thi CA,B, < CBiA, suy ra AASB, < CAB, < 90° < AAs va do d6, trong AA; B,Ag ta. c6 AyBy > AyB, nén déi chiéu véi (6) ta. duce AiBy < AyBy (8) 88 2) Néu a; > b; thi cling lap lun tuong ty (ban doc ty vé hinh tuong ting) ta duge AVBaB, < 90° < AyByBo va do d6, trong AA;B; Be ta ob AyBp > AxBy nén d6i chiéu vi b&t ding thite (7) ta ditge: A:By < AsBs (9) Céc bat ding thie (8) hoe (9) thu duge tit gid dinh c6 cée bat ding thite nghiém ng&t (2), mau thudn véi gid thiét cia bai toén 1a c6 day ding thie (3). Mau thuan 46 ching t6 ho&c a; = a2 = as va do d6, by = b = bs, [nghia lA cdc diém C; (i = 1,2,3) tring nhau 4 mot diém Cy nao 46 trén cung tron A7B), hode it nhAt hai trong be s6 a; bing nhau (hay hai trong ba s6 b; bing nhau, i € {1,2,3}). Bay gid ta chiing minh ring: Néu hai trong ba s6 a; (i € {1,2,3}) bing nhau thi c& ba s6 a; d6 bing nhau, do dé cd ba s6 6; cing bing nhau. That vay, ching han gid sit a = ag, thé thi tir (1) suy ra by = by va bay gid thi day ding thuc (1) thu vé chi con ‘mot, dé 1a: a + by = a2 + by. (10) Bai todn quy vé chi cdn hai tam gidc, trong d6 diéu kien (*) duge thay béi (10). Sau 46 cing ching minh tuong tu nhu é phan trén, sit dung bé d8 4 néu thi chimg minh duge rang: a, = a2 = a3 va do dé b, = bz = bs, ta duge d.p.c.m. Bude 3. Rét cudc bai ton quy vé bai todn dying hinh don gidn sau day: Bai toan 9c. Tim trén cung trin AyB mot diém C sao cho: BC +CA AB =c, c vid la nhitng do dai cho trudc. Bign luan: Kéo dai tia BC vé phia C réi dung diém D sao cho CD = CA va do d6, BD = BC+CD = BC+CA=d. (Cé ding thitc nay 1a do ta gid sit ring C trén cung AyB a diém da tim duge, théa man diéu kign dat ra). Thé thi ACAD can 6 C c6 géc 4 day GDA (= BDA = 3). Te d6 suy ra D 1a mét trong hai giao diém Dy, Dz cia didng tron (B, d) tm B bén kinh d va cung tron ASB chita gée 6 = $ [c6 tam 1a trung diém Cy cia cung A7B da cho] dymg trén doan AB va nim cng hia véi AyB. 4, trong dé - Tir d6 suy ra cach dung diém C: Diém C la giao diém cia tia BD va cung A7B, trong 46 D € {D,, D2} = (B,d) 9 ABB. Dé théy ring BC+ CA = d. (Hinh 3) - Bign lun: Bai todn 6 lki gii (nghiém hinh) khi va chi khi: Fi aay 7 CoD (ua) AB=c 0) rt+yed S tt+y=d a? +y? —2nycosy ay = SF > 0(ie< d) Vay x va y 1 hai nghiém duong X;, X» clita phuong trinh bac hai: e-2 oe = aX + Tay 0 (12) Digu kign cn va di dé phuong trinh (12) c6 nghigm (nghiém duong) 1a: Biet s6 e- Psin?Z 2a cos? Z A 20 # cddsindwac e>dsind (13) DE ¥ ring A 6 thé duge viét lei nhu sau: _ 22 = (1 cosy)? 4c? fe 1+ cosy (14) ‘Ti d6 ta duge két qué (sau khi dé thay —2cos-y = k, véi |k| < 2) a 1 42 — (24+ ye {eu} = (%,%} = 5 (de (SEES (05) Két qué (15) nay tim duge phi hop véi két qua (9) tim ra 6 bude 2, tiéu muc a). e) Nhan xét va lai binh (vé hai Idi gidi ciia bai todn 9). Hai bai ton hinh hoc Ya) va 9c) chfnh 1a noi dung hinh hoc tuong ting véi hai bai ton dai s6 ma Idi gidi dug dé cp dén trong hai bude gidi (bude 1 va bude 2) lien tiép cita bai todn 9 vé gidi va bien ludn mot hé phuong trinh dai s6 bac hai. Ching Id hai phan cu thanh bai ton hinh hoc 9b — bin phién dich sang ngon ngit hinh hoc eta bai toan 9 (mét bai toén hon toan dai s6 vé hé phuong trinh) ma chiing ta cé thé cho né mot tén goi 1A "Mo hink hinh hoc" cia bai toin 9, mot bai ton vé gidi va bién luan mot he phuong trinh bac hai. No cho ta mét hinh anh true quan trong hinh hoc vat ly cia mét he phuong trinh dai s6 bac hai. Tuy nhién, méi 1di gidi cia bai tosn 9 mae dik khéc nhau vé phucng phép tiép cn nhung déu thé hién siic théi dac thi, riéng biét va. rat &n tuong cia tig Idi gidi. Néu nhu 1di gidi 1 (10% gidi dai 36) 06 doi hoi bj nang bién déi va tinh toén tinh té vi cdn phai thyc hién nhiéu phép todn nhung lai cho due chiing minh true tiép diéu khang dinh quan trong phét biéu trong bitéc 1 cia 1d gidi 90 bai todn 9 [va diéu dé cing c6 nghia I chi ra duge céch chiing minh true tiép tinh chat hinh hoc phat biéu trong ndi dung cia bai todn hinh hoc 9a)} thi loi gidi 2 (Joi gidi hinh, hoc) cita bai todn 9 lai khong doi héi tink ton nhung ddi hdi thong minh séng tao, dung thém dutng phu, hinh phy va chi cdn huy dong vén kién thtte ft di thuéc chuong trinh hinh hoc 8 cing di ding cho viée chtig minh tinh chat hinh hoc néi trén tuy 1di gidi nay lai khong thé cho duge céch chiing minh truc tiép ma sit dung phuong phép phén chiing_mot phutong phép ching minh gidn tiép_dé khng dinh tinh ch&t hinh hoc nay. ‘Trén day 1a phéc hoa déi nét vé tdng thé khi déi chiéu, so sfnh hai 1di gidi cita bai todn 9 v8 phwong phdp tiép cdn cing nh phwong phdp gidi quyét udn dé cia ting li gidi dé ma bai viét nay néu ra nhim muc dich trao déi véi ban doc nhing suy ngam, nhiing ¥ tudng dé xu&t va kinh nghiém xung quanh viéc gidi va khai théc m6t bai todn toan hoc hay (vé dai s6 cing nhu hinh hoc) nao 46. Néu xem xét ky lung, chi tiét hon thi truée hét, phai néi rang: Bai toén hinh hoc 9b) chi 1 mé hinh hinh hoc cia mét phdn bai todn dai sé 9 ma thoi. That vay, tré lai phan bién ludn cia Idi giai thit nhAt cita bai todn 9 (trinh bay 4 bude 2) ching ta nhan va ngay rng Idi gidi thit hai (Idi gidi hinh hoc) cita bai todn nay chi dé cap dén nhing gié tri cit tham s6 k théa man bat ding thite |k| < 2 sao cho ¢6 thé gan cho k yj nghia Rink hoc k = —2cos7, trong d6 0 <7 <7 18 d6 lén cia mot géc cia mot tam giée ndo d6 ma canh déi dign voi gée dé c6 do dai c va hai canh ciia géc y dé c6 do dai z vay ma tong cia hai canh dé ¢+y = d, déng thdi d > c. Nhiing gié tri cia kd day ddi héi |k| < 2 chi 1d mot tap con nhiing gié tri k théa min bat ding thitc (7) chit chuta n6i téi nhitng gié tri cia k théa man bat ding thie (8) duge chi ra day dit trong phan bien luan thudc bude 2 cita tiu muc a) trinh bay Jdi gidi tht nhit cila bai todn 9. Tée gid bai viét nay con dé trong, chua thdy duge ¥ nghia hinh hoc cia nhing gid tri k khéc, ngoai |k| < 2. f) XuAt >i cia cdc bai todn 9, 9a), 9b) va mGt hung dé xudt bai todn mdi Khac. Trong ky thi Olympic Ton quéc té lan thit 46 (46* IMO, 7/2005) té chtte 6 Mexico 6 hai bai toan hinh hoc phng, trong dé c6 bai todn (Bai 1) sau day: Trén céc canh ciia mét tam giée déu ABC sdu diém da duge chon lan lugt: Ay, A> trén BC; By, Bz trén CA; Ci,C2 trén AB. Céc diém nay la cdc dink cia mot luc gidc Wi A:AzB,B2CiCz 06 tét ed cde canh bling nhau. Chting mink ring céc dudng thing A,Ba, ByC2 va CyAz ddng quy. Chinh tix bai todn hinh hoc may cia ky thi IMO, 7/2005 vira qua ma téc gid da dé xuét bai toén hin hoc 9a), 9b) va bai todn dai s6 9 4 tren. Dé dugc thuan tien cho viée phét biéu cdc bai ton ndy duéi dang nhwr da trinh bay trong bai viét, téc gid di dat lai ‘ten nhu hinh 4 dé ghi: Tam giée déu xuét phét duige ky higu lai 18 CiC2Cs, cdn luc gide Ii duge xét ky higu I A,B, A,B, AsBs véi céc cap dinh Ba, Ag; Bs, Ar va By, A2 lin higt trén C2Cs,CaC; va CrC2. Ba duttng thing duge xét sé la: Ay Bo, AzBy va AsB1. Thé th, theo gid thiét cia bai toan va véi ky higu nhu trén hinh 4 ching ta thiét lap dugc ngay day ding thc (*) néu trong gid thiét cia céc bai ton 9, 9b) va 9a). Tuy mhién, rieng 91 v8 g6c, g6c 60° 6 cée dinh Cj (cita A déu CiC2C; va cing 1a cia ba AA:BiC)) da duge thay béi g6e 6 do lén 7 téng quét hon, mién AO <7 <7. Bai toan 10**. Gidi vd bien ludn he phuong trink bac hai 5 dn x,y, z,u,v sau day zu-yu_ aw (Ho) trong dé a,b,c,d,e, f la cdc 6 thue khéc khong da cho. A. Léi gidi "so cp" cita bai todn. 1°) Nghiém tdm thudng ciia bai todn. a) Hé phuong trinh 1, (*) 06 thé viét Iai duéi dang sau day: Néu dat \ 1a gi tri chung (ma ta thudng goi 18 dn phu cia hé) clia day 5 ti s6 bing nhau (*) thi ta c6 6 dang thite sau: ©) b) Nhan théy ring véi z, y, z,u,v tity J, bao gid ta cing c6 dong nhét thite sau day (d8 dang kiém nghiém): (az —y)(zv —u?) + (gu ~ y2)(zu— yo) + (cv ~ yu)(yu— 22) = 0, (Wz,y,2,u,0); (*) Déi chiéu (I) va (*) ta dude he thite sau day (néu he (*) da cho c6 nghi M(af + be + cd) =0 ‘Ti d6 suy ra: 1° Hole A = 0, va do dé, he (*) c6 dang don gidn (I) sau: {az—y? ru yrs av yu=yu~2=2u-yo=z-w=0); (r) 2 Hoke » # 0 thi he (*) cling cdn nghigm khéc nifa khi cdc hing s6 da cho ring bude véi nhau béi diéu kign sau: af +be+cd = 0, (*) Vay ta cdn xét hai trutmg hop: Tru@ng hgp 1. Néu céc s6 thc khée khong a,b,c, d,e, f da cho Ia tay §, khong adi héi phai théa man digu kien (*) thi he phuong trinh (*) duce xét chi c6 nghiém tém 1BAl tofn 10** nay dé duge dua vio cubn séch ciia GS. Nguyn Vin Mau "Da thite dai s6 vA phin thie hau t7", NXB Gio due, 2002 (Bai toda 8, trang 48) 92 thudng, (ng v6i gid tri A = 0 trong (I). Nghiem d6 18 nghiém cia he phuong trinh (P) nh da chi ra 6 trén. Tit he (I) nay ta dé dang thiét lap duoc day diing thitc sau (1) ‘Tit (1) suy ra he (I’) nay o6 nghiem sau day: eryrziuivealiti? set, (véit tay y, tER); (2) Néu dé f ring 1 = 7? thi 2,y,z,u,v theo thi tu ty 1é v6i lay thita cia #, tir 0, 1,2,3 dén 4, Diu dé goi ¥ ring néu ta thay déi ky hiéu céc dn 56 x,y, z,u,v lan lugt bai %o)Z1, 22, 13, v4 thi biéu thttc nghiém cite he (I) duge viét gon lai dudi dang: a = pt’ (0 #0 bat ky, i € {0,1,2,3,4}) hay dac biét, 06 thé cho p = 1 thi: a =t (i € {0,1,2,3,4}); (3) ‘Trudng hyp 2. He H, duge bé sung them did kien (*) dé trd thanh he (H) = {(+), (+#)}. Tit day khong nhing chi thay déi ky higu d6i voi céc dn s6 nh trén ma cing thay ddi ky hiéu d6i y6i cdc hing s6 dé cho; cu thé 1A a,b,¢,d,e, f lan lugt duge thay bdi bo, bi, ba, bs, be, bs. Vide thay déi nay cling con c6 tac dung gitip chiing ta theo doi vige gidi he phuong trinh phan nao duge dé dang hon. Véi quy uée 46, ta viét lai he phuong trinh (1) = {(+),(##)} = (Me) U(#e) sau khi duge bd sung them didu kien (*): (% sect sagan = mais = BN _ aeons aed (_ \ 49) (*) bobs + biby + baby = 0 (+*) Con déng nhét thtic (*") 6 trén thi bay gid duge viét lai nhu sau: (Coa ~ a)(zamr4 — 23) + (oxy ~ 2129) (wary ~ 232) + (0x4 — 2129) (2425 - 23); (*”) 2) Tim nghigm khong tim thudng cita bai ton, ¢) Ngoai déng nhét thic (*”) lien quan dén céc dn s6 2; (i € {0,1,2,3, 4}) da duge phét hien trén, bing nhing quan sét dic biét céc nhj thttc thudn nhét cia céc An c6 cling bac (déng bac) va c6 cing téng céc chi s6, ta cn phét hien thém 5 déng nhét thitc nifa sau day (ma ban doc ciing dé dang kiém nghiem dugc): (war ~ 13)ot2 + (xoe3 — ay24)29 + (m2 ~ 23)x4 = 0, @ ~2(aare — 28)x1 — (wars ~ 2124) 02 + (core — 205 + 22)e9 —(aota — 2122)24 =0, (ii) (x24 ~ 23) 20 — (wots — 22%4)21 — 2(aor4 — © Z3)Z2 + (ots ~ 2122):t9 + (toma — 23)rq = 0, (ii) (2x03 — m24)z0 + (tots — 2m123 + 29)ay + (Zo ~ 2y2)2 — 2(z9z2 - 27)z3 = 0, (iv) (2129 — 23)a0 — (sors — 2122)21 + (or, — 2?) = 0. () 93 Bay gid ta thay cée nhj thifc 3 trong ngo&e béi nhj thite tuong tng 06 gi tri Ab, (= 0,1,2,3,4,5}) rit tir day ti s6 bling ohau (*) 6 trén, sau d6 khit dn phy ta thu duge mot he méi 1, gém 5 phuong trinh tuyén tinh thudn nhAt vdi 5 dn s6 @o, £1, L203, 24 ma dinh thite 4 cia he trigt tieu (vi A duge phan tich thinh tich hai thita s6, trong dé ‘mot thita $6 1a bobs + bibs + babs (= 0); thita s6 cdn lai, ky hiéu O(b,) # 0. beta + bets + bray = 0 (i 2b, - byte + (be —bs)ts - bite = 0 (ii), (11) { bs:t0 byt, - bytr + bits + bore = 0 (iii); bato + (bo—ba)tr + bray —2boxs 0 (iv), bsxo —bz, + dor, 0 (v)i Vi dinh thie A cia he (H:) o6 dang: A = (bobs + bibs + bubs) .6(b,) = 0 ma (0) £0 nén (741) 06 nghigm khong tam thudng duy nhét, (xéc dinh sai khéc mot thita s6 #0). Gidi he (74:) ta thu duge nghiém sau day: Auto = bf ~ bo(ba + bs), bobs + bibs, 2 — bobs, (4) = (bibs + babs), Aize = bj ~ bs(ba + bs). {é gidi he phuong trinh tuyén t{nh thudn nbét 1, nay (c6 dinh thie A = 0) ta cht vige 6 di mot trong 5 phuong trinh ciia he d6, ching han bé phuong trinh tht ba (iii); sau 6 chi vide gidi hé {74,\(iii),} gém 4 phuong trinh cdn lai véi 5 én Lo, Li, 22, 23,24 thi thu duge nghiém duy nhét c6 biéu thie (4)]. ‘Tom Iai la, sau hai bude gidi ta thu duge két qui He phuong trinh bac hai (1) {(),(#«)} v6i 5 dn zo,a1, 22,19, 24 06 nghiém tam thutng (3) vA nghiém kh6ng tam thudng (4) nhu da chi ra 6 tren, B. Co sé ly thuyét toan hoc din dén hé phuong trinh tuyén tinh thudn nhAt (74) dé tim nghigm khong tam thuong cia hé (71) néu trong bai todn 3. Ching ta cin thm hiéu ngon nginh dé ly gidi con dudug din dén Idi gii "so ef" ciia bai toén 10** nhu da néu ra trong myc A 6 trén. 1°) Trude hét, dé y rang cée hiéu sau day: ToRa— Tj, Bots ~ m2, Boe, Tyzg, tyTy~ 23, very — aia, yt — 22 Ia cée nhi thite bac hai thudn nhét eta céc dn x; (i € {0, 1,2, 3, 4}), xufit hién trong day (*) cée ty s6 bang nhau cia hé (H.), duge tao thanh tix ede dinh thitc con (minor) ofp 2 rét tit ma tran chit nhat (2x 4) sau: zo @% 22 25 jz) Zo 25 24 4 C6 thé xem chéing la céc toa dd Pliicker cli mot dudng thing xa anh trong khong gian xa dnb 3 chiéu P; xéc dinh béi hai diém 06 cfc toa d6: diém A(zo, 71, 22, ra) va diém B(x, 2,23,24) trong toa dé xa dnh thudn nbét. 2) Clic ¥ tudng co bin trong qué trinh tim t0i di gidi (nghiém) ctia hé phutong trinh (H) 6 tren. a) Nhu d phan A da chi ra: Hé phuong trinh 71, (*) va do dé, hé phuong trinh (1) 06 nghiém tm thutng ma biéu thitc nghiém c6 dang (3) a= pt, 4 {0,1,2,3,4}, p #0. b) Hé phuong trinh 11, (*) v6i cde dn 2; duoc viét Iai nhwt sau: ry bo = tote — 2}, phy = tomy — 2122, pho = zox4 — 2125, pbs = 103 — 03, py = t9%3— 2174, pbs = xatq — 23; trong 6 p la gid tri chung cita 5 ti s6 bling nhau ofa hé phuong trinh (*) ma gié tri p= 0 cho ta nghi¢m tam thudng (3) cia hé (71) va chua doi héi dén diéu kign (**) cia céc hing s6 cho trudc b; (i € {0, 1,2, 3,4, 5}). Ciing nhu nhan xét b) phin A da chi ra: Voi ao, 21, 22,29, 24 tuy ¥ bao gid ta cing 6 déng nhat thite (*’): Way (i = 0,1, 2,---74) : (woe — 29) (wae — 23) + (toa — 2109)(973 — 2424)+ + (Gots — 2123)(m123 — 23) =0; (*) Chink déng nhAt thie (*’) nay cing v6i ding thite diéu kign (**) vé céc hling 36 by (i= 0,1,-758) la co sé quyét dinh cho vige tim nghigm khéng tém thuing cia he (2). c) Ta chimg minh didu khing dinh sau: ‘Néu 06 he thitc (**) nghia 1a cdc hing s6 b; (i = 0,1,...,5) rang bude béi digu kién bobs + bibs + babs = 0 thi, ngodi nghi¢m tam thutng (3) (x; = ¢ (i € {0,1,2,3,4})], he phwong trinh bac hai thudn nhét (71) = {(4), (++)} con o6 mot nghiém khong tam thudng duy nhét ma (4) 1& biéu thie nghigm da chi ra 4 phan A. ‘That vay, hé thitc (**) duge viét lai duéi dang tuong duong sau: 2p\bobs + bibs + babs) = 0 (voi p #0), hay la bo(pbs) + bs (abo) + bx (abs) + bs(0bs) + ba(pbs) + Ba (pbs) = 0. @) Thay céc gié tri ctia pb; (i = 0,1,-.-,8) tit (1") vao (*") ta dude he thie sau day (sau khi da nhan hai vé cia (*”) véi 2): 2lbo(zaxa — 23) + bo(ore ~ af) + bi(z2xq — 2124) + by zor — 2122)+ + bo(mizs — 23) + ba(tors — 2129) = 0. (I") 95 V6i céc by (¢ = 0,T,..., 5) 1a da cho thi (I") chinh la phuong trinh trong toa do xa anh thudn nhét cia mét si¢u mat bac hai, suy bién think mot siéu nén trong khong gian xa. anh 4 chiéu P, Bay gid ta viét lai phuong trinh (I") dudi deng ma tran 27Qax. 0: 4 Dawes = 0, (gy = ait) ©) ijno trong dé cfc hé sé qi; c6 cdc gid tri cu thé nhu sau: Gos = bs, 13 = be — bs, Gs = bf, (6) G33 = —2bp, dua = 0, ‘Vay duéi dang ma tran thi phyong trinh siéu quadric (5) c6.dang: 0 0b & b\ fae 0-2; —by b—bs | fr (Zo 1 2 25 t4)] bs by -2% b bo | |x] =0, (6) by b2—bs by = 2b 0 ts bs bh by) Ny hay viét (6) mot cach nggin gon hon, duéi dang toén tit: 2™Qz =0, Ve #0 = (0,0,0,0,0) (7) trong 46: x7 = (a9, 21,2, 5,74) IA véct hang (chuyén vj clia véct cot (x9, -..,24), va Q = Q(b) 18 mot ma tran vuéng cp 5, c6 cée phan ti gy tao thanh ttt cdc hing s6 5; d& cho (théa man (**)) theo bang (5') 6 trén), Sau khi thye hign phép tinh dinh thiic |Q| cha ma trén Q(b) trong 46 } = (bo; bi, bays, bay bs) thi duge: |Q(b)| = (bobs + bibs + babs).-O(bi) = 0 (vi theo (**) thi bobs ++ bibs + babs = 0), nghia 18 Q(b) 1& mot ma tran suy bién va c6 hang r = 4. Boi vay, (6) 1a phuong trinh cia mot siéu mat baé hai suy bién thanh mot siéu ndn bac hai 6 O-phing_ dink, tite phing-dinh nay 1a mot diém, dugc xéc dinh boi hé phutong trinh tuyén tinh thudn nhét sau: Dui dang toén til, he phuong trinh nly duge viét gon lai Ia: Qzr=0, (8) Vi ma tran vudng cép 5 Q cé hang r bang 4 nén trong 5 phuong trinh tuyén tinh thuin nhét cia hé (8) chi c6 4 phuong trinh la doc lap tuyén tinh. Boi vay, hé phuong trinh tuyén tinh thudn nhét (8) xéc dinh cho ta mét diém duy nhdt trong khong gian xa anh. bén chiéu Pj. Diém ndy chinh 1a dink ctia sieu nén bac hai cb phutong trinh (6) hot (7) trong khong gian xa anh P, nay, 96 That vay, vi (*’) la mot déng nhét thi, két hop voi ding thie digu kien (**) thi (*”) va do d6, (6) hay (7) cing la mt ding nhat thite. Va tir (7) suy ra (8) la mot phuong trinh toan tit. Néu viét cu thé ra thi (8) cho ta hé 5 phuong trinh tuyén tinh thudn nh&t (Ha) nbu da viét 16 6 tiéu myc 2°)c) cia phn A 6 trén, Sau cling gidi hé phuong trinh tuyén tinh (8) cing tic 1a gidi he (74,) ta duge nghiém Radng tdm thudng (4) cia hé (7) nbit dé chi ra trong phan A, tiéu muc 2°)c) & trén. Digu khing dinh trén day da duge chting minh. 4d) Véi Ibi gidi trén day cia bai toén dai s6 10 trinh bay 6 tiéu muc c) ta théy ring vige gidi bai toén nay 6 lién quan dén mot ly thuyét hinh hoc vé méit bac hai trong khong gian 2a dnh, trong d6 6 mat bac hai suy bién thank nén bac hai. V8 mot ¥ nghia ndo 46 ma néi thi cing c6 thé goi 1di gidi nay 1 14 gidi hinh hoc ctia bai todn 10**. Dibu dé dn néi Ién ring bai todn 10** chic hin cn nhiing J3i gidi khdc nita. Hy vong bai toan nay cdn nhén duge Idi gidi hay khde 6 ban doc. C. Nguén géc lich sit hay xudt xtf cia bai toan 10**. BAi todn 10** xudt hign céich day dé dugc 42 nam. Num 1965 trong mot bai béo “Sur un espace riemannien & absolus locaux" ding & Tap chi Acta Scient. Viet. (Sectio Math et Phys) Tome II, p. 5-42, nhan mot vin dé nghién citu dau tién vé mot khong gian c6 tuyét déi dong, gido su Nguyén Canh To&n da dé xudt va gidi bai toan dai sb ma bai viét nay di dua vio danh muc céc bai todn can khéo citu v8 "Mot sd hé phuong trinh dai s6 dac bigt", bai ton d6 dich thyc 1A bai todn 10. Xudt phét tir mot vin dé nghién cau vé hinh hoc, tuy cong cu sit dung Jai 1a dai s6, téc gid bai bao vin gidu tri tudng tugng khong gian di gidi bai ton bing phuong phap "edt", "chiéu',... Va téc gid bai béo da cho dép sé nhu chiing ta da biét vé hai cOng thitc nghiém (3) va (4) tren day; tuy nhien ngudi mé dudng "khém pha" d& cho lai gidi khé dai (do tinh ton nhiéu va c6 phan phite tap). Nam 1970, 5 nd sau d6 téc gid bai viét nay da cho Idi gidi nhut da trinh bay 6 muc B. Khodng 20 nm sau nifa, gido su Nguyén Vin Mau da quan tam dén bai todn 10 nay va da gidi né bing phuong phép "hinh hoe hoa" (vi cé sit dung khai nigm va tinh chét cia tich vo huéng). Téc gid bai viét nay hy vong giéo sut Nguyén Van ‘Mau khoi phuc lai di gidi dé dé bé sung mot li gidi dep va hay nifa cho bai toén 10** nay, D. Bé sung mét bai todn tudng ty: Bai toan dao (ngudc) cia bai toan 10**. Bai toan 10’. Gidi he 6 phwong trink bac hai thudn nhét (dn yo,4n,¥2, Ys; Yas Ys) sau: Yous + vids + yas = 0, Plea = yi — yo(ye + us) par = you + Ys, Bian = 43 ~ vous Has = —(1nys + ys) ag = 98 — us(v2 + va); trong dé a; ld mhiing 36 thuc khdc khong dé cho (i= 0,1,....4). (Ha) 97 1°) Chitng minh ring hé phueng trink (Ha) nay c6 mot nghiém duy nhét, tring véi nghifm cia hé phuong trink tuyén tinh thudn nhét (H,) sau day. 9 0 0 %& a a \ (% , 0 =a a3 a3 -a -2a,| |% a Au= | a os 2a. 0 a a | |) = 11; 04) —2a3 a a -a ow 0 ] 7 a = 0 a 0 o } \% 2) Gidi hé 6 phuang trink tuyén tinh thudn nhdt (H,) nay ta thu duge biéu thite nghiém ctia hé (H2) nhu sau: (Ti hé (H;) suy raz PYo = doa2 — a3, Pa = A903 ~ 0102, Pia = A904 ~ a0, Ps = aya3 — a3, Ps = 0203 — a404, Ps = aa04 ~ a8 2 Mét sé bai todn cuc tri dai sé, gidi tich va ludng gidc c6 xuat xi tiv hinh hoc ‘Truée hét, xin gidi thigu mot bai ton cy éri gidi tich c6 xudt x tit hinh hoe Baj toén 11. Xét ham ba bién f = f(z, 22,23) cho bdi biéu thie: f= Ve ~ a1)? + (2a = a2) + (a = 03)? + Var — br) + (Wa — be)? + (Oy ays @) trong dé céc bién 2,2, 25 rang bude vdi nhau bdi diéu kien (céc déng thitc) sau: no Tae o 77 ces + YER (a) con ai, by ct (i = 1,2,3) la nhimg s6 thue cho tritde sao cho céic b6 ba 36 sip thit 4 (21,2, 43), (brs bass), (C1, €2,¢3) doi mot phan bigt (ciing 06 nghfa 1a trong ba bb dé Khng c5 hai 69 nao tring nhau) vd u? + uz + uj 40. Hay tim cue tiéu fm ctia ham f. Bién luan. a) Nhan xét. Day JA mot bai todin cue tri gidi tich (c6 rang bude), cu thé IA tim eve tiga ota ham (*) ba bién thuc 21,22, 29 rang bude véi nhau boi hai ding thite (1). [Tuy nhién, cing cé thé phat biéu khéc di dé duge xem IA mét bai ton v8 gidi mot hé phuong trink dai s6 (bac nhét, ba én 1, 22,23) nhumg gin voi mot diéu kién khéc nifa, cu thé "a bidu thuic (*) phai dat gid tri nh nhét fy (thay cho vige bot di mot phuong trinh 98 Chinh vi vay, chang ta c6 thé nghi ngay tdi viée sit dung phuong phép gidi tich vu thé 1a dinh ly cia gidi tich toan vé diéu kien can cia cuc tri cia ham mot bién 86. That vay, dat gid tri chung ctia ba ti 96 trong céc ding thiic (1) bing t, rdi thay = wt-+c (00 0, khong déi) voi moi diém M tren (A) MA+MB=Vw+di+ v4 (4) Nhung ta lei cé (bat ding thtic Mincopski): Vert y+ > Suresh +d)? (5) Dau dang thie dat duge khi va chi khi: (6) ‘Tit d6 suy ra: uv > 0, nghia la M € [4’B']. Vay ta duge: MA+MB= ++ frre > fee b) = VP + Add; (AB = d) Trad 10), Téng MA+ MB dat gié tri nbd abt fq = VE F Adds d diém M thude doan thing [A’B’] (18 doan hinh chiéu cia doan thing AB trén A) va chia trong doan dé theo ti 6 86 hoc A’M : MB! = dy : da, cing tite Ia: (4B) M) ©) ‘Tren day 18 10: gidi dai sé (vi phutong phép gidi ddi héi sit dung bét ding tht dai sb, cu thé 1a bit diing thite Mincopski) cia bai toda cvfe tri hinh hoe 11° duge phien dich Sang ngon ngit hinh hoc tit bai ton géc (bai todn 11) vé cute tri gidi tich. Bay gid ta lai phai phién dich trd lai két qua cia bai todn 11° sang bai toan géc (bai toan 11). ~ Dé xée dinh eée toa d9 cita diém M theo (6), ta clin xée dinh céc toa 46 cite A,B’ va céc khodng céch dj, dy AA L A= AY, suy ra A’ duge xée dinh bai AA’.ZA' = 0, trong a6 AA (oy — a1 + ust, 02 — ap + Uat, cs — a3 + ust); GA ut, uot, ust); (v6i W da duce chon sao cho WZ? = 1 tite W IA véc to dan vi chi phuong cia 4), nghia la vécto @ da dude chon sao cho W(uy, tig, us) ma. (@)=1; wt 4 =[eP 100 Ti d6 ta. duge gid tri cia tham sé t = t; ting voi A’; tuong tu t = ty tmg voi B’, t= WOA (4 =06A'=C); = WCB. Chuyén qua biéu thitc toa d6, ta duge: ty = un(ar — ¢1) + Ua(aa — 02) +s (a3 ~ 05) = DL, wi(ai — ci) (7) ta = ua(bi — c1) + mabe ~ cx) + ua(ba ~ ca) = Dh walbs — e) Tit (6) ta duge (1—k)OM = CA’ — KCB’ Thay k = —4 thi duge ayy a diet oot CM = Set aby, dtd" ® ‘Vay tham s6 t = ts ting véi diém M phai tim duge xée dinh béi dytz + dat) ty = GREG Oath ® Ciing tit d6 ta suy ra diém M phai tim trén A duce xée dinh bai OM = OC + OM = OC + ts. (20) Tit ding thiie vécto (10) ta duge eée toa d6 cia diém M can tim B= +uta, (i= 1,2,3). (11) Cée khodng céch dy = AA’ vad = BB duce xéc dinh tir @ = AA? = CA? — CA? = CA? — (CA)? = WEA? — (EA)? = (Zt ATA)? Nhu vay 4 [wa cal d= [Bacel, (=n; (12) trong d6 cde tich vécto @ ACA va WA GB cé céc toa. dO nhut sau: #Aai(| pee Us uy * Jag cy a1 — ea uy up aye a — om] a a us on Mu w ba — o| (14) ‘Thay gid tri cia dy, dz tit (14) vio (9) thh dude ts, sau dé lai tiép tuc thay gid tri cia ts ‘vio (11) thi xéc dinh duge hoan toan céc toa d6 21,22, 73 cha "diém cuc tiéu" M trén A phai tim. Con gid tri cuc tiéu fm cia f duge xe dinh béi: FR = (a = by)? + (a2 = be)? + (a5 — bs)? + Addo (15) trong d6 dy va dz duge xé¢ dinh béi céc he thite (14). hii thick: Phan 1Qi gidi dai s6 cia bai todn cue tri hinh hoc 11’ chua da dong dén bien Jugn cia bai toén dung hinh nay méc dau da chi ra dép s6 ding ciia bai todn vé gié tri cut tiéu fn cia dai lugng f, kem theo vi trf cia "diém cue tiéu" M phai tim trén dung thing A, cy thé la trén doan A’B’, hinh chiéu cia doan thing AB trén A. Chinh vi vay, khi ta phién dich tré lai két qué thu duge trong bai tod cue tri hinh hoc 11’ song bai toan g6c (cute tri gidi tfch 11) cing chua théy xuét hien phin bién luan Ia buée cubi cing ciia vige gidi bai todn g6e (bai ton 11 vé cuc tri gidi tich). Boi vay, dé ho&n chinh Idi gidi bai to4n dung hinh (bao gém ca bai toén eye tri hinh hoc) ching ta cdn bé sung phan bién fudn ma 1é ra phan nay phai xem JA phan cuéi cing cita 1di gidi. Tuy nhién, dé nhfn manh va lam néi bat digu ndy ching ta tach khéi l0i gidi thanh tiéu mue rigng. c) Bign lian: ‘Ted lai, theo doi lai gidi (dai s6) cia bai ton cue tri hinh hoc 1’ ta thy ring ti s6 don (A’B’, M) 06 biéu thitc (6) dude hoan to&n xéc dinh khi vA chi khi & +d} #0. Béi vay, bai todn 1’ c6 nghi¢m duy nhdt khi va chi khi d? +d? # 0 tute 18 khi va chi khi it nh&t mot trong hai diém (phan bigt) A va B khong ném trén A, cling tite 1& thi va chi khi hose ba diém A,B,C khOng thing hang ho&c khi C' tring A, hoe C tring B ahung A ¥ (AB). Bai tosn c6 v6 56 nghiém (va bat cit diém M ndo nim trong doan AB (ké ca hai dau mit A va B) cing la diém phai tim] khi va chi khi cd hai diém A va B déu nim tren dudng thing A. Nhu vay: f dat min f,=AB @ {M}=[AB)CA (f=MA+MB, Med) ‘Tren co sé két qué phan bién luan nay cia bai tosn 11° ta phién dich ngugc lai sang ngOn ngit ciia bai toan géc 11, ta cb két qua sau day. N6i chung, bai todn 11 ¢6 nghiém duy nhdt va biéu thite nghiém 2; (i = 1,2,3) 6 dang (11), trong d6 ty c6 biéu thite (9) va ti (i = 1,2) ¢6 dang (7) con d; duge xAc dink bai (12). Gia tri cue tiéu fn cita f duge xéc dinh béi (15). Bai ton 11 c6 v6 sé nghiém khi va chi khi @y — C1 12 — Cy: Gy — Cy = by ~ Cyt be — Cpt by — Cg = Uy : Uy 2 Ug Gié tri cuc tiéu fn cia f duge xéc dinh béi (15), trong dé d = dy = 0. d) Loi binh bé xung. Véi nhan xét néu ra é myc 2.1 a) trén day bai toén 11 vé cutc tri gidi tich da. duge gidi nhd hinh hoc héa noi dung bai todn cuc tri géc. Bai ton 11 da duge phien dich sang ngOn ngit hink hoc than bai ton cue tri trong hinh hoc khong gian (bai ton 11’). Ching ta da sit dung mot B. D. T dai s6 co ban (d6 l& B. D. T. Mincopski) dé gii bai ton cue tri hinh hocl!'nay. Sau cling chiing ta lai phai sit dung dng thai c& kién thtte vé dai s6 (phuong phap toa dd) va hinh hoc (cy thé la tich v6 hudng va tich véc to cia hai vée to trongkhéng gian) dé biéu thj két qué tim duge (gié tri eye tidu fr, cha 102 ham f) hoan to&n theo céc dit kign (ce s6 thie da cho a;, bj, ci, w(t = 1,2,3) trong noi dung bai toén), Cu thé lA khong nhimng chi tinh gid tri cuc tidu fp cia ham f ma con phai xéc dinh gié tri cu thé cia céc bién 23, 2,73 theo cdc s6 d8 cho khi f dat eye tiéu fry. Vi vay, 101 gidi khong sit dung phuong php gidi tich (cling cdn duce goi 1 pong phép ham s6) nay thuc ra cing khongduge gon ging va tién Igi cho lim . Dé ¥ quan sat ky vé lai gidi 1 (10% gidi hinh hoc) nay chting ta thy viee sit dung B. D. T. Mincopski c6 phan da duge goi ¥ tix noi dung biéu thitc (*) nim ngay trong noi dung bai todn 11 réi, nhat 18 khi (*) 44 duge bién ddi vé dang (2) cia f = f(t) phu thudctham s6 t. Digu 46 khién ching ta cin quay tré lai thém mét lan nifa tim cach sit dung B. D. T. Mincopski dé gidi bai todn cyc tri gidi tich may hodn to&n bing phuong phép dai s6 d) Léi giai 2( 13i gidi dai s6) cia bai ton 11. ‘Trude hét dé thyc hién céc phép ton duge gon gang ta luén ludn gid thiét duge ring Beeeareea uy tug tug = va dua (1) vé dang (phwtong trinh tham s6): =e tut, (§= 1,2,8), @ trong dé uj tha man didu kign (1") va (—00 (ei — bi)? = #7 + ast +8 (i) = Tier (cr — a4), F a 1 { Thea wl — 80), 8 = Toa (4-8 (= 1,2,3) ®) Bay gid 4p dung bat ding thitc Cauchy ta duge Of = [ua (c1 ~ an) + ua(co — a) + us(ea ~ as))? < (u+ud+u8)[(er ~ a1)*+(c2'— a2)?+(69 — a3) ri dé ¥ dén (1") va (i’) ta dugca? < +2. Tuong tu ta cing dugca? < 73. ‘Tuy nhién, ap dung hing ding thic Lagrange thi ta c6 cic ding thitc sau: a= of + BF; (ii) B= 08 + BF; 103 trong d6 1 — a = BE = [ua(ea — as) ~ us(ee ~ a2))? + [us(cr — a) — us (cs — a)]? + [en (ce — 2) — ale 1 ~ 03 = 3 = [uals — bs) ~ ua(c2 — ba)” + [ua(cr — br) ~ wales ~ bs)]? + [un (ce ~ ba) — ua(cy (iv) Déi chiéu (ii) va (iii) ta duge: 2 Dole ai)? = (t+ an)? + 62, a 3 Deb)? = (t+ a2)? +63, a vi do d6 thu duge biéu thite gon cilia f nhu sau: fa yleran e+ VeranP om, a trong dé a4), 6 (i = 1,2) déu 1d ning hing s6 theo thit ty duge xée dinh béi (i’) va (iv) nhut da chi ra 8 tren. Dén day véi biéu thttc (*) dé duge thu gon ciia f chéing ta c6 thé thm dutgc gié tri cute tiéu (gid tri nhé nhét) f, cia him f bing phuong phép gidi tich (sit dung didu kign cfn dé ham f dat cyc tiéu thong qua vige tinh dao ham "(t) réi gidi phuong trinh J") = 0 dé chira gid tri t = to, 3.46 f dat cue tiéu f,, ma ta phai tim. Tuy nhien vige gidi phuong trinh /’(t) = 0 cing chua that sy don gidn nhu ching ta mong muén. Béi vay ching ta nghi dén phuong phép sit dung bit ding thite dai s6 dé tim gid tri nhé nhat fn cia biéu thite f. That vay, vi on, a2, 6: va 2 déu IA nhiing hing s6 dé cho, trong d6 fj vA By déu Ia nhiing s6 khéng am; béi vay ta nghi dén bat ding thre Mincopski. Theo b&t ding thie Mincopski, ta dug: fa ytbayP + Aet V(t on) +92 Yla—ax)? + (A+B) — (v) (Dé ¥ ring (t + a2)” + 6? Khi —t— a2)? + 8f). Diu ding thite & (v) dat duge Kkhi va chi (t+ax) __ (t+) a OB va do d6 dé dang suy ra khi va chi khi O12 + a6 fs Bath * (vi) 104 trong 46 ay = Do u(r — a4); aa = Daler — by) ‘I va 61,2 duge xdc dinh béi ding thite (iv). Cuéi cing ta thu duge két qua cho gié tri nhé nh&t f, cia f minf = fm = \/ (oa — a2)” + (61 + a)? (vii) Khi va chi khi ¢ = to, xc dinh béi (vi), com a4, 6; (i = 1,2) duge xéedinh béi ("va (iv) nhu da chi ra 6 trén. Bién luan. Bai toén cé nghiém khi gié tri cla ¢ (xdc dinh bai (vi) duge hodn to&n xéc dinh. Vi 9 > 0 (vi = 1,2) nen By + 62 > 0. Bai vay bai todn c6 nghiem duy nh&t khi va chi khi ? + 63 # 0, ciing tite 18 ki va chi khi st nh&t c6 mot nghiem f; (i = 1,2) £0 (va do d6 hoe fi, > 0, hoki 62 > 0). Bai ton c6 v6 s6 nghiém Khi va chi khi 6 = 2 = 0. Va do 6, theo céc he thitc (iv) xc dinh 6? va. 63 6 trén dé dang suy ra: By = Ba = 0.4% cy — 0! Cn — a2 : 65 — 03 = Cy — by 2 0g — ba: 03 — by = ty : U2 Us. (viii) ‘Tom lai: Bai ton 11 06 nghiém duy nh&t fn xéc dinh bai (vil) + 6? +6 4 0. Bai ton 11 c6 v6 86 nghiém cé6 cling gid tri fm =| a, — a2 |¢> 6, = Bo, cling tite la cdc bo sb 4, bis ci, (i= 1,2,3) thda man didu kign(Viii) Cu6i ching, 46i chiéu hai Idi gidi 1 va 2 (hinh hoc va dai s6) cia bai todn 11 ta théy duge § nghia hinh hoc cia a va fi; (i = 1,2) cia Idi gidi dai 96. D6 Wa:ty = ts, A = dy, By = —da va a; = —t; (i = 1,2). Sau day IA hai bai todn eve tri dai s6 va heong gide dau 6 xudt xt tit cing mt bai ton cyc tri inh hoc (trong hinh hoc phing). Hai bai todn cxfc tri d6, néi khée di chinh 1A 121 gidi dai 96 va ldi gidi long gide cia bai toan cue tri géc. Bai vay, truéc hét xin d@ cap bai todn eve tri hinh hoc géc, xuét xit cla hai bai tosn cue tri dai s6 va ligng gide (v8 noi dung cting nut Idi gidi) dé ban doc d& theo doi va d6i chiéu. Bai ton 12, (Bai toan cuc tri hinh hoc géc) ‘Trong mat phing cho dutng tron tam O bén kinh a va mot diém P c6 dinh nim trong dudng tron (OP = d < a). Tong cde tt giée 1bi ABCD noi tiép duong tron néi trén sao cho céc dutng chéo AC va BD vuong g6c véi nhau tai P, hay xéc dinh tit gide c6 chu vi Ién nh&t va tit gidc c6 chu vi nhé nh&t. Tinh cdc chu vi d6 theo a va d. Cha thich. Bai toén ndy chinh 1a mOt bai toan hinh hoc phing (Bai 1) trong ky thi chon hoe sinh gidi quée gia mon Toan (Bang A), théng 3 nim 1997. Bai toén nay c6 nhiéu céch giai. Sau day a lai gidi hinh hoc cia bai todn (Ldi gidi 1), 105 a) Ky higu p = AB + BC + CD + DA la chu vi cia tt giée ABCD, ta tinh p= (AB + CD) + (BC+ DA). Ta dude: (AB? + CD*) + (BC? + DA”) + 2(AB.CD + BC + DA)+ +2(AB.AD + CB.CD + BA.BC + DC.DA). Vi ABCD néi tiép dung tron (O, a) nén ta 06 (Hinh 1): AP.PC = BP.PD =-®P/(0,a) =a? ~@ = 8, (1) AB.CD + BC.DA = AC.BD (Dinh lf Ptolémé); (2) Lai vi ACLBD = P nén ta dé dang thiét lap dugc céc he thite sau: AB? + CD? = BC? + DA? = 4a?, (3) ABAD _CB.CD _BABO _ DADC _ 4, &) APE 2 2C bP PD) " AC? + BD? = 4(20? — d?) = 4(a? +8?) () Dén day, tit (1), (2), (3), (4), (5), v8 2AC.BD = (AC + BD)? — (AC? + BD?) ta thu duge bigu thite thu gon cia p? (vé phai cia bigu thitc (*) sau day): p= (AC + BD)? +4a(AC + BD) + 4(a? — 8°). (*) b) Bigu thite (*) cita p? cho ta biét : p? va do d6, p dat id tri lon nhat py hay gid tri nhé nit pr khi va chi khi tong AC + BD do dai hai dudng chéo cita tit giée ABCD, tuong tmg dat gié tri max hay min, Bay gid tit he thite (5) va he thite (6) san day lien quan dén AC va BD: (AC + BD)? + (AC — BD)? = 2(AC? + BD), (6) ta suy ra: AC + BD dat gid tr] max(min) khi va chi khi [AC — BD| dat min(max). +) Ty (6) suy ra: max(AC + BD)? = 8(a? + 8°) 4 AC = BD. Bai vay, ta duge: AC + BD dat gis tri max = 2/2? +B) AC=BD=VUP+. (7) Cui cling thé gid tri (7) vao (*) ta thu duge két qua cn tim cla py Pm = Aav2+ V+), (b? =a? — a) (8) # AC = BD = JX +8). 106 Vi AC va BD 1a hai day cung cia dudng tron (O,a) nén ching bing nhau khi va chi khi ABCD la m6t hinh thang cin 6 vj tri A1B,C,D, (hinh 2) nhan (OP) lam truc d6i ximg va ciing 1d trung true chung ciia hai day B,C; vi AiDi. +) Ciing tir (6) suy ra. min(AC + BD)? = 8(a? +b?) — 4(a — 6)? + |AC — BD| dat max = 2(a - 8) A(a +6)? va do d6, khi va chi khi mét trong hai duéng chéo (AC hote BD) la dutmg kinh di qua P ola (0,a), cdn dutmg chéo kia (BD hoke AC) la day cung ngfn nh&t di qua P. Ta di dén két luan p dat gié tri nhé nh&t Pye Pm = 4Vala+8), (9) khi va chi khi ABCD 4 vi tri cita tit gidc AyB,CzD2 nhan dutong chéo lén lam true déi xiing, tring véi dudng kinh di qua P cia (O,a); (ching han AyC2 3 P & hinh 3) Loi gidi 2. (Lai gidi Luong giéc) a) Ky higu do 1én céc géc 4 tam AOB, BOC, COD va DOA lan lugt 1a 2, 2y,22 vA 2t. Khi dé ACB = ADB = 2, BAC = BDO = y, CAD = CBD = z, DBA = DCA =1; (Minh 1) ddng thdi thod man eéc digu kien sau O 2/2sin(z Fy) cos( 4) (20) 108 Diu dang thite 6 (20) dat duge khi va chi khi: arty om © yes eetyarttas (21) @ sin(z + y) =sin(z +t) = 1. Béi vay, f(x,y) det gis tri nhd nhét Sm = f(t0,¥0)s Jn = 2VB.cos (! (22) trong 46 79,44, IA nghiém cia hé hai phuong trinh {(13), (21)}. Gidi ra ta duge c0s(7%9 ~ Yo) = sin 2yy = £ rdi dé dang tim duge: eVEETD = z+ 2) = ale (23) fete { 1+ @ a y Sau d6, tit (24) ta thu duge Pm = 2afrq va théy lai biéu thite (9) cita pm nbu di chi ra trong Idi gidi hinh hoc (Bi tosn 12), Sau day 18 di gidi dai s6 (Lai gidi 3) cite bai togn 12. a) Dat b= Va? —@; PA = 2; PO = 2; PB =y;PD sau day cia chu vi p cia tt gide ABCD: =P2,2, yy) = VP+et VP tats Jaret Vy ee (*) Ngoti ra, dé ding chi ra. duge cdc bién s6 2, 2’,y,y/ rang bude véi nhan béi cée diéu kien (bao gém cd bét ding thite va ding thiic) sau day a-d=a-V@-P<2,2'y,y daca! = 4b8, XP +X? = 4a, +45? va do 6, X? < 4a. Tit d6 ta duge bat ding thitc va dang thie sau: 2b = to < X; < 2ay < 20 Shr X? = 4a? + 2(n — 2)6? + 2nd? = 4a? + 4(n — 1)0? = X23 + (n — 1)22 +) Ta hay tinh pi theo Xi, X;,aij vi b. Ter (ii) ta suy ra /23 +03 + ya? +03 = let ay? + (aj +2), [23 +a? + oP +23 = feito)? + (a +a); va do d6, Big = Bis (in 21, 1,25) = yf (ae + 24)? + (aj + 24)? + f(s tay)? + (2h + ai), (1) i Sau khi binh phuong hai vé ciia (1) va st dung cée hé thite (ii) vA ky higu rdt gon a} +a) +2342} = 4a}, ta duce biéu thie rit gon sau ciia p2, dui dang mot tam thite bac hai d6i voi bién X; +X} Bly = (Kit Xj)? + day (Xi + X,) + (03, +8), (2) trong dé gitta X; va X; c6 cdc he thitc sau day: XP +X} = 403, + 407, (3) (Xi 4X4)? + (Xi — Xj)? = (a3, + 82). (4) +) Vi X; > 0, X; > 0 nen biéu thite (2) cha. Phy cho biét: p}, va do dé, py dat gid tri maxi hay gid tri minpy,, khi va chi khi X; +X; dat max hay min. Do d6, tit (4) ta suy ra: Xi +X; dat max(min) ¢ |X; — Xj] dat min(max). Tit (4) ta duge: (Xs + Xj)? < 8(02, +B) va do d6 thay vio (2) ta duge: : Hh my s2{ Sy (ayv8+ (ohm), (°) isigjen isigisn isigjen trong d60 % isicisn —1)a?+C2_,6? = const (10) = 1)maza?, + C2_,mina?, He thi (10) chiing td ring: - Khong thé xy ra a,j = b v6i Vi,j (Dibu nay dé dang suy ra bing phuong phép chimg minh phan ching). Ty dé suy ra ring phai c6 mot s6 gié tri a; = a. M&t khéc, ©6 tht c& C2 = (n- 1) + C2, gié tri cia aij, mA theo (10) thi t6i da c6 C2, gié tri aij = b va vi thé, phai c6 n — 1 gid tri aj; ndo d6 dat cuc dai bing a (ching han, yz = yg = ++ = ayn =, tite 1 ayy = a(j # 1). Cudi cing, suy ra: Pm = 4[ln— DN Valar8) + Fin 1m —2)0v3], (a) hay la minp = alm = nayi+® + Ct _bva]= A(n = 1) fala +8) + 2(n — 1)(n — 2)dv2. (12) Xpaatzh =2, (6€ {1,2,....n}), X41) = 08, Gi Voi n = 2, ta duge: minp = 4,/a(a +6), ta thy lai két qua da thu duge 6 phan (I) 113. ~ Bay gid tim gis tri lén nh&t pyr ctia p. Ta str dung bit ding thie — < y/—, (u; > 0) dé danh gid tiép bit diing thic 6 vé phai clia bat ding thttc kép (*") 06 sit dung dai luong bat bién (10). Thé thi, vi D2, ui < Vn Sou? nén ta duce: ®) Vicicjen ts S CHD a3), hay viét cu thé hon la: XS ass [ean =e + OE aa = [BORD istgjsn rat 2 -)(n- Be —2y] + Do dé ta duge E vs (C2) verre ar ee) 5 + IA R (XO a+ cx) iin 1hGien = O2(n - 1a? + C20? _,b2 + (C2)%B? = (A) vere a. Do dé: 2] (Veh + )] < VF A. 4) istjgn Cui cing, tir (13) va (14) ta duge két qué (sau khi thay vao vé phai cita (*’)) PS (n=) VmmRFT DP] + VIPs ea Dey]. (15) Vay Ta: pu =(n-1) [ vantBaF + DP] + VInlat al] (16) eX ==. 2 ack Fmt 1 (Kis ai +a5 @=1,2,...,n) Voi n= 2, thi duge py = 2(av2+ Va? +B), ta thy lai két qua da thu duge & phan I. ©) Chi thich: Dén day, bai toén 15 - mot bai ton eve tri dai s6da duge gidi xong. Nhung vi bai tofn cue trj dai s6 nay lai chinh la noi dung (ng6n ng) dai s6 cba mot bai toan eye tri hinh hoc trong khong gian Oclit n chiéu E,(n > 3), titong tu voi bai toan eye tri hinh hoc géc trong mat phing (da dude gidi thigu 3 1d gidi); béi vay, chi vige cho n = 3 vao céc biéu thttc (12) va (16) trén day cita Pm va pay ta duge ngay tite ‘che dép 56 cia bai todn cute tri tuong tu trong hinh hoc khdng gian khi ta thay dudng tron (O,a) béi mat edu (0, a), tt giée ABCD noi tiép duvng tron (O, a) e6 hai dudng 14 chéo AC, BD vudng géc véi nhau 6 mét diém P n&m trong dudng tron bdi hinh bat dign K(A.BCB'C’.A’) noi tiép mat cu C(O,a) ¢6 ba dudng chéo AA’, BB',CC’ cing a ba day cung cia C(O,a), vudng géc véi nhau doi mot 6 mét diém P cé dinh nim trong mat cu C(O,a) (OP =d b>0), @ aa, =, (i= 1,2,3) (i) Dh (a? + 2?) = 4a? + 207, (tii) Tim gid tri lén nhét sy vd gid tri nhé nhdt 5, cia ham 6 bién 2,21 (i = 1,2,3) sau day: f= s(en.zhiza zhi aa.a4) = ava (*) trong dé: {2,y, 2} C {1,2}; 22, 245234} vd x,y,z khong cb cing chi s6, tric la sao cho {x,y} A (uth, {uz} A (xia) 08 {2,2} # (ta aihs 2 © (1,2,3}. Chai thich: Dudi dau E cé tat c& 8 can thtte bac hai ma biéu thitc duéi diu cdn thitc 18 8 da thite thudn nhét bac 4 cia ba trong 6 d6i s6 2z,.r; nhumg khong cing mot chi s6 (i € {1,2,3}). Cu thé la biéu thie (**) ciia s c6 thé duge viét lei nhu sau: S= +S, +S. t+S,4+ Sgt 53+ Sat Sey trong d6 4S? = fe} + af(e +23), 457 = agg + 2}(xp +29); 483 = 30} + ap} +2}), 497 = 2fry + 2i(2z + 27); 453 = cha} tof (a} +23), ASf = ajay + 23(07 +22); 2g gdp dy 9 2y2, 48? = che} + o3e? + oj}, poy + afay + apap 1a nhiing biéu thttc duéi déu cin thiic bac hai é vé phai cita (**) e) Goi ¥ hudng gidi. Sir dung dai luong bat bién (ii), hy tim céch rit bét s6 cain thite & vé phai cia biéu thie (**) tir 8 xubng 4 rdi 2. Dat X; = 24 +24 (i = 1,2,3), ching 66 rang: 2 Dox? = 4a? + 86? = 4a? + 2.40? = XG + 205 (Xo = 2a, a0 = 26). (iv) iI Ngohi ro, sik dung eée dai lugng bét bién (i) va (il), hay, thiét ap shem ning he thie lien quan dén cdc dai luong Dy, 5? + 97, Dresejea XPX} va che dai luong bat bién (i), (ii) va (iv). 15 £) Dap sé cia bai toan. Hay chting minh ring biéu thitc dai s6 (**) cia dai long bién thién $8 = 8(1, 24; 2,05; 23, 2) cla 6 bién 2j, 2{ (i = 1, 2,3) thyc duong rang budc bai céc bat ding thite va ding thttc diéu kién (i), (ii) va (iii) v8 céc bién 2,,2/, dat ou dai sae khi va chi khi X; = Xz = Xz va dat cuc tidy sy, khi va chi khi, ching han: Xy = Xp = 2a va Xp = X3 = zo = 2%. Tir d6 ma thiét lap duge cc gid tri cyte dai syy VA cue tidu gm theo a va b. g) Vi ndi dung bai toan cyc trj dai s6 nay chinh 14 ngon ngit dai s6 cia bai ton va cuc tri hinh hoc trong khéng gian duge d8 c&p dén & muc c) "chti thich" vita méi néi é trén (cy thé la cue tri vé din tich toan phan cia mét ho cdc hinh bat dién KC noi tiép mot mat cdu (O,a) theo céch cfu tao d& chi ra trong bai toan) nén gidi bai toén nay ciing tite lA chiing ta da gidi bai toén cuc tri hinh hoc khéng gian bling phutong phép dai 86. Va do d6, dap s6 chi ra 6 day cing la dap 96 cita. bai ton cye tri hinh hoc khong gian dé. 116 Bat bién, don bién va ting dung Trdn Nam Dang Tit cd réi sé déi thay, chi Tink yéu vd Niém tin la mai mai Mé dau Bat bign 1a mot trong nhing khéi nigm trung tm ofa toén hoc. N6 6 mat trong hhdu hét eée linh vue cia ‘Tod hoc: Dai s6, Hink hoe, To-po, Ly thuyét s6, Xéc suft, Phuong trinh vi phan A Chang han, céc bat bién duge sit dung trong viée nghien ctu cée dé thi phing (dinh ly Kuratowsky), gidi tich ham (ching minh dinh ly v8 diém bét d6ng Brawer hay ching minh hinh cau khong déng phoi vai xuyén. Kh6 6 dink ly vé Phan loai (nhém, dai s6, 48 thi A) nao la thiéu su 06 mat cia ofc bat bién. C6 hin mot ly thuyét bat bién nghién citu cde dang bat bién dai s6 ciia céc bién déi tuyén tinh, Bat biém la nhiing dai Ingng (hay tinh chét) khong thay déi trong qué trinh ching ta thy hien céc phép bién déi. Ching han khi thie hien phép tinh tién thi khong céch Mot vi du khéc v8 bét bign: Léy mot s6 nguyen duong NV (viét trong he thap phan). Phep bién déi 7 bién N thanh téng cfc chit 86 cla N. Vi du: 1907 B88. A Vay c6 gi bat bién 6 day? C6 day, tat cd c&e 86 N,T(N),T(T(N)),... A ddu 06 cing 86 dut khi chia cho 8. Dé chinh la bat bién Dan bién, tréi lai, IA mot dai Iugng Iudn thay dé nhung chi theo mot chigu (tite 1a tang lén hay gidm xuéng) Ching ta sé dinh nghia mot cach chat ché vé b&t bién cling nh dan bién trong cée phin sau, 8 day chi ding lai & mot s6 vi du. Xét b9 6 nguyén duong (a, ,c). Phép bién déi T bién (2,0,¢) thank (6c, Jcal,Jabj) Khi d6 c6 thé ching minh dude rang ham s6 S(a,b,c)=atb+e 18 mot ham khong tang, tite la mot don bién d6i voi phép bién déi 7. ‘Trong bai viét nay, chting ta sé tim hiéu va bat bién, don bién va ting dung cla ching trong vige gidi cée bai toin Olympic. Cs hai mAu bai toan téng quét thutng duge gidi quyét bang bat bién va dan bién: u7 Bai 1. Co mot tap hop ofc trang thai O wa tap hop ede phép bién déiT ti va0 0. C6 hai trang thdi a va 8 thuge 2. Héi 6 thé diing hitu han cdc phép bién déi thude T dé dua trang théi a vé trang thai 8 duoc khong? Bai 2. C6 mot tap hop cdc trang thai © va tap hgp céc phép bién déiT ti Quao Q. Can chiing minh ring, bdt déu tit mét trang thai a bat kj, sau mot sd hitu han cde phép bién 46i tT, ta sé di dén trang théi két thiic (trong nhiéu trudng hop, a6 la trang thai én dinh, titc la sé khong tiép tuc thay déi khi tae dong cdc phép bién déi tix T, tinh Auéng T(8) =8 6 trén day la mot vi du). Bat bién va don bién sé gitip ching ta gidi quyét cdc tinh hudng cain ban nay. Tat nhién, céc tinh huéng 4p dung sé muon hinh van trang, nhung cling can c6 nhitng kién thitc co ban va 16i tu duy chung dé tiép can céc vin dé. Ching ta sé bit dau bang mot 86 vi dy don gidn, 1 Cac vi du mé dau Vi du 1. Xét mot bing wudng 4 x 4 0. Tai méi 6 cia bing vudng cb chia déu + hote déu —. Méi mét lin thyc hién, cho phép déi déu cia tét cd céc 6 trén cling mét hang hode ciing mot cbt. Gid sit bang vung ban déu c6 1 ddu + vd 15 déu —. Héi co thé dua bang ban dau vé bing cé toin déu céng duoc khéng? Cau tré Idi 1a khong. Va lai gidi kha don gidn, Thay déu cong bing s6 1 va ddu try bing -1. Xét tich tat cA cdc 96 trén bing vudng. Khi dé, qua méi phép bién déi, tich nay khong thay ddi (vi sé déi dau 4 s6). Vi vay, cho di ta thyc hién bao nhiéu lan, tir bang vudng (1, 15) sé chi dita va cée bing vudng cé sé 1 diu -, 06 nghia 1a khong thé dita v8 bing 06 ton d4u cong. Vi du 2. Trén bing cé cde sé 1/96,2/96,3/96, 96/96. Méi mét lén thuc hién, cho Phép zod di hai 96 a,b bt kj trén bing va thay bing a + b2ab. Héi sau 95 lin thwc hign phép od, 56 cin lai trén bing la 6 nao? ‘Toi rat thich vi du nay. Khi dat bai todn cho céc hoe sinh, ban n&o cling léc dau 18 Iwi vi dé bai yéu cau tinh todn qué nhigu. Hon nita, trong bai tosn trén, the tur thye hién cdc phép toan lai khong duoc néi rd, tao ra mot tinh huéng gan nhu khong thé xit ly ndi, Nhung chinh nhing kh6 khan dé lai goi mé ra céch gidi. O day toi sé khong trinh bay cu thé cdch phan tich dé tim ra hung gidi. Chi biét rang, khi loi gidi duoc dua ra, cée ban hoc sinh ddu v6 cing bat ng va thich thé. Ciing tit sit thich thi nay, viée dua ‘tip cdc vi du tiép theo duge don tiép mot céch néng nhiét hon han. Sau day 1a di gidi do. Gi sit céc s6 trén dang 1a a,,02,...,a,. Ta cho tuang ting bang nay véi (2a; ~ 1)(2a — 1)(2a4 ~ 1) 118 hi d6, sau mi lan bién déi, tich trén bi mét di hai thita s6 (2a — 1)(2 ~ 1) va duge them vio thita sé 2(a +6 2ab) — 1 = ~(2a ~ 1)(26 ~ 1). Do dé tich trén vain khong déi (chi déi déu). Vi tich ban dau bing 0 (do bang ban dau c6 chita s6 48/96 = 1/2!) nén s6 cudi cling s cing phai cho tich s6 bang 0, ttic IA 251 = 0, suy ras = 1/2! That dn tugng. Két qua khong phu thude vio thit tt thuc hign (diéu nay 6 thé dy dodn duge qua cach dat cu héi 6 dé bai). Va ldi gidi méi ngiin gon lam sao! Khong cin mét tinh todn no, Vi du 3. Cho 3 sé nguyén khong am a,b,c bat ky. M6i mét lan thyc hién, ta bién b6 (a,b,¢) thank 66 (bel, loa| lab). Chung mink rng sau mot s6 hu hon cdc phép bién déi, ta thu duce 06 6 chita sé 0. Dat M = max{a, b,c}. Ta chting minh ring néu bd (a, ,) khong chita s6 0 thi M S@ gidm sau khi thue hign phép bién déi. That vay, khong mit tinh téng quét, gid sit a> b>c. Khi dé ta c6 [be] ,>,<. Ngoai ra, tap dich N ciing c6 thé duge thay thé bing mot tap hop c6 thit tu t6t bat ky. Don bién duge sit dung trong viée chiing minh mot qué trinh la dig. Ching ta minh hoa ting dung nay thong qua mét s6 vi du. Vi du 10. Cho ham sé f:N x Z—Z thod man déng thai céc didu kién sau: i) F(0,0) = 795, F(0,n) = 0 vidi moi 6 nguyen n ¥ 0, ‘m—1, eee m—1n+1 ii) f(omn) = f(m —3,m) ~ 2[ = Dam) [feape 4). [L ety) voi moi sé te nhién Chitng minh ring tén tai sé nguyén duong N sao cho f (m,n) = f(m,n)¥m,m > N, n€eN. Vi du 11. Cho 2n diém trén mat phéing, trong dé khéng c6 3 diém nao théng hang. Chiing minh ring nhiing diém nay cé thé phan thanh n cap sao cho céc doan thang néi chting khong cit nhau. Loi gi Dau tién ta phan c&p céc diém mét céch ngdu nhién va néi ching lai voi nhau. Goi $ Ia téng cdc doan thing duge néi (Chit ¥ ring, do chting ta c6 hitu han cach phan cap nén tp gid tri cia S 1a hiru han). Néu cé hai doan thing AB va CD cit nhau tai O thi ta thay AB, CD bing AC, BD. Vi AB + CD = (AO + OB) + (CO + OD) = (AO + 0C) +(BO+0D) > AC+ BD theo bat ding thie tam giée, nén néu c&p doan thing ndo dé giao nhau, ta cé thé thay thé cach néi dé S giém xuéng. Vi S chi c6 hifu han céc gié tri nén mot lic nao d6 qué trinh phai dimg. Va khi d6, sé khong c6 cée clip doan thang giao nhau. 4 Bai tap 1.6 Vuong quéc OSée mau ky 400 6 45 higp si: 13 hiép sf t6c dd, 15 higp si téc vang, 17 higp si t6c xanh. Khi hai higp si 6 mau téc khac nhau gp nhau, téc cia ho sé lap 124 tite di sang mau tht ba. Héi cé thé cé mét lite nao d6, tt c& cdc higp si d&u cé mau t6e giéng nhau? 2. 06 7 chiée cbc dung nuéc: chiée céc thit nhAt chtta 1/2 nude, chiée cbc thit hai chifa 1/3 nutéc, chiéc thit ba chia 1/4 nuéc, chiéc thit tu chita 1/5 nut6c, chiéc thit nam. chita 1/8 nutée, chiée thit sau chiia 1/9 nude va chiée thit bay chtfa 1/10 nuée. Cho phép dé tit c& nude tix cbc nay sang cc khéc hoc dé nude tit céc nay sanng céc khéc cho dén Kkhi cbc chua day. Co thé sau mot sé lin dé nude, mot chiéc c6c nko d6 chia a) 1/12 nwtéc b) 1/6 nutéc? 3. C6 1 bang vudng n x n. Trong n — 1 0 cita bing cé ghi céc s6 1, trong cde 6 con Jai ghi s6 0. Cho phép thyc hign tren bang phép bién déi sau: chon mét 6, gid s6 dang viét 6 6 dé di 1 don vi va ting tét cd céc 56 4 cae 6 cling hing, cing cot véi 6 nay len mot don vi. Héi c6 thé tit bing ban dau, sau mot s6 phép bién déi, thu dude bang gdm tolin cée s6 bang nhau? 4. Trén bang cé 4 s6 3, 4, 5, 6. Méi mot lan thuc hin cho phép x6a di hai s6 2, y 06 trén bang va thay bling x+y + /z?+y?,0+y— a? +y®. Hoi sau 1 s6 hitu han bude thuc hign, trén bing c6 thé xudt hign 196 nhd hon 1 dutge khong? 5. Trén ban c6 100 vien keo. Hai ngudi cing thay phién nhau béc di k vién keo, trong dé & € {1,2,3}. Hoi ai la ngudi c6 chién thugt thang? Cing cau héi trén voi K€ {1,2,q}, 18.86 nguyen duong cho truéc. 6. Trén bang c6 1 s6 nguyén. Ngudi ta ghi nhé chit s6 cui cing cia sé nay, sau dé xo di va Ong thém vao véi s6 cdn Iai trén bang 5 lin chif s6 vita xod. Ban du trén bang ghi s6 71°. Hi c6 thé sau mot s6 s6 lan thyc hién nhu thé, thu duge s6 19987? 7. $6 nguyen duong c6 4 chit s6 trén bang c6 thé bién di thinh m@t sé c6 4 chit sb khac theo quy tic sau: hodc cong thém 1 vao hai chit sé lign p ctia né, néu hai chit s6 nay du khong bing 9; hole trir di 1 tit hai chit s6 lign tiép cia n6, néu hai chit s6 nay déu khong biing 0. Héi bing cdc phép bién déi nhit vay, c6 thé thu duge s6 2002 tit s6 1234? 8. Hai nguéi choi trd choi sau, Ban dau 06 céc s6 1, 2, 3, 4. Méi mot lan thuc hién, ngwoi thit nhét cong vao hai sé canh nhau nao dé 1 don vi, cdn ngudi thit hai déi chd hai s6 canh nhau nao d6. Ngudi thit nhat thing néu sau mot nuéc di ndo dé t&t c& cde 86 bang nhau. Héi ngudi thts hai c6 thé cn trd ngudi thi nhat chién thing? Tai liéu tham khao 1. Tolpygo, Bat bién, Kvant, 12/1976 2. N.Agakhanov, Olympic Toén toin nuéc Nga, Nha xu&t bin MCCME, 2007 (tiéng Nga) 3. A.Schen, Trd choi vd chién thuat dudi quan diém todn hoc, Nha xuét bin MCCME, 2007. 4. Kin Y, Mathematical Games (I), Mathematical Excalibur, July-October 2002. 125 M6t s6 van dé cia Toan réi rac Nguyén Van Tién Dat van dé Trong chuyén d8 ndy ta sé xét cdc bai toén ma déi tugng nghién citu Ia céc dai lugng ri rac. Trong cude s6ng ta c6 thé hiéu céc hién tugng rdi rac !& nhimg hign tugng ma giita cdc thdi diém xudt hién cdn cé mot khoang thdi gian nao d6. Trong todn hoc ta 6 thé dinh nghia cdc dai lugng rdi rac nhu sau: Dinh nghia 1. Phdn ti 2 €.A duge goi la phan tit rdi rac cia tap A néu Fe > 0, did bé, sao cho V(xje)UA = {x} trong dé ta ky hiéu V(x; €) = (x —6;2 +6) va goi la e~ lan cncia diém x. Dinh nghia 2. Tap A duge goi la tap roi rac néu nd chi gdm che phin tt roi rac. Cac phan ti cia tap rai rac ciing duge got la cdc phan ti roi rac nhau. Chang han céc tap: N, Z, 0.9 := {0;1;2;3; 4;5;6;7;8;9} , ... la nhitng tap rdi rac. ‘Toan rdi rac nghién citu céc tap rdi rac cing nhw cde phan tit cia tap roi rac.VI céc khéi niém ctia Gidi tich nhut gidi han, dao ham, nguyén ham, tich phan cing nhw céc ham s6 so cp duge hoc trong chuong trinh phd théng chi duge 4p dung cho cdc ham s6 lién tuc nén trong Ton rdi rac khéng thé 4p dung c4c phutong ph4p va kién thite cha gidi tich (Trit mot s6 trutng hop dac bist.) Toén hoc nghién cifu va mo hinh hoé thé gidi thong qua céc quan tric. Cac két qua. quan trie thu duge trong nhiing thdi diém rdi rac nhau. Béi vay c6 thé néi Toén hoc chit yéu TA dita trén co sé rdi rac. Tinh lien tuc cia céc su kign, hign tuang xdy ra trong thé gidi thyc chat 1a mot sy gid dinh, tién dé, in - Prior ciia cdc nha todn hoc ma khong thé nao kiém dinh duge tinh ding - sai. Diu nay cing giéng nhut nh séngla IuOng tinh: 06 ca tinh chat séng va tinh chat hat. Béi vay, trong nhimg nam gan day, nhat 1a khi Tin hoc c6 nhimg thanh tyu dang ké va thdi gian tinh ton duge cai thign nhiéu thi Toan ri rac cing duge quan tam nhiéu hon ‘Trong nhing nam cuéi ca thé ky XX, 6 My cing nhw nhiéu nude Tay Au da dua Toén rdi rac vao gido trinh gidng day trong nha trudng phé thong cila minh. Tuy nhién, ‘6 Viet Nam hién nay, tuy c4c bai tap Toan rdi rac cing thutng duge thay trong céc ky thi hoc sinh gidi cae cp nhung Ton rdi rac vin 18 mot diém yéu trong ging day va hoc tap khong nhiing trong c4c trudng phd thong ma ngay ca 6 cdc trudng dai hoc. Khong nhiing d6i véi hoc sinh ma ngay cd déi véi cdc thay 6 gido. Hy vong tai ligu nay phin nao khée phye duge hién tugng trén, 126 1 Phuong phap phan chiing 1.1 Tiéu dan Phuong phép phan chting c6 1 14 mot trong nhitng phuong phép chtmg minh sém nhat ma lodi ngudi da biét dén, dac biet trong nghé thuat dién thuyét va tranh luén. Trong Todn hoc, phutong phép phan chting (FC hay la CM: Contra-Method) cing 1a, mét phuong phép rat thutng duge sit dung, dac biét khi cin ching minh tinh duy nh&t cla mot d6i tugng T ndo d6 tho man diéu kign (*) cho truéc(ma sy tdn tai cla T da duge ching minh truéc d6) ta thudng gid sit cn VT" # T, T’ cling thod man didu kien (*), tit d6 suy ra mot digu v6 ly. Vay diéu gia sit cia chting ta Id sai, ttc I T duy nhét Céc vi du minh hoa xin xem thém trong ching minh céc dinh ly hinh hoc, SGK HHI) 1.2 Cosé ly thuyét Co sé ly thuyét cia phuong phép FC 1a ede dinh luat trong Logic: Goi pj qj 7 1B cée ménh dB toan hoc nao d6 thi Dinh li 1 (Céc dinh luat co ban). 1. (Phi mau thudn): p &B 2, (Bai trung): p hotic p=T Dinh li 2 (Céc djnh Iuat phan ching). 1. (Phan ching): |= (p> 9) # (T=). 2, (Phan chiing suy rong) F ff ane? =9 (a) F 3. Eo saka=P Ak > F)=4. Céc dinh luat trén cé thé duge ching minh ching han bing phung phép lap bang gid tri chan lf hoc phugng ph4p bién déi tuong dugng. Chang han, ta cé thé chting minh (a) nhv sau: (pkg =r) p&q hote r (doa = b 7 hose b) Bf holic J hoic r (doa&b 47 hodc 6) 4 (B ho&e 7) hoke G (giao hoén, két hop va F @ r) +> p&F hokc 7 (P&T = 2) (dpem) 1.3 Ndi dung phuong phap phan chiing Dé chting minh khing dinh: p + q (a) bing phuong phép phan ching ta gid sit q sai (tie 1a 7 I ménh d& ding). Nén tit dé thu duge mét diéu vo ly (vl) thi diéu do 127 chitng t6 gid sit cia ta IA sai, tite la q ding B Diéu vo ly vl c6 thé thude mot trong cdc dang sau: +) Didu trai véi gid thiét p (vl =p). +) Diéu tréi voi mot trong nhiing kién thite da biét (v +) Diéu tréi véi gid sit phan chting (ul = 9). Khi xy dung ménh dé phan chiing (9) ta cin nhé: Lfep. 2. peg + B hote 7 3. p hose g B&G. 4. Vz, p(a) @ Iz, pla). 5. Fz, pla) Vz, pla). 6. Vx, pla) + Fe, pla) 7. 3x, p(x) + Va, p(a). 8. Néu p(x) := f(x) Rg(z) (a) thi p(z) la: f(x) Rg(z) trong dé, R va K duce xae dink nhw sau: Néu[R: thi | via Ngoai ra cn c6: Dinh If 3 (Dinh Iuat vé am ban). Cho A la mot cong thite Logic ma trong dé chi chita cde phép todn: Phi dink ; &; hotc d6i di céc ménh dé: py;pe;-- ;p,. Am ban cla A, ky higu la Ag la mot cong thite thu duge ti A bling céch thay: R~ PS BY Bis hoic ~ &;&~ hode . Khi dé BAe Ae 1.4 Trinh bay Idi gidi cia phudng phép phan ching Bai toan: Chitng minh p= ¢. Lai gidi. Gia sit nguoc lai, q sai, tite 187. Mags +s ul Vay gid sit cia ta 1a sai, tic Ia g ding. & 128 2 Thi du minh hoa Vidul. Cho f(x) = az? + bn +c. Gid sit a] +|6|+ [el > 17 (1). Chitng minh ring 3x € (01), |f(2)|>1 (1) Léi gidi. (phan chting), Gid sit (1) sai, tc Ia: Wee (0:1), [fe <1 (1) Chon x = 0; - 1, tit (7) ta duge: <1va la+b+el<1 ateeer = lal + [0] + fe] < 17 ao gtas Do la didn vo ly (trai voi (*)). Vay digu gid sit cia ta IA sai, tite I (1) ding, 6 Vi du 2. Chting minh ring tap cae s6 nguyén t6 P 1a tap vo han. Peigidl. (phan ching) Gid sitnguge lai, tap Pla hit han. Gd sit P = {p1;pai--- spn). Khi d6, tén tai s6 nguyen t6 én nhét, Goi dé la py. Tite la: Pu € PEVDEP, D< pm Xét 86 2 = pi.pr-+ pa +1. Ta c6, 2 €N; x khong chia hét cho cée 86 Pi; Pri GPa (i néu x ! p; mdo dé thi 1 p, : vO ly). Vay p € P. Ma hién nbign, 2 > p,. D6 la didn Yo ly (trai v6i céch chon p,). Vay diéu gid sit cia ta IA sai, tue 1a P hiu hen. Vi'du 3. C6 thé chia céc s6 tu nhien tir 1 dén 21 thinh eée nhém doi mot rei nhau sao cho trong méi nhém sé Ién nhat bing tng cia céc s6 cdn lai hay khéng?. Loi gidi.. Gia sit chia duce. Khi d6 téng cc s6 6 méi nhom la mot sé chin (bang hai Jan 86 lén nhfit ). Vay téng cia tat cA 21 s6 da cho IA 86 chin (vi céc nhém déi mét rai nhau va téng ciia cdc sé ch&n IA sé chan). Nhung téng cla 21 s6 dé IA 21.11=231 la sé 18, Digu vo ly do chiing t6 gid sit cia ta IA sai, tite 1a khong chia duge thanh cde nhém thod man yéu cu bai ra.62 Vi du 4. Co thé tim duge hay khong 5 sé nguyén sao cho ci trong 5 s6 dé lap thinh 10 sé nguyén lien tiép?. i. Gia sit tim duge 5 s6 nhu vay, goi ¢ 1a téng ciia 5 sé dé van la gid tri cita téng céc c&p hai sé. Khi dé 10 sé nguyén lién tiép n6i trong dé bai 1a m B+ Lv m49. Ta tinh téng T cia 10 s6 dé theo hai céich khéc nhau: Mot mat, Ton (n +1) + (n+2) +--+ (n+9) = 5(2n +9). Mat khée , T = 4s (do trong T méi s6 da cho cé mat ding 4 lin). Ti d6 suy ra 4s = 5(2n +9) la diu vo ly. Vay gia sit ban dau Ia sai, tife la khong thé chon duge 5 s6 thod man yéu cdu bai ra Vidu 5. Cho ba diém 4, B, C phan biét trén mat phang. Ching minh ring néu tén tai diém G thudc mat phiing 6 thod man téng cia hai sé mot OA +B +00 = thi diém G d6 18 duy nhét. Vidu 6, (Méréng cia vi du 5) Trén mat phing chon diém phn biét Ai, As, «+, An va n s6 thy m, mz, -++m, thod man m; +m +--+ +m, =m # 0. Chitng minh ring tén tai duy nh&t diém T thod man: m PA; + TA, +--+ 1m,TA, = 0. Vi dy7. (IMO 1982) Cho phuong trinh 23 ~ 3zy?+y'=n (1), (nw EN’). 1) Chiing minh rang néu (1) c6 nghiém nguyén thi né khong c6 nghiém nguyen duy nhat. 2) Tim nghigm nguyén cita (1) khi m = 2005. Lai giai . 1) Dé dang bién déi: 2 = Bay? +? = (y = 2) — By — 2)2? + (—2)? = (—y)? ~ 3(-y)(e - y)? + (ey). Tir dé ta cé: néu (z;y) la mot nghiém ctia (1) thi (y—2;—2), (—y;a— y) cing IA céc nghiém cia (1). Ba nghiém nay doi mot khée nhau vi néu 6 hai nghiém ndo d6 bing nhau thi ta cé z= y = 0 1a diéu v6 ly do n > 0. Vay néu (1) c6 nghiém thi né c6 ft nhdt ba nghiém phan biét, tttc 1a (1) khong thé c6 nghiém duy nhét.& 2) Véi n = 2005 gid sit (1) c6 nghiém (x;y) € Z?. Ta viét cdc nghiém theo mod 3, phuong trinh da cho tré thinh a+yi=-1 (mod3)s2+y= (mod 3). Do vay ta c6 céc trutng hap sau: (i) 5 =0 (mod 3) va y = -1 (mod 3); (ii) 2 =1 (mod 3) vay =1 (mod 3); (iii) = —1 (mod 3) va y = 0 (mod 3). ‘Trong trutng hop (i) dat x 3n—1, thay vio phuong trinh da cho ta due VT = ~1 (mod 9) con VP = 2005 = -2 (mod 9) => vo ly. ‘Trong trung hgp (ii), do (y — 2;~z) cing 14 nghiém, ma y — z = 0 (mod 3) va —# = ~1 (mod 3) nén ta cing thu duge mot diéu v6 ly. Tuong ty, tir truéng hop (iii) ta cing thu duge diéu v6 ly. Vay véi n = 2005 phutong trinh da cho khong cé nghiém nguyén. Vi dy 8. (Ti IMO 83) Tim t&t cd nhing ham s6 f(z): R* — Rt, la toan anh va thod man déng thdi hai diéu kign sau: Y) fe f(y) = vf), Ve, ye Rt 2) f(t) +0 khi 2 + +00. , Lai giai. Gia sit tn tai ham s6 f(x) thod man yéu ca Do f(z) l& ton énh val € R* nén 3y) € Rt: f(yo to dutoe bai ra, 1. Trong 1) chor =1, y= F() = F.-1) FL. F(yo)) = yo (1) = yo = 1 (do f(1) > 0) > yo =1 > f(1) = 130 Vay f(z) ¢6 mot diém bat dong la z = 1. Ta sé ching minh dé 1a diém bat dong duy lt. That vay, gid sit f(x) e6 hai diém bat dong x, y phén biet, khi dé ta c6: (fey) = fF) = vf (a) = ya = cy = zy ciing 1a diém bat dong cia f(z). @)1= 40) Gia st f(z) con c6 didm bit dong x # 1, khi dd, theo (ii) f(z) cdn 6 diém bat dong 5: Trong hai s6 d6 phai c6 mot s6 > 1. Gia sit x > 1. Theo (i) f (Fila) = ai(2) > + cing la diém bét dong cia f(z) ) 88 06 v6 sé diém bat dong 2, = 2", n € N’. Ta c6 day d6i s6 (2) > +00 (do x > 1), do dé day ham (fGa)) = (Gn) + +00. Digu dé trai voi 2): f(z) + 0 khi 2 —+ +00. Vay gid sit cha ta la sai, tric 18 f(z) co duy nh&t mot diém bat dong la 2 = Nhung trong (i) cho y= a ta duge f(xf(x)) = f(z), V2 € Rt > 2f(z) cing 1a diém bat dong cita f(x). Vay ta phai cé VoeR* af(a) =14 fle) = 2 1 Dé théy ham s6 nay tho’ man céc diéu kign cia bai ra. Vay f(z) = g ham sé cin tim. ‘Vidy 9. (Tit IMO 83) Cho cc s6 a, b, cE N*, doi mot nguyén t6 cing nhau. Chitng minh ring phuong trinh abe+ yea + zab = 2abe-ab—be—ca (1) khong c6 nghiém tu nhién. Lai gia. Gia sit (1) 6 nghiem (2; y; 2) € N? Tu (1) ta 66 (2 + L)abic. Ma (a,c) = (b,c) = 1 z+1ic, ma z+1>0 nen tit do eéztlze Tuong ty, c6 2 +1 >a, y+1>b. Tit céc dénh gid nay ta duoc VT(1) > (e-1)be+(b~1)ea+(c—)ab = Babe—ab—be~ca > 2abe—ab~bo~ca = V P(1) Diéu vo ly nay chiing té gia sit ctia ta IA sai, tite 1a phong trinh (1) khong c6 nghiém tut nhien. Vi du 10. (Putnam 1998) Chimg minh ring véi moi s6 nguyén a, 6, c, ludn tim duge 6 nguyén dung n sao cho sb f(n)=n3 + an? +bn+c khong phai 18 s6 chink phuong Léi gidi. Ta cin ching minh ménh d@ sau: V(a;b;¢) € 29, In EN*: f(n) khong phai ld s6 chinh phuong. (p) Nhan xét ring moi s6 chinh phutong déu = 0 (mod 4) hote Gia sit (p) 1A ménh dé sai, tite 1a: (mod 4) 3(a;bjc) € Z, Vn EN": f(n) ld s6 chinh phuong. (p) 131 Dac bist, tir dé 06: f(1) =a+b+c+1 — lasé chink phuong(?) f(2) =4a+2b+e+8 — 1&s6 chinh phuong(ii) £3) =9a+3b+¢+27 1a s6 chinh phuong(did) F(4) = 16a+4b-+¢+64 las6 chink phuong(iv) Ti (i) va (iv) ta c6 (4) — (2) = 2 (mod 4). Ma 2b la sé ch&n, cn theo nh§n xét thi: £(4) = f(2) chi c6 thé = 0, 1, -1 (mod 4) = 2b = 0 (mod 4). Ta (ii) va (iii) ta 6 f(3) — f(1) = (26+ 2) (mod 4). Tuong ty trén ta cing c6 26 +2 =0 (mod 4) = 2 gid sit cia ta la sai, tite 1a (p) 1 ménh dé ding. (mod 4) 1a diéu vo ly. Vay 3 Phuong phap cuc han. 3.1 Tiéu di MOt trong nhing lai khuyén 6i véi ngudi lam Tos la: "Hay xét cac trudng hgp dic biét" Khdi niém trutng hop dac biét cé thé theo nhiéu géc do: 1. Déi véi mot da gide, cdc diém diac biét 14 nhimg diém thudc canh (diém bién), cae dinh (diém eye bién) hoge céc diém ma tai dé c6 mot dic trung nao dé cia hinh dat gié tri dic biet (= 0 ; =00 ; khong x4c dinh ; dat Min ; dat Max ;v.v...), th dy diém trong tam cia hinh, 2. Déi v6i mot tap hgp sp thit ty, ede diém d&c biét 1a nhiing phan tit lén nhat hose phan tit nhé nhat cia tap hop. 3. Trén truc s6 6 ba diém dic bigt 18.0; +00 ; ~oo. 4, Déi vdi mot bai toan 6 diéu kign, cdc trutng hgp dic biét xdy ra khi céc bién 6 mit bing nhau hogc xdy ra diu bing trong céc danh gié cita diéu kién, 5. Déi véi mot ham sé, cdc diém dic biét 18 nhing diém ma tai dé ham s6 khong xéc dinh, ho&e = 0, ho&e dat eye tri, v.v... 6. Déi vdi mot dudng cong, dé Ia cdc diém gién doan, diém cue tri, diém uén, diém bien, diém tu cét, v.v. 7. Khi tim quy tich, tap hgp diém ta thutng xét céc vj tri dic biét cite diém chay, tir do tim ra cdc vj trf tutong ting cia diém thudc quy tich va du doan quy tich. 132 8. Trong Tam thin hoc, Kinh t6 hoc 6 mot 6 mot bign php chia tri duge goi la "Liéu phép séc", dé chinh la phuong phap dia hé thOng dang xét v8 trang thai Khiing hodng (dic bist) cé kiém soét, Cése diém, trutng hop dac biét néi trén duge goi chung la cae diém cyc han. ‘Tom lai, diém cuc han la diém ma tai dé mot dic trumg nao dé cia déi tuong dang xét dat khiing hoing, hay 1a 06 thay déi v8 chit. Tuy theo trutmg hyp ma diém cute han cé cde ten goi khée nhau, Ching han: "Diém chuyén pha" trong Vat lf, "Diém ky di" trong ly thuyét ham phitc, "Diém t6i han" khi xét sy bién thién cia him s6, "Digm gién doan" khi xét tinh lien tuc cia ham s6, "Hé den" trong Thign vin hoc (nhan tien néi them lA méi day, tée gid cia khdi nigm "H6 den", nha Vat ly kiét xudt ngudi Anh 1a Hawkins da phi dinh khdi niém nay) vw... Cac diém cuc han cia mot he théng cé mot vai trd quan trong trong viéc khdo sat he théng do, béi vay trong nhiing thap kj cubi cia thé kj XX da xaft hien mot ly thuyét soi la ly thuyét khiing hong hoac ly thuyét v8 cic diém ky di 3.2 Co sd ly thuyét Ta sé-chi xét mot s6 trudng hgp dac bigt, riéng cia diém eye han. Muén vay, trudc hét phi chi ra sy tn tai cia n6, Dinh li 4. (Ve su-tén tai diém cuc han cita tap hop) Trong tp gom hitu han phén tit ld céc s6 luon tn tai phdn tit ln nhét va phan tit nhé nat. Dinh ly trén 18 trudng hop dac biét cia Dinh li 5. Xét tap A gdm hitu han phan ti. Méi phan tit € A duoc dat twong ting vdi Ot trang thai f(x) ndo dé cia n6. Khi dé, néu mdi trang thai f(x) 06 mot dée trang P(f(x)) va tap cde dae trung: {P(F(2)) | 2 € A} co thé sép thit tu thi tén tai MinP(J(2)) va MaxP(f(2)) H6 qua 1: Néu vai tro cita cdc chi s6 trong tap gém n 86: 21; 22;+++ ;rq 1A nhu nhau thi ludn c6 thé gia sit: NSM ay Mot trong nhitng ting dung ciia diém cue han 1A nguyén ly néi tiéng ca Pontri Néu 3Min f(z) ; Maz f(x) thi cée gid tri Min ; Max dé thuang dat tai nhitng diém cue han ciia A 3.3 Noi dung phuong phap Khi khdo sét mot tinh chat ndo d6 cita cée phn tit cia tap A ta 06 thé xét tinh chat 6 tai diém cye han € A. Do z la diém oye han nén ta c6 them nhitng thong tin (diéu 133 kign) phu d6i véi a. Tir ede két qua cia vige kho sét tai nhiing diém cuc han ta 06 thé dit doan két qua chung cho céc phan tit € A. 3.4. Thi du minh hoa Vidy 1. (DN-IMO 1983) Ching minh ring trong moi tam giée ABC ta luén 06: h(a — b) + Be(b—c) +a(c~a) 20 (1) Lei gidi. Gia sit a = mar{a;b;c} + a—b>0; a~c 30. Tacé: VI(1) 2 a(b +c~a)(b—c)? + (a —b)(a—c)(a+b—c) 0 (dpem) Vi dy 2. (DS: 150; VDCHLBDic) Cho a, b,c € (0; 1]. Tim Maz cita: R a b c = ty 4g aaa Pyeti ‘avert tavos FO~MO-HA-9 Gia sit a = man{a;b;c}. Khi a6: bo. b ie 2 atet+1 b+e+1' atb+1 b+et1 (a) Theo bit ding thite Cauchy, c6 (1-H -Q+b40< (Ettingttbeey* 5 = = (1-1-9 < + (do14b4e>0) b+e4+1 Tit dé cé: ie (=a) < 2 B) (do1-a 30) Tit (a) ; (8) c6: a b c 1 ee ee dyesi bees] bet tote atbtctl-a b+etl Dau "=" xdy ra chdng han khi a = 6 = c =0 € (0; 1). Vay Maz F = 1.8) Vi du 3. Ching minh ring bén hinh tron c6 cée ding kinh 1a bon canh cia mot tit gide 18 thi phi kin mién tit gide a6. Léi gidi. Goi M 1a mét diém bit ky nim trong mién tit gide ABCD. Ta cé: AMB + BMC + CMD + DMA = 360° (tinh chat tit gide 18i) 134 Do dé, géc lén nht trong bén géc nay sé > 90°. Gid st BMC > 90°. Khi d6, M nm trong hinh trdn dutng kinh BC. Tir d6 c6 Vidu4. Trén dung thing cho tap X gém cdc diém sao cho méi digm M € X 1a trung diém cia doan thing néi hai diém € X. Ching minh rang tap X la v6 han. Loi gidi. Gid sit tap X 1a hau han, khi d6 tén tai diém Bla bién cia X, nhung B khi d6 khong thé la trung diém ciia hai diém € X, vay gid sit sai, tite Ia tap X 1A vo han. & ‘Vi du 5. Trén mat phang cho tap X gdm cdc diém sao cho méi diém M € X la trung diém ciia doan thing néi hai diém € X. Ching minh ring tap X JA v6 han, Vi dy 6. Tai mdi 6 ciia m@t bang 0 vuong vo han viét mét s6 sao cho méi s6 dutge viét bing trung binh cong cia bén s6 ké véi n6. Ching minh ring tat 8 cée s6 duge vit déu bing nhau. Vidu 7. Trén mot ban cd quéc té cd n x n dat cdc quén xe thod man diéu kién sau: néu ¢6 mot 6 ndo dé khong c6 quan xe thi tng s6 cic quan xe dimg cing hang va cling Ot vai 8 dé khong hs hon n. Ching minh wing trén bin c® e6 khong it hon quan xe. Lai giai . Ton tai mot dudng (hing hose cét, gid sit la hang ) c6 s6 quan xe nhé nhdt a 2 2 (gid sit 1A & quan xe ). Neu k > 5 thi tng s6 cdc quan xe S thod man S > n. ; = = Néuk < 5 thi trén dudng dang xét con n—k 6 tréng. Trong mai cot cia n—k cot di qua nhitng 0 tréng d6 06 > m— k quan xe va téng 96 cée quan xe nhit vay Ia > (n— k)? Trong k c6t con lai 6 > k? quan xe. Mat khéc, d8 théy: [(n— k)? + #3] Ti d6 suy ra Vidu 8. Mot nuéc c6 80 san bay ma khoang céch gitta céc céip sén bay bat ky déu khac nhau va khong c6 ba sin bay nao thing hang. Cling mot thai diém tir méi san bay c6 mét chiéc san bay ct cénh va bay dén sn bay nao gin nhat. Ching minh ring trén bat ky san bay nado cing khong thé cé qué 5 may bay bay dén. Loi gidi. Tir gid thiét suy ra néu céc may bay bay tit cdc sin bay M va N dén sin bay O thi MN 1a canh 16n nhét trong tam gidc OMN vi vay ZMON la g6c lén nhft trong tam giéc OMN va do dé ZMON > 60°. Gid sit O 1a san bay c6 s6 may bay bay dén la nhiéu nhét (= n) va cée méy bay dé bay dén ti cée sin bay Mi, Mp, --- My. (Ten cita cdc san bay duoc dat sao cho céc tia [OM,) va [OM1) 1 ké nhau va duge xép nguge chiéu kim ding hd, Mny1 := M.). Do khong c6 ba san bay nao thing hang nén ta sé duge n géc ZM,OM:y1 06 téng bing 360°. Géc nhé nh&t trong s6 cic goc dé phai < = Nhung theo trén, géc d6 phai > 60°. Tir dé te 06: 360° > OP >n 1a doan ngéin nhdt trong cic doan AyA; va Ap Ta duy nhat. Xét céc doan AzA, va tim doan ngiin nhét trong sé céc doan a6. Co thé xay ra hai kha nding: Kha nang 1: Min Ap, = AzA). Khi d6 duong di két thie tai Ap va khong 5eT. 2005342 6 thanh phé nao duge di qua hai lan. Kha nang 2: Min AzAj = AzAs. Khi do Ag la duy nat va ApA3 < AzAy. Ta eT 28572 di tiép dén diém A3. Déi vdi diém Ag ta cing lam nhu vay. Gid sit dudng di két thiic tai diém A, thi theo lap lugn trén, méi diém Ax, (k € Tn) duge chon mot céch duy nhat va AjA2 > Ardy > 0 > Aida > > Ant An Gia sik tren dung di ta di qua diém A; hai lan, thé thi dutng di trén chita mot dung g4p khtic khép kin AiAig Aisa AmAie ‘Theo nhan xét trén ta c6: AiAigs > Aisi Aga > > Am-1Am > AmAi => AAua > AiAm: Dé la diéu vo ly (trdi véi céch chon diém Ajy1 1a diém duy nhat cé khoang céch ng nhét ti A, Vay trén duttng di khong cé thinh phé nao duge di qua hai lan. 4 Nguyén ly Dirichlet. 4.1 Tiéu dan Phuong phép sit dung nguyén ly Dirichlet 1a phuong phap ma hoc sinh duge lam quen s6m nhét (tir tiéu hoc) va IA mot trong nhting phuong phép thé hién ré cai dep cia Ton hoc, lm cho hoc sinh thém yéu thich mon todn. Lap lun cia phwtong phép Dirichlet thudng duge sit dung trong céc bai toan cho hoc sinh gidi va ding dé chting minh sit tn tai mot kha nang nado dé ma kh6ng cn chi r6 khé nang dé tén tai khi nao, 6 dau va c6 bao nhiéu kha ning nlu vay tén tai. Phuong phép ching minh nhu vay con duge goi la phuong phap ching minh khong kién thiét. 136 4.2 Co sé ly thuyét Dinh li 6. (Nguyén ly Dirichlet. Peter Gustav Lejeune Dirichlet 1a nha todn hoc ngudi Diic, 1805-1059) Cén phan ti: dice xép hét vdo m tap hyp. Khi dé, tn tai (c6 tt nhét mot) tp hop chita khong it hon (>) = phan tt. Hay la: a» 4 |A| FA=QA; +34, ke Tm, [ay| > Ml i Vis6 phan tit cia mot tap hgp X, ky higu 1a [X], phai 18 86 tu nhién nén ta hiéu Khai nigm: [X| > = hu sau: . 2 Niu =ken m) thi +) Néu [= kEN (nim) thi X|> 2 @ ix|> +) Néu = NN (n khong : m) thi: IX > =e [x|> [#2] (ta ky hieu [2] = phan nguyen cia s6 2 € R) Dé dé nhé, nguyén ly trén cn duge phat biéu nhut sau: C6 n con thé dude nhét hét vao m cAi léng. Khi dé, tén tai (c6 ft nhdt mot) lang chita khong it hon (>) con thd. m Dac biét, néu 56 thé nhigu hon s6 léng (n > m) thi tdn tai lng chita it nh&t hai con thd. ‘Thuing ta chi xét trudng hop n > m. = |A| Lai gidi. (phan chimg) Gia sti nguge ai, tte Ia VA,, j € Tym, [Ay] < a @. Khi do: lal = Al <|Al la didn vo ly. 137 = |A| Vay gid sit cita ta 18 sai, tite Ia: 3Ay, k € Tym, [Aa] > i (dpem) Hé qua: Néu A la tap vo han va A duge phan hogch thank hifu han cée tap con Ay thi o6 it nh&t mot tap con cling 1A tap vo han 4.3 Noi dung phuong phap Dé sit dung nguyén ly Dirichlet ta phai lam xuét hign tinh huéng nhét "thé" vao "chudng" thod man céc digu kign: +) $6 "thé" phai nhigu hon sb chudng +) "Th6" phai dutde nhét hét vao céc "chuéng", nhung khong bit bude 1 "chuéng" no cing phai c6 "tho" ‘Thudng phuong phép Dirichlet duge p dung kém theo phuong phép phan ching Cha ¥:C6 nhidu bai tAp c6 két Iuan "ging nhwt" két Luan cita nguyen ly Dirichlet, tuy nhién, Idi gidi hoan toan khong sit dung nguyén ly Dirichlet. 4.4 Thi du minh hoa ‘Vidy 1. Trong hinh vudng 06 canh bling 1 dat 51 diém bat ky, phan biet. Ching minh ring c6 ft nhét ba trong s6 51 diém d6 nim trong mot hinh tron ban kinh ; Li gidi. Chia hinh vuéng da cho thanh 25 hinh vung con bing nhau c6 canh bing g Theo nguyén ly Dirchlet, tdn tai t nhét mot hinh vudng con (a) chia it nhét ba trong s6 51 diém da cho. Dudng tron ngoai tiép (a) c6 bén kinh 1a ra < ; Vay ba diém n6i trén nim trong hinh tron déng tam véi dung trdn ngoai tiép (a) c6 ban kinh BR ; (épem) Vi dy 2. Trong hinh tron (C) c6 dign tich bing 8 dat 17 diém phan biét, bat ky. Ching minh ring bao gid cing tim duge it nhat ba diém tao thanh mot tam: gide 06 dign tich bé hon 1 Ldi gidi. Chia hinh tron (C) thanh 8 hinh quat bing nhau, méi hinh quat ¢6 dién tich bang 1. Theo nguyén ly Dirchlet, tn tai ft nhét mot hinh quet (a) chia it rihét ba trong s6 17 diém da cho. Tam gidc 6 ba dinh la ba diém d6 nam tron trong hinh quat (a) nen c6 dign tich nhé hon dign tich cia hinh quat, ttc 1a bé han 1. 6 ‘Vi dy 3. Chon 5 ngudi bat ky. Ching minh ring 6 it nhét hai ngudi cé ting s6 ngudi quen trong s6 5 ngudi da chon. ‘Vi dy 4. Trong mot gidi v6 dich bong d4 c6 10 doi tham gia. Hai di bat ky phai thi déu véi nhau ding mot tran. Chimg minh ring tai moi thai diém cia gidi luén 6 hai doi da 06 86 tran déu bing nhau. ‘Viidy 5. Chiing minh ring tit 12 s6 ty nhien bit ky Iuén chon duge hai s6 c6 hiéu chia hét cho 11. Vidy 6. Viét-n sé tut nhién thanh mot hang ngang. Ching minh ring hode c6 mot 86 chia hét cho n ho&e c6 mdt s6 s6 lien tiép c6 téng chia hét cho n. 138 ‘Vi dy 7. Ching minh ring tis 52 s6 ty nhién bat ky sao cho hoc téng, hoée hiéu ciia hai s6 d6 chia hét cho 100. Két luan con ding khong déi véi 51 562. Vi du 8. Trong mot hinh vuéng don vi chon tuy ¥ 101 diém (c6 thé thude canh cita hinh vudng) sao cho khéng cé ba diém ndo thing hang. Chitng minh ring tén tai tam gidc vi ba dinh la céc diém duge chon va c6 dién tfch nh han 0, 01. ‘Vi dy 9. Trong hinh lap phuong don vi c6 2001 con rudi. Ching minh ring 06 it " 1 nhét ba con rudi nim trong mdt hinh cu bén kink 7 ‘Vi du 10. Mot s6 cung cita mot dudmg tron dude to mau den, céc cung cdn lai duge t6 mau dé. Biét ring téng do dai cde cung mau den nhé hon nita chu vi cba dutng tron, Chting minh ring ¢6 thé ké duge mot diftng kinh cia dung tron véi hai du mit duce t6 mau dé. Vi dy 11. Trong hinh vudng ABCD c6 canh bing lcm dit mét s6 hinh tron 06 téng céc ban kinh bing 0,6cm ( céc hinh trdn c6 thé oé diém chung hoc ké ca tring nhau ). Chimg minh ring tdn tai mot dutng thing song song véi canh AB va cé diém chung véi it nhat hai trong s6 nhiing duéng tron néi trén. Vi'du 12. Trong s6 100.000.000 s6 hang dau tién cia diy Fibonacei: pel gO Bie 6 tén tai hay khong s6 hang tan cing bing 4 chit s6 0?. ‘Vi du 13. Ching minh ring véi moi s6 ty nhién n, trong day s6 Fibonacei néi trén lu6n tén tai sé hang tan cing bing n chit s6 0. Vi dy 14. Trong céc 6 cia mot bing c.n x n dat mot cach tuy ¥ cdc s6 nguyén tix 1 dén n?. Xét khiing dink sau: "Luén tim duge hai 6 c6 canh chung sao cho hiéu ctia hai s5 ném 6 hai 6 dé lén hon 5" 1) Ching minh ring khang dinh dé ding véi n = 10. 2) Chimg minh rang khang dinh d6 dting véi moi n > 10, 3) Ching minh ring khang dinh d6 ding véin 4) Chitng minh ring khang dinh d6 khéng ding véi n = 5. 5) Hay xét tinh ding, sai cia khiing dinh 46 khi n = 6, 7, 8 ‘Vi dy 15. Trén mat phing cho 5 diém A, B, C, D, E c6 cdc toa do 1A cdc 86 nguyén. Ching minh ring trong sé cc tam giée dugc tao thanh tit 5 diém dé c6 it nhat ba tam gidc c6 céc dign tich nguyen. Léi gidi. Nhan xét rling néu thay ddi toa do cla mot dinh mot s6 chin don vj thi tinh nguyén cia dién tich khong déi. Ta dich chuyén toa dd cita cdc diém da cho nhitng 86 chin don vi sao cho thu dugc cée diém méi 4’, B,C’, D’, E’ ma céc toa dé chi la cite 86 0 ho&e 1. Chi 6 4 trutng hop (050) 5 (051) 5 (250) ; (451) ma c6 5 diém nén sé c6 hai diém tring nhau. Gid sit A’ = BY = 0(0;0). Khi dé, ba tam giée OOC’, OOD’, OOE’ sé cé céc dién tich bing 0 € Z nen céc tam Bide ABC, ABD, ABE sé cé céc dién tich nguyén.@ 139 5 M6t chit vé sang tao trong Todn hoc ‘Trong phan nay ching ta sé lam quen véi m6t trong nhiing phucng phap séng tao trong Todn hoc, d6 la phuong phép téng quét hod. 5.1 Thi du mé dau Ta sé tim cdch mé rong mot khang dinh hinh hoc da biét sau: Qua hai diém phan biét c6 thé ké duge mot va chi mot dung thang | (I) Hign nbién, vi dé 18 mot két qua cia hinh hoc nén truéc hét ta tim nhiing mé rong vé mat hinh hoc. Moi ngudi déu biét, qua ba diém khong thing hang c6 thé dung dugc mét va chi mot dung trdn. Mot s6 ngudi véi kién thitc todn hoc rong hon c6 thé biét ring qua bén diém 6 thé dymg duc mot Parabol, cdn qua nam diém cé thé dung dugc mot Elip. Nhung sau d6 Ia ng6 cut! Ta khong nghi ra, duge dung cong n&o di qua su diém. ‘Ta sé tim céch mé rdng theo con dutng dai s6. Ta biét ring moi dudng thing khong vudng g6c véi 2’Ozx déu c6 phuong trinh: y=art+b (d) Khing dinh (1) "dich" sang ngon ngit dai s6 c6 dang (II) Luén tim duge duy nhat b6 sé a;b sao cho dudng thang (d) : y=ax +b di qua hai diém M(2x1; 11) ; (x2; ye) cho truée (x; # 22). N6i c&ch khdc, hé antb=n — ) ary +b= yp a; 0) ludn c6 nghigm duy nat véi moi Tis Mi t23 yo (ti #22) Nghigm cia he (1) 1a Waw, 4 _ tik man 7 Ty — 22 ‘Tir phuong trinh (2) ta rit ra duge nhiing nhan xét i? 1) Trude hét, 2 — 22 1a biéu thitc duy nhét nim 6 mAu thttc. Digu dé c6 nghia la néu x; = zp thi biéu thite 6 V P(2) 1A vo nghia. Khang dinh dé phi hop v6i bain chét hinh hoc cita bai todn vi khi a, = za thi (d).La’Or ma ta chi xét nhiing dudng thing khong L2”Oz. 2) VP(2) la bac nhét d6i véi yr va ye. Noi cach khéc, khi ta cho 2); 22} z nhing gid tri cu thé, tuy ¥ thi (2) luén cé dang ys Aut Bas trong d6, A; B la cée hing s6. Tit nhgn xét ny ta viét lai (2) vé dang bac nhdt d6i véi yy va yo, duge n @ #y=n ($2) +n (2) = nue) +y2-0(2) (8) 3) Cac ham u(2) ; v(x) c6 nhitng tinh chat sau +) Khong xée dinh khi x; = 29, +) way) = 15 u(z2) = 0 va tutong ty v(z1) = 0; v2) = 1 6 M6 hinh hod toan hoc ciia cdc bai toan Phiin 16n cée bai tap todin ma ta gp du o6 két Inn duge phat biéu dudi dang twong ‘minh, tife lf trong dé bai yeu cu r6 ta phai lam gi. Chiing han phai gidi phuong trinh, bat phuong trink, hé phwong trinh, ching minh dang thite, bat ding thie, v.v. ‘Tuy obién gid thiét cia céc bai tap thuting duge téc gid "phitc tap hod” nhim lam cho ching ta kh6 phét hign ra méi lign he giita "cdi di cho" vA "cdi can tim", khong phan biét duce diéu kien chink va diéu kién phy, hoc doi khi khong phét hien duge nhing diéu kién thita, didu kign gay "nhiéu" Nhiém vu cia ngudi gidi ton 18 "phét biéu lai" bai toan da cho, lam cho gid thiét va két lun tré nén c6 lién hé véi nhau va duta bai ton cn gidi v8 mét hotic nhigu bai toan quen thudc, da c6 thut toén gidi san. Mun vay, Kkhi gid thiét cita bai togn duge cho bing "Ibi" , duéi "dang vin hoc", ta phat "dich" ngon ngit "vin hoc" ciia bai toan da cho vé ngOn ngit todn hoc quen thude Doi khi chiing ta cén phai ding cdc phuong phap dic biet dé chuyén dang bai toén. Ching han ti bai ton dai sd vé bai toén hinh hoc ho&c lugng giée hay 1& ngugc lai. Con duting nhu vay duge goi 18 "m6 hinh hod" bai toén da cho. Trong phin nay ta sé dua ra mt s6 phuong phdp "m6 hinh hos" thudng gap, chii yéu ding dé gidi cfc bai toan té hop. Cée vi du va bai tép ciia phan nay xem them trong tai ligu: Ly thuyét hé théng. 6.1 Té mau Vi dui. Ching minh ring trong sdu ngudi bt ky hoe e6 ba ngudi doi mot quen nhau, ho&e c6 ba ngudi doi mot khéng quen nhau 141 Lai gidi. Bigu dién m6i ngudi béi mot diém trong khong gian sao cho khong c6 ba diém nao thing hang va khong ¢6 b6n diém nao ding phing. N6i hai diém bat ky bai mot doan thing. Té miu céc doan thing thu duge (15 doan) béi hai mau: xanh va 46 nhu sau: ‘Néu hai ngudi quen nhau thi doan thing néi hai diém biéu dién hai ngudi d6 duge 10 bing mau dé. ‘Trong trutng hop nguc lai doan thing tuong ting dude tO mau xanh. Chon mot diém A nao d6. ‘Trong 5 doan thing n6i A v6i céc diém cdn lai cé it nhat ba doan thing cing mau (?). Khong gidm tinh tng quét gia sit d6 1a mau 46, Goi ba doan d6 1a AB; AC; AD. ‘Xét mau cia ba doan BC ; CD; DA. Cé thé xdy ra hai kha ning: +) Néu c6 mot doan mau d6, ching han BC mau dé, thé thi tam giéc ABC c6 ba canh cing mau dé, tite 1A ba ngudi tuong img vdi ba diém A; B; C doi mot quen nhau. +) Néu khong c6 doan nau mau dé, thé thi c& ba doan déu c6 mau xanh va khi 46 ba ngudi tuong img véi ba diém B; C; D doi mot khong quen nhau. Tom lai, luén tén tai hodc ba ngudi doi mot quen nhau, hodc ba ngudi doi mot khong quen nhau. D6 chinh la dpem. ‘Vidy2. Ching minh ring khong thé lst kin mot ban c¥ quéc té (8 x 8) bing 15 L — domino va 1 V — domino nhu hinh sau: L— Domino ee V — Domino Ban c¥ quéc té Lai giai. To mau ban cd nhwr hinh vé. Khi dé, 06 32 6 den va 32 6 trang. Voi moi vi tri, mai V — domino phii hai 6 den va hai 6 trang, cn mdi L — domino phii mét sé 18 6 den va mét s6 1é 6 trang. Nhu vay, 15 L — domino va. 1 V — domino chi phii dutge mot 86 18 6 den va do dé khong phii kin hét ban cd (dpcm). Vi du 3. Cho mét bin ci cd 4 x 50. Mot con ma ditng 6 6 sét canh ban c¥ va di theo duong chéo hinh chit nhat 2x 3. Héi cé tén tai hay khong mot dutng di cia quan ma lien tiép qua t&t cA cdc 6 cia ban c¥ mdi 6 mot lan hay khong? Lai gidi. TO mau céc 6 ciia ban cd bing 4 mau: Tréng (T), den (D), vang (V), xanh (X) nh hinh vé sau: 142 Xanh: Vang: Vang Xenh | Ban cd 4 x 50 Gia sit tén tai mot dudng di thod man yéu cdu bai ra va ban déu quan ma ding 66 XT, khi do céc 6 tiép theo ma quan mA cé thé di dén I: XT VD3XTsVD—- Nhu vay eée 6 VT, XD s® khong bao gid duge di qua. Vay khong tdn tai mot dudng i thod man yéu cu bai ra. 6.2 Xay dung bat bién Dé gidi céc bai toan dang: Cho tap M ( ma cdc phan ti ciia né sé duge goi la eéc trang thdi). Cho mot quy tc Q sao cho Khi dp dung quy tde Q d6, tx mat trang thai € M ta thu duge mot trang thdi khdc € M. Cho truéc hai trang thai a; 8 € M. Héi sau mot $6 hiu han bude dp dung quy téc Q, tit a ta c6 thé thu due 8 hay khong? ‘Trude hét ta xay dung mot 36 ky hiéu: +) Ta ky higu 9 = Q(a), hay la a + néu theo quy tée Q, tit trang théi a ta thu duge trang thai 2. +) Ta ky higu G ~ a (doc 18 a tuong duong véi 8) néu tit a c6 thé thu duge B san mot s6 hiu han bude ép dung Q. Phi dink cita diéu d6 duge ky hiéu lk 8 « a. Ta sé chi xét cdc quy tic Q cé6 nhiing tinh ch&t sau: 1) (Tinh phn xa) Va € M, a~a 2) (Tinh déi xing) Vae Ma~ ps B~na. 3) (Tinh bée cu) Neuan~ B&B ~7tha~ 7, Quy te tho& man ci ba tinh chét trén cdn duge goi IA quy tie o6 tinh tong dung, Dinh If 7. (Vé sy phan Iép tap trang thai theo quy tac tudng duong) Neu Q Ia quy the 06 tink tong dong thi M duge phan thanh ofc tap con déi mot khong giao nhaw M = M,UM,U Myu-- thod man: Véi moi i, JEN’, ludn 06 Va; GEM, ang. 143, ii) Neui#j thiVaeM, VBEM, a8 Dinh nghia 3. 1 Méi tap con M, néi trong dinh lj trén duge goi la mot qui dao ciia quy the Q (zéc dinh tren tap M) Hién nhién, céc qui dao cé tink chét quan trong sau: Néu ta zudt phat tit mét trang thdi bit ky thugc quy dao M, nao dé thi sau mot hoac mét sé lan dp dung Q ta chi cd thé thu duoc nhing trang thdi ciing € M; dé va khong thé thu duge cde trang théi thuée cde guy dao khac. Ngowi ra, néu M la tap hitu han thi sé quy dao ctia quy tée Q xéc dinh trén tap M a hitu han. Dinh nghia 4. 2 Ham sé f xéc dink trén tap céc trang thai M duoc goi la mot bat bién ctia quy tde Q (zéc dinh tren tap M) néu nd thod man a~ B= f(a) = f(9).(1) Dinh nghia 5. 3 Bat bién f duge goi la bat bién téng quét néu né thod man a = Hla) # £(8).(2) Vay f la bdt bién téng quat khi va chi khi Va, BEM, a~6@ f(a F(A), Vay néu ta xéc dinh duoc mot bit bién téng quat f thi vdi hai trang thdi a ; 8 bat ky, 06 thé xéc dink duge a ~ 8 hay la a ~ i bing cdich so sénh f(a) va f(B). Néu ta chi ze dink duge mot bét bién f vd udi hai trang thai a ; f ta cb f(a) £ (8) thi ta két ludn a ~ 8. Tuy nhién, néu fla) = f(B) thi khong thé két ludn gi vd méi tuong quan gitla a va §. Dé cay dung bat bién téng quat thudng sit dung dink ly saw Dinh li 8. Néu a) Tén tai tap A := {61 ; 62; «+ ; &} gdm trang thi phan biét cia M sao cho VaeM, 346A, a~6, 6) Bat bién f nhan it nhét l gid tri khéc nhau. Tha f la bat bién téng quat va céc trang thai € A la doi mot khong tuong dung. Ching minh: Tita) ta thdy c6 khong qué l qu dao. Tit b) ta théy c6 khéng it hon t quy dao. Vay c6 ding l qug dao. Tic b) ta thy f nhan diing | gié tri, suy ra f la bat bién tong qué, Cuéi ciing, tit a) ta thdy céc trang théi khdc nhau ciia A sé thuge cde guy dao khac mhau nén doi mét khong tong duong 144 6.3 Vidu Vidul. Hinh tron duge chie thank n hink quat. Ta 2ép n vin séi mét céch tuy ¥ vion hinh quat dé. Duge phép déng thai chuyén mét vién sdi tit mot hinh quat baaats ‘ky sang hinh quat ké bén theo chiéu kim ding hd va chuyén mot vien sdi tir mét hinh quat bat ky sang hinh quat ké bén theo hudng ngudc lai. Héi ring lam nhu vay sau mot 86 hitu han bude cé thé tir vi tri ma 6 méi hinh quai 06 ding mét vién sdi thu duge v4 tri ma tat od ode vién séi duge tap trung vé trong cing mot hinh quat hay khong? HDG: Dénh 36 céc hinh quat tir 1 dénn theo chiéu kim déng hd ké te mot hink quat nao d6. Voi méi trang thai ép séi a, goi x(a) 1a sd vién séi trong hinh quat thit k. Dat q(a) = 1.a(c) + 2.09(a) +++ + n.ag(a) war(a) = q(a) (mod n) Ta chiing minh cdc khéng dinh sau: 1) r(a) la bat bién tong quét. 2) Goi 8 la trang thai ma d méi hinh quat 06 ding mot vién s6i ; + ld trang théi ma 7n vién dt tép trung vio hinh quat thit | thi r(y) = 0 va r(8) =0 4 n la sé Ie 9) Tit d6 suy ra Bryern la sé te 6.4 Xay dung ham dic trung Voi bai toén duoc phat biéw trong phan trudc, néu chi yeu ctu ching minh tt trang thai a sau mét hoac mét sé hitu han lin ép dung quy tde Q ta cé thé thu dugc trang thdi 8 (a; B la cée trang thdi cho trude) ta cb thé gidi nhu sau: 1) Xay dung ham s6 f(k) trong dé k € N° la sé lan dp dung quy tie Q vao trang théi a. 2) Chiing t6 f(k) la ham khong gidm ( hoc khong tang) 5) Ching t6 f(k) nhan gid tri lén nhat ( hotic nhd nhét ) 4) Chitng t6 trang thai 2 twang ting vdi trudng hop {(k) nhan gid tri lon nhét ( hoae nhé mhét ) dé. 5) Tit d6 thu duge dpem. Ham roi rac f(k) edn duce oi la ham dae tring ct bai toén, Si tén tai Max (Min) ctia f(k) thueng duoc chiing minh dua vdo nguyén ly cue han. 6.5 Vidu Vidul. Trén mat phiing cho 2004 diém phan biét. Chiing minh ring 06 thé dung cde doan thing néi 2 diém mét vdi nhau sao cho 1002 doan thing dugc dung dai mét khong 6 diém chung, Loi gidi. Néi hai diém mot udi hau b4i cdc doan thang, Néu cdc dogn thing vita ding doi mot Khong c6 diém chung thi bai todn dé duoc chiing minh. 145 Néu cé hai doan thing, chéng han AB va CD cét nhau tai O (tem Hinh 1). Hinh 1 Goi f la téng do dai ode doan thing duge néi. Khi dé, néu ta thay hai doan thing AB ; CD bdi hai doan dit nét AC ; BD thi do AB + CGD > AC + BD (2) nén tong 6 dai céc doan thing duge néi khi dé sé gidm (do céc doan thing khéc van duoc git nguyén). Ma tap gid tri ctia f la hitu han (2) nén sau mét $6 hitu han lan thay déi lai céc dogn théng nhu vay f sé dat gid tri nhd nhdt. Khi f dat gid tri nhé nhdt thi khong 26 hai doan thing duoc ndi nao cit nhau v néu cb hai dogn thdng cit nhau thi ta lai tigp tuc lam nhu trén va sé thu duoc trang théi ma trong dé f dat gid tri nhé hon, trai v6i gid thiét la f mhd mhét. Vay 06 thé dung duge 1002 doan thdng thod man yéu cdu bai ra (dpcm). 6.6 Bai tap luyén tap Bai toan 1. 1 (D7) Cho mt ban cd cd 7 x 7. Mot quan ma dig d 6 (1;4) vd di theo dudng chéo hinh chit nhat 2x 3. Héi cé tin tai hay khong mét dudng di cia quan ma lién tiép qua tat cd cdc 6 ciia bin co méi 6 mét én hay khong? Dép 96: Khong. Té mau nhu bin ed quéc té va sti dung tinh chdn, Ié (cb 25 6 den va 24 6 tring. Ban dau, quan ma diing 6 6 tring.) Bai toén 2. 2 (D71) Trén matt phiing chon ducing théng trong dé khong cb hai dating no song song vd khéng 06 ba duéng ndo déng guy. chiing minh trong méi mién mat phéing ma cdc dudng thiing a6 chia ra cd thé dat mot sé nguyén € (—n;n) \ {0} sao cho téng céc sé dude viét 6 méi phia ciia mét dudng thang bat k) luén bing 0. Bai todn 3. 3 (D78) Trén ban cd quéc té dat 8 quan tét sao cho d méi hang va 6 méi cbt déu 06 diing mot quan t6t, Chiing mink ring 56 quan t6t nim d céc 6 den la sé chin. 146 Bai toan 4. 4 (D85) Trén méi canh ciia ta dign ABCD ldy mot diém bat ky. Qua méi cp ba diém thudc ba canh cing cudt phat ti mét dinh ctia td dién dung mot mat phdng. Chiing minh ring néu ba trong s6 bén mat phing thu duge ma tiép mic vdi cdu noi tiép tit dign thi mat phéng thit tu cing tiép mic vii cdu dé. Bai toan 5. 5 (D70) Co 4m déng cu, trong dé cé mot mita la cic déng zu gid. Céc déng tu that od kidi luong bing nhau, cdc déng cu gid ciing cd khbi lugng bing nhaw nhung mhe hon. ching mink bing khong qué 3m phép can dia ( khong cé qud cin ) ta cb thé nic dinh duge tdt od céc déng zu gid. Bai ton 6. 6 (D71) Trén mang ludi nguyén dénh déu ode miit ndo dé ( 36 mit duoc dénh déu > 1 ). Cho mét tap hitu han céc vecto cé toa do nguyén. Biét ring néu dat diém adc ctia cée vecto dé tai mot diém mit duge dank dau bét kj thi 56 diém ngon duge dénh déu nhiéu hon sé diém ngon khéng duoc dank déu. Chitng minh ring c6 v6 sé diém mit duoc dénh déu. Bai toan 7. 7 (D 73) Cho diém Ag vin ecto @i, S, @, ---, TH o6 téng bing D. M6i hoén vi (B1, ba, b3, +++, bn) ctia ode vecto dé zie dink mot tap céc diém Ay Aa, +++, An = Ag sao cho 0 = AA, b= Ah, B= BR, R= aR, Chitng minh ring tén tai mot hodn vi ctia cde vecta da cho sao cho t&t cd cic diém At, Aa, s+, An1 déu nim trong mién géc cé s6 do béing 60° udi dink la Ao (mién géc duge ké cd canh cita géc). Bai toan 8. 8 (D 72) Trén bd mét con song thing dat hitu han cdc stic g6 khéng chéng len mhau va méi stic 96 tao vi bd sdng géc < 45°. Ching mink ring ta cé thé lan leet lan tét ed céc stic 96 dé audng séng sao cho khi lm khong cb stic g5 ndo va cham vdi che stic 96 khée. (Céc stic 96 chi duge lain theo phuong vudng géc véi nd vd khong duce quay.) Bai todn 9. 9 (D 69) Ching minh ring néu trén mat phing cé mot sé hitu han cdc da gidc (6i dat tuy yj, khong cé diém chung thi véi moi tia Ox luon tén tai mét da gide ma Ahi tinh tién song song né theo hudng cia tia sé khong cham vao céc da gide khéc. Bai todn 10. 10 (D 65) Trén ban ci quéc té dat céc déu +, — mét cach tuy ¥ vdo cée 6. Duge phép déi déu déng thai 6 15 6 thudc ciing mét hang va cling mét cot. Héi cé thé Jam nhw vay dé sau mot sé hitu han bude, tix mot ban od di cdc déu duoc dat bat ky thu duce mét ban cd vdi cde wi tri ede dau dé cho trude hay khéng?. 147 7 Cuc tri trén tap rdi rac 7.1 Phuong phdp xay dung cAu tric cia diém ma tai d6 dat Min, Max Dy dodn Min, Max ctia biéu thite f(z) dat tai x* nao dé va ching minh dy doén 6 Trong phuong phép nay thudng ta phdi sit dung cdc hién thite cla Sé hoc, Té hop va Toén rii rac. Viée du dodn thudng duge thc hién béi phép quy nap dia vdo mét sé tink todn ban dau. 7.2 Vi du minh hoa Vidyl. Cho métn - gide Idi (n > 4), trong dé khong c6 ba dudng chéo nao déng quy tai mot diém ném trong da gide. Goi f(n) la 36 cdc dudng chéo cé thé ké duce sao cho cée phan ma ching cdt ra khéi da gide da cho déu la céc tam gide. Hay tac dink Max {(n) Lai gidi. Goi f(n) la 86 ee dudng chéo c6 thé ké duge thod man yéu céu bai ra. Xét mot sé trudng hgp ban déu cia n. Bang cach vé hinh cu thé ta dugc n= [45] 6] 7/8] 9] 0] [feys 2] s]sfe} ay oy i) 10) <[B]-4. Trong 46 ta ky hiéu [x] 14 s6 nguyen lin nhit khong wut qué x. Dé ching minh (*) trude hét ta. ching mink Bé dé: Néu tap S céc dudng chéo ciia mot n - gide di (n > 5) chia n - gide dé thank cdc tam gide thi cé it nhdt mot dudng chéo € S khong cit tt cd céc dudng chéo khde € S tai diém trong ciia n - gide dé cho. Ching minh: Xét cdc hinh vé sau Tit dé ta c6 dy dodn A B p-gide. D. Hinh 1. Hink 2. Hinh 8. 148, Tit cdc hinh vé trén ta c6 cde nhan xét hién nhién sau: 1), Khong 6 dang chéo nao € S oft > 2 duidng chéo € § khdc (tem Hinh 1). Do hai géc a; B cb a+ 8 > 180° ) 2) Cid sit moi dung chéo € S déu ct diing mot dung chéo € S. Xét hai dutong chéo € S la AC ; BD cit nhau tai O (xem Hinh 2)). Don > 5 nén cd it nhét mot canh ctia tt giée ABCD la dudng chéo ciia n - gide da cho. Gid stt dé la canh AB. Ta 6 AB € S wh néu khong thi phan n - gide udi bién chita dudng gdp khiic AOB sé khong la tam gide nén sé cé mot dudng chéo € $ cdt AB, khi dé dudng chéo dé con ct hode AC, hoc BD. Diéu do tréi voi nhan 2ét 1) (CO mot dong chéo € S cht hai duang chéo € S) Tit hai nhan cét tren suy ra két ludn ctia bb dé Trd lai bai todn dang xét. Hién nhién, (*) da diing vdi n = 4;5;6. Gid stt (*) ding udi moi k, 5 6. Theo bd dé, tdn tai mot dudng chéo ¢ S khang oft cde duuing chéo Rhéc € S va chia n - gidc da cho thank hai phén rdi nhau. Gid st dung chéo d6 la. AyA, (nem Hink 8 ) va. né chia n - gide dé cho thanh p - gidc va q - gide (v6 34. Vay sé lén nhét cae dudng chéo ké dugc thod man yéu céu bai ra la [FI -4.8 Vi'du 2. (VMO 2003) Tim sé m € N én mhat sao cho hé phuong trinh sau cb nghi¢m nguy (+P + uf = (@ +2 +8 = (043) +42 = e+n)+y2 (1) 149 Lai gidi. Hign nhién néu hé (1) khong c6 nghiem (©; yr 5 yos--- 5 Um) nguyen khi thi n6 ciing khong cb nghiém nguyén vdi n > k. Vin = 3 dé thdy hé 06 nghiém: (2; m1; yo: ys) =(—2; 0; 1; 0) Ta sé chitng minh udi n = 4 thi hé (1) khong cd cic nghiém nguyen. That vay, gid sit tén tai céc $6 x 541 ; yo; ysis vs Z va thod man: (e+1P ty = (e+2? typ =(@ +3? +08 = (244? +93. (2) Tit (2), doz+1 vax+3 cing tink chén, 1é nén y va yp ciing tink chén, lé. Tuong tu, Yo vd ye cling tinh chén, Id Dox+1 vaz+2 khéc tinh chén, lé nén y: vd yp Khdc tinh chén, ld. Tuong ty, ys vd ya khde tink chin, le. Nhan zét ring VoiaeZ; a lé tia? Tes (2) ta 06. 1 (mod 8), cdn via € Z; a chén thi 2a? 0 (mod 8). (e+ 1) + yf + (e+ 3)? +43 = (a+ 2)? +8 oo AR=yP+yZ+2. (+) (e+ 2P +o + (ota) tuf = Ale 4 3) +u3 Weave + ye +2 (+x) Néu yp chin thi theo trén ta cd yiiys lé, va tir (*) ta duce: O=(1+142) (mod 8) : v6 Wy. Néu yp lé thi theo trén ta cé ys lé ; ya chén va tit (**) ta duce: 1+1+42) (mod 8) : v6 ly. Vay gid sit ban déu la sai, tic 1a vdi n = 4 thi he (1) khong 6 céic nghiém nguyen. Do dé n=3 la gid tri lin nhét dé hé phuong trinh da cho cé nghiém € Z. Vidu 3. Chon > 3 va céc 96 x1; 2)--- ;tn1 EN, bién thién vd thod man =n (Gite Foyt tone 2) 21 +229 +329 +--+ (n—1tq=2n-2 Tim Ming ui f = f(ayjr2}--- j2n-1) = )> kate. (2n — k) a Lai gidi. Vi tap gid tri ciia f la tap con ciia N nén tén tai Minf. Gid si Minf = f(x") = f(xfixgi-- 3x44). Tit diéu kién 2) suy ra a3, <2. bang xdy ra khi va chi khi 2 (theo 1) ) 150 D6 la diéu v6 ly vin > 3. Vay a1_, <1 2t_, € {0;1}. +) Néuxh_, =0 thi Daptapeapte tap yen ) 2) af +203 +323 +---+(n—2)z5_,=In-2 a) Névu3meT mJ, 2%, >0 cdnz}=0, Vigm thi ten we far. => mn = 2n—2 = n(Q—m)=2: wijdon>s b) Néu Bi, j, 1Bi+2 sao cho zy, 2 >0. Xét b6 96 2! = (xh32'5-+- ;24,4) xdc dink nhw sau {a8 ik ¢ (i; f:i+1;9 +1} wat 15 ti = hy B= aF— 1s tha = Ohya +1 DE dang kiém tra ring 09 cic 36 a! := (x4;2y;-+- ;2',_y) 2c dink nw vey thod man diéw ign (ii). Nhung ' PCB a3 Fa) = Fai ays eta) = = 20-4) >0 nén f khong dat Min tai 2” = (24; 25;--- ;23_,) Vay ta phat c6: Fi, xf, af, >0 vd at =0, VA Ai, i+ 1, Khi dé +c,=n i) + ‘ at => 24, = (2—i)n —-2. @ {& Ginn mo) aa ae) Dori, >O>i<2>i=l. Vayrp=n—-2, of =2 vd F(a") = f(2;n — 2;0;0;--- ;0) = 2(2n — 1) + 2(n — 2)(2n — 2) = 4n? - Bn +6. +) Néuat_, =0 thi We I ap +eptap+---ta%,=n-1 2) af +205 + 30h +--+ (m— Dor, Sah=a}=---=2'.,=0,2}=n-1 => f(a") = 1.(n — 1)(2n- 1) + (n= 1).L.(2n—n + 1) = 3n? — Bn. Nhung vdi moi n > 3 ta luon 06 (4n? — 8n +6) ~ (3n? — 3n) = (n — 3)(n—2) 20 nén ta duge két gud sau Minf = 3n? — 3n = f(n — 1;0;0;-+» 5051). Vidu4. Ham sé (f(n)) :N* > R née dink nhu sau n(nt]) $1) =0; fm) = s(G)+(-) Fn 22 1) Xéc dinh Min; Max ctia f(n) vdin < 1999 va fim tét cd cdc gid tri n € 1.1999 dé f(r) dat Min; Max. 2) Cé bao nhiéu gid trin € T1999 dé f(n) = 0. Lai gidi. Ta cd n(n +1) a, fr) - HB) Nhan zét ring trong hé co 36 2, néun = ajap- Geapaeaiag thd Gl=aa aes, Nogodi ra ee n=0;3 (mod 4) + aya; = 00 hose TL n=1;2 (mod 4) = ag-ay = OF hod 10. Nhe vay: F(n) tang 1 don vj d6i uti #(V%]) née = TG AGAGE voi ae = Ona TATE Ubi ay # Oe-1 f(n) gidm 1 don vp déi voi 4G) néun = apoE Kg hiéu u(n) 1a 36 cap 00 ; 11 cdn v(n) la s6 cp 01; 10 trong céch wiét theo ca 36 2 ctia 36 n. Ta sé chting mink J(n) =u(n)- on), Vn EN’ (+) Ta sé chitng mink (*) bing phuong phdp quy nap todn hoc theo k la s6 chit s6 trong céich vidt nhj phan ctia sé n. That vay, khik = 1=>n=1 tha f(1)=0; u(1) = v(1) = 0 => (+) ding vik =1 Néuk = 2 thi +) Hode +) Hoicn 102 = u(2) La = u(3) i u(2) i ¥(3) $(8) = f(1) +1 = 1 = $08) = ula) - v(9). con f(2) = -1 => f(2) = u(2) — v(2) Gid sit (*) diing t0i k, hay la (*) ding v6i moi sé n = Tay Oa. Xét m = Gay Oya. é my a Néu anys =O [F] =n, m+ Ue vt Fm) = f(r) + (rer. 152 Néu a, = 0 thin chén va u(m) = un) +1; v(m) = v(m); (-1)"™") = 1 = fm) = f(n) +1 Néu ay = 1 thin lé va (n) — v(n) +1 = u(m) =v ulm) = u(n) ; v(m) = o(n) 41; (-1)"™) = = f(m) = fin) - Néu ag¢1 = 0 thi lap lun tong ty ta cing duge f(m) = u(m) ~ v(m). Vay (*) cing diing toi k +1 nén theo nguyén Ii quy nap ta duge (*) diing di moin € N°. Trd Iai bai todn da cho 1) Tit (*) ta thdy dé f(n) lon nhét thi u(n) phdi nhé nhdt. Ma v(n) > 0 nén to phai tim sé n ln nhét, < 1999 sao cho v(n) = 0. Naan xét ring =u v(n) = 11 = u(m) = v(m) 111111111112 = 2047 > 1999 ; 11111111112 = 1023 < 1999 nén sé n l6n nhat € 1999 cd v(m) nhé mhdt la sé 1028. Khi dé f(n) =9-0=9, Vay | Max... f(n) =9 = f(1023). Dé f(n) nhé nhét ta phdi 06 w(n) mhé nhét, déng thei v(n) lon nhét. Lap luan twong ty trén ta duge s6 101010101012 = 1365 ta sé 06 u(n) la nhé nhat vd u(n) = 10 la lén mhét. Khi d6 f(n) = 0-10 = ~10. Vay Min. £(n) = -10 = f(1365) 2) Trong biéu dién n = Gar Gk-20.-1ap, gitta hai chit sé gidng nhaw ta dat déu ”.” con gitta hai chit s6 khdc mhau ta dat déu”,” Khi dé u(n) déu "” vd u(n) = 36 du *,”. Néu nhé rang chit sé déu tién phdi la chit s6 1 thi méi day gém m ky higu ”.” va ",” 38 xéc dink duy nhdt mot 55 cd m +1 chit sé trong hé co $6 2. 1g udi sd 1110112 Chéing han day (..,,.) sé Nh vay f(n) = 0 4 u(n) = v(n) 4 96 du "" bing $6 déu”,”. Te.dé > m chén. Voi m chén c6 Cf cach xép ; déu ”.” vao m ché . (Céc ché con lai dat cdc déu ”,”). M& 1999 < 100000000000, = 2048. Trong 2047 36 n < 2048 3 C+ Cp + CP + 08+ C$ + C8, = 351 86 c6 f(n) = 0. Nhung trong cae s6 06 f(n) = 0 dé c6 5 s6 > 1999 = 11111001111 1a cdc 36 11111010010, = 2002 ; 111110101002 = 2004 ; 11111010110) = 2006 11111011010, = 2010 ; 111111010102 = 2016. Vay 06 tdt cd 351 — 5 = 346 36 € T1909 a nghiém cia phuong trink f(n) = 0. Vi du 5. Choa >2 va ham sé a(n): NR ade dink nhu sau a(0) = 1, a(1) =a, a(n +1) = (oes 7 2) a(n), (ne N’) Voi k EN; dat ‘i : L(k) = ao +w® Chiing minh ring Supf(k) = 3 (2+a- va=1) te" Lai gidi. Ta cb A> 25 3b> Lambe teat 2av yd, a Ta phdi chting minh 18) <5 (24a Varma) =e (), VEEN wa 6 vé phdi ciia (1) khdng thé thay 9 béi s6 nhé hon. Tit dink nghia cla him sé va tit céch dat s6 b ta cé a = (se -2) a(0) = (a? ~2).a = ("+ z) (+3) ae (38 = 2) a(1) = (a? 2).a= ( S z) (* + Bing phuong phép quy nap ta ching mink dude (di M = (0? + 1)(b4 +1)-- (8 +1). Mat khde: 1 1 a 1 30+ 542-0 p)) =145 (Dob>1=b-}>0) Tu céc két qué tren ta 06 8 B BO 1 1 an ee and Mette Srey tt Brey ey 0, (i € Tm) ta ludn 06 ee ee @) " & (i+ a)0 a2) (a5) (1+ a1)(1 + a2) That vay, c6 S$, = —S4 - +. 5 (3) ding win =1 Re Tra l+a, ms Gid sti (3) diing dénn, khi dé dat P = (1 +a,)(1 +42) -+-(1 +a) ta 05 Ont Sntt = Sn t+ Bo 1 : P(L¥ anti) Vay (3) ciing diing wSi'n +1, tite la theo nguyén ty quy nap, (3) ding udi moin € N°. Theo (3), ta cd VI(2) = <15VT(2) <1, PANG + Fey Vay (2) diing nén (1) ding. Mat khéc , khi cho b> 1+ (es a —+ 2+) ta c6 VT(2) > 17 nén 6 VP(2) khong thé thay 1 béi sé nhé hon, hay la 6 VP(1) khéng thé thay 9 bdi sb nhé hon. Tém lai, ta dude Sups(k) =e= 5 (2+a-vaF=a) (Apem). hee Vidu6. Cho S, = {1; 2; cia song dnh p: Sy — Sy néu p(3) Got f(n) la s6 song anh khéng cd diém bat dong, con g(n) la sb song dnh c6 ding mot diém bat dong : Sy > Sq Tim gid tri lon nhét va gid tri mhé nhét ela H(n) véi +i nj. Phén tj € S, duoc goi la diém bét dong H(n) = [f(n) ~ 9(n)|, ne N?. Goi p la mot song anh : S, > Sy ma ed dring mot diém bdt dong j. C6 n céch chon 5 € S, va f(n—1) cach lap cde song dnh : Sy \ {3} + Sq \ {i} ma khong 6 diém bat dong. Vay g(n)=nf(n—1) (1), V¥n>2 Bay gid, goir la song anh: S, +S, ma khong 6 diém bit dong. Khi d6, r(1) = j voi j #1 vd c6m—1 cach chon j nhw vay. Néu r(5) = 1 thi 65 f(n — 2) céc song dnh : S, \ {155} + Sa \ (1s5} ma khong 6 diém bét dong. Néu 66 sung thém r(1) = 3, r(j) = 1 thi song énh r d6:S, > S, cting khong 6 diéin bét dong. Vay e6 (n — 1) f(n — 2) ec song dnh : Sy —+ Sy loai ni Néu r(j) #1 thi 6 f(n - 1) cde song anh : S,\ {1} —* Sn \ {1} ma khong 06 diém bat dong. Goi g la mét song énh mhu vay. Bang cdch dat Pi) = (4) néu q(i) # J pli) =1, p)=5 néu gli) = 35 ta cing duge song énh p khong c6 diém bit dong : Sp + Sq. Vay 06 (n ~ 1) f(n = 1) cdc song énh : Sq —+ Sy loai nay. Tom lai, ta cé J(n) Tit (1) va (2) ta dude voin > 2 thi f(r +1)—9(n+1) = nff(n) + f(r 1] - (n+ Dslr) = nf(n=1) = F(n) = gn) - Fn). Mé f(1) — 9(1) =0—1= —1 = f(2) — 9(2) = 1, Bing phuong phép quy nap ta ching minh duge —Uf(n—2)+(n-Vf(n-1) (2), Yn BB. F(n) = 9(n) = (-1)" + H(n) = |f(n) - 9(n)] VneN’ Te dé ta 06 Min H(n) = Max H(n) = 1. net nes Vii dy 7. Tren mat phing cho 2n +1 dudng thing (n € N‘). Chting eit nhau tao thinh cae tam gide. n(n +1)(2n +1) 6 Lai gidi. Goi sé aac tam gide nhon tao thanh la f(n). Ta phdi chting mink Chiing minh ring s6 cdc tam gide nhon tao thanh khong vvot qué n(n + 1)(2n +1) fin) < MM (), Vn en C6 thé gid sit trong 2n +1 dudng théng da cho khong cé hai dudng thing nao song song, khong o6 hai dudng thdng nao vudng géc vdi nhau vd khong c6 ba dudng théng néo déng quy. That vay, néu cd hai dudng thing nao song song hodc vudng géc thi ta chi vige quay chiing mét géc dti hd sao cho céc tam gide nhon wan la cdc tam gidc mhon, khi dé 6 cde tam gid nhon khong gidm. Néu cé ba dudng thing nao dang quy thi ta tinh tién song song mt ditdng véi khodng céch dii nkd, sé tam gid nhon cting khéng gidm. Nhu vay, ba dudng théng bat ky trong 36 cde dudng thing dé cho lun cit nhau va tao thank mét tam gide hodc nhon hode th. Goi g(n) la s6 cdc tam gidc tw. Ta goi mét tam gide tao bdi ba dudng thang a,b,c ndo dé la "gid nhon canh a” néu cdc géc chung canh a ciia tam gidc dé la céc géc nhon. Chon mot duéng thing | nao dé va coi né la truc hoanh. Cac dudng thang cin lai duge chia thank hai tap: Tap T+ gém cdc dudng thing vdi hé s6 géc duong va tap T~ gdm cde duéng thing vdi hé sé géc am. Hai dudng thang tao vdil mét tam giée "gid nhon” 156 néu mot dudng € T*, dudng kia € T~. Goi p la 86 dudng thing € T+, p la sé duong thing € T-. Khi dé p+q = 2n vi 36 tam gidc "gid nhon cank L” sé 1a pq. Nhé ring : ms (254) =n Nhung | cé thé la dudng thdng bét ky trong s6 2n +1 dudng thing da cho nén ta co sb cap (dudng thing | ; tam gide "gid nhon canh |”) sé 3 va cée 36 my;02;-++ ;tn-1 EN, dién thién vd thod man {? B+ tayte ton rey 2) ay + 2p + 845 +--+ (n= aaa = 2n—2 Tim Minf vdi f = f(y) x95--+ jtm-1) = D> kote. (2n — hk). Léi gidi Vi tap gid tri ciia f la tap con cia N nén tn tai Minf. Gid st Minf = f(x") = f(xjsx3i--- 3x41) Tit didu kien 2) suy ra ct, <2. Déu bing ray ra khi va chi khi ai sa}=- 2%) =04n=2 (theo 1)) Dé la diéu vo ly vin > 3. Vay 23, <1 = a4, € {051}. +) Néu xy, =0 thi Naltaptagt- ta an @ 2) at + 2u3 +323 +--+ + (n— 2) 157 a) Niu3mel.(n—D), 2, >0 cone 0, Vitm thi : Voli don>3, 5) Néu3i, j, 10. Xét 06 85 a! s= (at325;-- ;2!_1) ade dink nhu sau {a8 bhi k € {is j441,5 +1} Bat ls thy = thy 15 thay 15 wy =a $1 Dé dang kiém tra ring b6 ce 36 2! (@i3095-+- 3244) ade dink nhw vay thod man didu ign (ti). Nhung PSH Sta) — Fleaahi shy) = AG - )>0 nén.f khong dat Min tai a* = (2y;23;--- ;2 Vay ta phdi c6: 3%, 2}, xt, >0 va af = Wee (Uta an - iat + (i+ l)a,, = 2n-2 1) Vk Ai, +1. Khi dé in = (2—in-2. Dost, >O>i<2>i=1. Vayry=n—2, of = 20a F(x") = f(2;n — 2;0;0; 40) = 2(2n — 1) + 2(n — 2)(2n ~ 2) = 4n? — 8n +6. +) Néu ot, =0 thi (ye (Datta t te, 2) af + 25 + 305 +--+ (nary > ap=ap=- 0, zj=n-1 => f(a") = 1.(n—1)2n-1) + (n-1).1 (2n —n +1) = 3n? —3n. Nhung udi moin > 3 ta luén ob (4n? ~ 8n +6) - (3n? — 3n) = (n —3)(n—2) 30 ‘nén ta duge két qué sau Minf = 3n? ~3n = f(n — 1;0;0;--- ;0, 1), 158 5. 10. Bai tap tuong tu Cho z,y,z€N" ; 2+y+z=100. Tim Min, Maz eda P = cyz. Choz,yEN* ; 2-+y = 2003. Tim Min, Mas ciia P = xly! Choz,yeN* ; o+y = 2003. Tim Min, Mas cia P = 2! + y! Hay viét s6 2008 thanh téng ctia mét sé sé nguyén duang sao cho tich cia céc 86 duge viét la én nbét. Cho pi,Pa,-*+ Pn la cde 56 nguyén t6 khéc mhau, Chiing mink ring i L Inf |- )0 Hay tim wdc sé chung lén nhét ctia 20 36 tu nhién duong c6 téng bing 2002. Chon €N*. Cho tap hitu han. k $= {a1,02,-++ a4} via € (051); ra >n. Dat ds = Inf |n— > al xes| cx | Tim Macds khi S bién thién, thod man |S| < +00 Tim sé nhé nhat cé dang:( k,l € Z) 2 As |i St] © B= |36* — 56) © C= |53* — 37'|, Mot doi vin nghé gém cd hoc sink nam va hoc sink nit. Hay tinh xem d6i vin nghé nay c6 ft nhdt bao nhiéu hoc sink trong cdc truing hop sau: a) Sé hoe sink nit cita doi nhé hon 50% nhung lén hon 40%. b) 36 hoc sink nit ctia doi mhd hon 44% nhung lon hon 43%. Trong mét gidi bong dé cé m déi tham gia theo thé thtée vdng tron mét luot. Doi thing dige hai diém, doi hoa duoc mét diém, con doi thua khéng duge diém. Sau gidi, ban t6 chite xép thet ty cic d6i theo sé diém dat dusc. Héi hai doi o6 vi thit ké nhau c6 thé hon kém nhaw nhiéu nhét bao nhiéu diém? 159

You might also like